Вы находитесь на странице: 1из 84

CONSTANTES

23 −1
Constante de Avogadro = 6,02 x 10 mol
4 4 4
Constante de Faraday (F) = 9,65 x 10 C mol−1 = 9,65 x 10 A s mol−1 = 9,65 x 10 J V−1 mol−1
Volume molar de gás ideal = 22,4 L (CNTP)
−19
Carga elementar = 1,602 x 10 C
−2 −1 −1 −1 −1 −1 −1 −1 −1
Constante dos gases (R) = 8,21 x 10 atm L K mol = 8,31 J K mol = 62,4 mmHg L K mol = 1,98 cal K mol
−2
Constante gravitacional (g) = 9,81 m s

DEFINIÇÕES
−2
Pressão = 1 atm = 760 mmHg = 101325 Nm = 760 Torr
−2
1 N = 1 kg m s
Condições normais de temperatura e pressão (CNTP): 0 oC e 760 mmHg
Condições ambientes: 25 oC e 1 atm.
Condições-padrão: 25 oC, 1 atm, concentração das soluções: 1 mol L−1 (rigorosamente: atividade unitária das espécies),
sólido com estrutura cristalina mais estável nas condições de pressão e temperatura em questão.
(s) ou (c) = sólido cristalino; (l) ou ( A ) = líquido; (g) = gás; (aq) = aquoso; (graf) = grafite; (CM) = circuito metálico;
(conc) = concentrado; (ua) = unidades arbitrárias; [A] = concentração da espécie química A em mol L−1.

MASSAS MOLARES

Elemento Químico Número Atômico Massa Molar Elemento Químico Número Atômico Massa Molar
(g mol−1) (g mol−1)
H 1 1,01 Cl 17 35,45
Be 4 9,01 K 19 39,10
B 5 10,81 Ca 20 40,08
C 6 12,01 Fe 26 55,85
N 7 14,01 Cu 29 63,55
O 8 16,00 Zn 30 65,39
F 9 19,00 As 33 74,92
Na 11 22,99 Br 35 79,91
Mg 12 24,31 Ag 47 107,87
Al 13 26,98 Cd 48 112,41
P 15 30,97 Sn 50 118,71
S 16 32,06 I 53 126,90
Pt 78 195,08

Questão 1. Considere a equação química, não balanceada, que representa a reação do sulfeto de cádmio em
solução aquosa de ácido nítrico:

CdS + HNO3 → Cd(NO3)2 + NO + Y + H2O

Pode-se afirmar que, na equação química não balanceada, a espécie Y é

A ( ) Cd(HSO4)2 B ( ) CdSO4 C ( ) SO3 D ( ) SO2 E( ) S

Questão 2. Considere as reações químicas representadas pelas equações abaixo:

I. H3CCHCH2 + HI → H3CCHICH3
II. H3CCOOH + NaOH → H3CCOONa + H2O
III. LiAlH4 + 4(H3C)2CO + 4H2O → 4(H3C)2CHOH + LiOH + Al(OH)3
IV. C6H6ONa + CH3CH2Cl → C6H6OCH2CH3 + NaCl
V. H3CCH2OH + HCl → H3CCH2Cl + H2O

Assinale a opção que apresenta as equações químicas que configuram reações de óxido-redução.

A ( ) Apenas I e II B ( ) Apenas I e III C ( ) Apenas II e IV


D ( ) Apenas III e IV E ( ) Apenas V
Questão 3. Uma amostra de um ácido dicarboxílico com 0,104 g de massa é neutralizada com 20 cm3 de uma
solução aquosa 0,1 mol L−1 em NaOH. Qual das opções abaixo contém a fórmula química do ácido constituinte da
amostra?

A ( ) C2H2O4 B ( ) C3H4O4 C ( ) C4H4O4 D ( ) C4H6O4 E ( ) C5H8O4

Questão 4. Carbamato de amônio sólido (NH2COONH4) decompõe-se em amônia e dióxido de carbono, ambos
gasosos. Considere que uma amostra de carbamato de amônio sólido esteja em equilíbrio químico com CO2(g) e
NH3(g) na temperatura de 50 0C, em recipiente fechado e volume constante. Assinale a opção CORRETA que
apresenta a constante de equilíbrio em função da pressão total P, no interior do sistema.

A( ) 3P B ( ) 2 P2 C ( ) P3 D ( ) 2/9 P2 E ( ) 4/27 P3

Questão 5. Considere cinco frascos contendo, cada um, uma solução aquosa saturada de sulfato de cálcio em
equilíbrio químico com seu corpo de fundo. A cada um dos cinco frascos é adicionada uma solução aquosa
saturada, sem corpo de fundo, de um dos seguintes sais, respectivamente:

I. CaSO4 II. CaCl2 III. MgSO4 IV. NaCl V. KNO3

Assinale a opção que indica os sais cujas soluções aquosas saturadas aumentam a massa do sulfato de cálcio sólido
nos frascos em que são adicionadas.

A ( ) Apenas I e II B ( ) Apenas I e IV C ( ) Apenas II e III


D ( ) Apenas III e IV E ( ) Apenas IV e V

Questão 6. Um frasco contém uma solução aquosa de brometo de sódio e outro frasco, uma solução aquosa de
ácido clorídrico saturada nos gases componentes do ar atmosférico. O conteúdo de cada um dos frascos é
misturado e ocorre uma reação química. Qual das opções abaixo contém a equação química que melhor representa
a reação acima mencionada?

A( ) 2 Cl–(aq) + 2 H+(aq) + ½ O2(g) → H2O( A ) + Cl2(g)


B( ) 4 Br–(aq) + O2(g) + 4 H+(aq) → 2 Br2( A ) + 2 H2O( A )
C( ) Cl–(aq) + 3/2 O2(g) + H+(aq) → HClO3(aq)
– +
D( ) 2 Br (aq) + 2 H (aq) → Br2( A ) + H2(g)
E( ) 2 Cl–(aq) + H2O( A ) + ½ O2(g) → 2 OH–(aq) + Cl2(g)

Questão 7. Assinale a opção CORRETA que corresponde à variação da concentração de íons Ag+ provocada pela
adição, a 25 oC, de um litro de uma solução 0,02 mol L−1 em NaBr a um litro de uma solução aquosa saturada em
AgBr. Dado: KpsAgBr(298K) = 5,3 x 10−13.

A ( ) 3 x 10−14 B ( ) 5 x 10−11 C ( ) 7 x 10−7 D ( ) 1 x 10−4 E ( ) 1 x 10−2

Questão 8. O processo físico de transformação do milho em pipoca pode ser um exemplo de reação química. Se
for assim entendido, qual é a ordem dessa reação, considerando um rendimento do processo de 100%?

A ( ) zero B ( ) um C ( ) dois D ( ) três E ( ) pseudozero


Questão 9. A reação hipotética A(s) + B(aq) → C(g) + D(aq) + E( A ) é autocatalisada por C(g). Considerando que
essa reação ocorre em sistema fechado, volume constante e sob atmosfera inerte, assinale a opção que apresenta a
curva que melhor representa a variação da massa de A(s), mA, em função do tempo, desde o início da reação até
imediatamente antes do equilíbrio químico ser estabelecido dentro do sistema.

mA mA
A( )
B( )

tempo tempo

mA
mA

mA
C( ) D( ) E( )

tempo tempo tempo

Questão 10. Dois recipientes contêm volumes iguais de dois líquidos puros, com calores específicos diferentes. A
mistura dos dois líquidos resulta em uma solução ideal. Considere que sejam feitas as seguintes afirmações a
respeito das propriedades da solução ideal resultante, nas condições-padrão e após o estabelecimento do equilíbrio
químico:

I. A temperatura da solução é igual à média aritmética das temperaturas dos líquidos puros.
II. O volume da solução é igual à soma dos volumes dos líquidos puros.
III. A pressão de vapor da solução é igual à soma das pressões parciais de vapor dos líquidos constituintes da
mesma.

Assinale a opção CORRETA que contém a(s) propriedade(s) que é (são) apresentada(s) pela solução resultante.

A( ) Apenas I e II
B( ) Apenas I e III
C( ) Apenas II
D( ) Apenas II e III
E( ) Apenas III

Questão 11. Uma tubulação de aço enterrada em solo de baixa resistividade elétrica é protegida catodicamente
contra corrosão, pela aplicação de corrente elétrica proveniente de um gerador de corrente contínua. Considere os
seguintes parâmetros:

I. Área da tubulação a ser protegida: 480 m2;


II. Densidade de corrente de proteção: 10 mA/m2

Considere que a polaridade do sistema de proteção catódica seja invertida pelo período de 1 hora. Assinale a opção
CORRETA que expressa a massa, em gramas, de ferro consumida no processo de corrosão, calculada em função
de íons Fe2+(aq). Admita que a corrente total fornecida pelo gerador será consumida no processo de corrosão da
tubulação.

A ( ) 1 x 10−3 B ( ) 6 x 10−2 C ( ) 3 x 10−1 D( ) 5 E ( ) 20


Questão 12. Considere um elemento galvânico formado pelos dois eletrodos (I e II), abaixo especificados e
mantidos separados por uma ponte salina:

- Eletrodo I: chapa retangular de zinco metálico parcialmente mergulhada em uma solução aquosa 1,0 x 10−3 mol
L−1 de cloreto de zinco;
- Eletrodo II: chapa retangular de platina metálica parcialmente mergulhada em uma solução aquosa de ácido
clorídrico de pH = 2, isenta de oxigênio e sob pressão parcial de gás hidrogênio de 0,5 atm.

Assinale a opção CORRETA que expressa o valor calculado aproximado, na escala do eletrodo padrão de
hidrogênio (EPH), da força eletromotriz, em volt, desse elemento galvânico atuando à temperatura de 25 oC,
sabendo-se que log 2 = 0,3 e E o 2+ = − 0,76 V (EPH).
Zn / Zn

A ( ) 0,54 B ( ) 0,64 C ( ) 0,74 D ( ) 0,84 E ( ) 0,94

Questão 13. 300 gramas de gelo a 0 oC foram adicionados a 400 gramas de água a 55 oC. Assinale a opção
CORRETA para a temperatura final do sistema em condição adiabática.
Dados: calor de fusão do gelo = 80 cal g−1; calor específico do gelo = 0,50 cal g−1 K−1; calor específico da água
líquida = 1 cal g−1 K−1.

A( ) − 4 oC B( ) −3
o
C C ( ) 0 oC D ( ) + 3 oC E ( ) + 4 oC

Questão 14. Assinale o valor da constante de equilíbrio, nas condições-padrão, da reação química descrita pela
seguinte equação:
Sn2+(aq) + 2 Fe3+(aq) Sn4+ (aq) + 2 Fe2+ (aq)

Dados eventualmente necessários: Potenciais de eletrodo em relação ao eletrodo padrão de hidrogênio nas
condições-padrão:

E oFe2+ / Fe = − 0,44 V E oFe3+ / Fe = − 0,04 V E oFe3+ / Fe2+ = 0,76 V E Sn


o
4+ / Sn 2+ = 0,15 V

A ( ) 1021 B ( ) 1018 C ( ) 1015 D ( ) 1012 E ( ) 109

Questão 15. Qual das opções abaixo apresenta o elemento químico que é utilizado como dopante para a confecção
do semicondutor tipo-p?

A ( ) Boro B ( ) Fósforo C ( ) Enxofre D ( ) Arsênio E ( ) Nitrogênio

Questão 16. O explosivo plástico conhecido como PBX é constituído de uma parte polimérica, normalmente um
poliuretano. A formação do poliuretano é atribuída à reação entre um poliol com

A( ) um isocianato.
B( ) uma amina.
C( ) uma anilina.
D( ) uma estearina.
E( ) uma oleína.

Questão 17. Assinale a opção que contém o polímero que, por ser termoplástico e transparente, pode ser
empregado na fabricação de pára-brisa de aeronaves.

A ( ) polietileno B ( ) polipropileno C ( ) poli(tetrafluoroetileno)


D ( ) policarbonato E ( ) poli(álcool vinílico)
Questão 18. Considere que os quatro processos químicos, descritos a seguir nos itens I a IV, são realizados
isobárica e isotermicamente:

I. KNO3(s) → K+(aq) + NO3–(aq)


II. H2O( A ) → H2O(g)
III. C(grafita) → C(diamante)
IV. 2 Na(s) + ½ O2(g) → Na2O(s)

Qual das opções abaixo contém os processos químicos cuja variação de energia interna é nula?

A ( ) Apenas I e II B ( ) Apenas I, II e III C ( ) Apenas II e III


D ( ) Apenas III e IV E ( ) Nenhum processo

Questão 19. Assinale a opção ERRADA que apresenta (em kJ/mol) a entalpia padrão de formação (ΔHf) da
substância a 25 oC.

A ( ) ΔHf (H2(g)) = 0 B ( ) ΔHf (F2(g)) = 0 C ( ) ΔHf (N2(g)) = 0


D ( ) ΔHf (Br2(g)) = 0 E ( ) ΔHf (Cl2(g)) = 0

Questão 20. Qual das substâncias abaixo não é empregada na fabricação da pólvora negra?

A ( ) trinitrotolueno B ( ) enxofre C ( ) carvão


D ( ) nitrato de sódio E ( ) nitrato de potássio

AS QUESTÕES DISSERTATIVAS, NUMERADAS DE 21 A 30, DEVEM SER RESPONDIDAS NO


CADERNO DE SOLUÇÕES.

Questão 21. Considere as seguintes moléculas no estado gasoso: OF2, BeF2, AlCl2 e AlS2.
a) Dê as estruturas de Lewis e as geometrias moleculares de cada uma das moléculas.
b) Indique as moléculas que devem apresentar caráter polar.

Questão 22. Um cilindro provido de pistão móvel, que se desloca sem atrito e cuja massa é desprezível, foi
parcialmente preenchido com água líquida. Considere que o sistema atinge o equilíbrio químico à temperatura T e
pressão Pi. Num dado momento, o sistema é perturbado por uma elevação brusca do pistão, atingindo novo
equilíbrio a uma pressão Pf e à mesma temperatura T. Considere que água líquida permanece no sistema durante
todo o processo.

a) Esboce um gráfico da pressão interna no interior do cilindro versus tempo considerando o intervalo de
tempo compreendido entre os dois equilíbrios químicos. Indique no gráfico as pressões Pi e Pf.
b) A pressão final, Pf, será maior, menor ou igual à pressão inicial, Pi? Justifique.

RT
Questão 23. A equação Π = C + bC 2 é uma
Pressão osmótica/Concentração de soluto (atm.dm /g)

0,0750
M
3

expressão semi-empírica utilizada para a determinação 0,0745

de massas molares de solutos, M, presentes em soluções 0,0740

reais. Nesta fórmula, Π é a pressão osmótica, em atm; 0,0735

C, a concentração de soluto, em g/dm3; R, a constante 0,0730

universal do gases; T, a temperatura da solução e b, 0,0725

uma constante. O gráfico ao lado mostra valores 0,0720

experimentais de Π/C versus C para uma solução 0,0715

aquosa a 20 °C de um soluto desconhecido. Determine 0,0710

o coeficiente linear do gráfico e, com esse valor, 0,0705

determine a massa molar do soluto. 20 30 40


3
50

Concentração de soluto (g/dm )


Questão 24. Em um laboratório, a 20 °C e utilizando um sistema adequado, H2(g) foi obtido através da reação
entre uma amostra de uma liga de 0,3 g de magnésio e um litro de uma solução aquosa 0,1 mol L −1 em HCl. Um
manômetro indicou que a pressão no interior do recipiente que contém o H2(g) era de 756,7 Torr. Sabendo-se que a
pressão de vapor d’água a 20 °C é 17,54 Torr e o volume de H2(g) obtido foi 0,200 L, determine a pureza da
amostra da liga de magnésio (massa de magnésio x 100/massa total da amostra), considerando que somente o
magnésio reaja com o HCl.

Questão 25. Apresente as respectivas fórmulas químicas estruturais das espécies químicas (A, B, C, D, E)
presentes nas seguintes equações químicas:
KOH (etanol)
CH3CH 2 CH 2 CA ⎯⎯⎯⎯⎯⎯⎯
→ A
KOH (e tan ol)
CH3CHCACH3 ⎯⎯⎯⎯⎯⎯⎯→ A
KOH (etanol)
CH3CH 2 CHCACH3 ⎯⎯⎯⎯⎯⎯⎯→ B + C
H SO
2 4→ 2 H O, calor
(H3C)2 CCH 2 ⎯⎯⎯⎯⎯ D ⎯⎯⎯⎯⎯⎯⎯
→ E

Questão 26. Dois cilindros (I e II) são providos de pistões, cujas massas são desprezíveis e se deslocam sem atrito.
Um mol de um gás ideal é confinado em cada um dos cilindros I e II. São realizados, posteriormente, dois tipos de
expansão, descritos a seguir:

a) No cilindro I, é realizada uma expansão isotérmica à temperatura T, de um volume V até um volume 2V, contra
uma pressão externa constante P.
b) No cilindro II, é realizada uma expansão adiabática, de um volume V até um volume 2V, contra uma pressão
externa constante P.

Determine os módulos das seguintes grandezas: variação da energia interna, calor trocado e trabalho realizado para
os dois tipos de expansão.

Questão 27. Uma chapa de ferro é colocada dentro de um reservatório contendo solução aquosa de ácido
clorídrico. Após um certo tempo observa-se a dissolução do ferro e formação de bolhas gasosas sobre a superfície
metálica. Uma bolha gasosa, de massa constante e perfeitamente esférica, é formada sobre a superfície do metal a
2,0 metros de profundidade. Calcule:

a) o volume máximo dessa bolha de gás que se expandiu até atingir a superfície do líquido, admitindo-se que a
temperatura é mantida constante e igual a 25 0C e que a base do reservatório está posicionada ao nível do mar.
b) a massa de gás contida no volume em expansão da bolha.

Sabe-se que no processo corrosivo que originou a formação da bolha de gás foram consumidos 3,0 x 1015 átomos
de ferro.
Dado: massa específica da solução aquosa de HCl é igual a 1020 kg m −3 na temperatura de 25 0C.

Questão 28. Suponha que um pesquisador tenha descoberto um novo elemento químico, M, de número atômico
119, estável, a partir da sua separação de um sal de carbonato. Após diversos experimentos foi observado que o
elemento químico M apresentava um comportamento químico semelhante aos elementos que constituem a sua
família (grupo).

a) Escreva a equação balanceada da reação entre o elemento M em estado sólido com a água (se ocorrer).
b) O carbonato do elemento M seria solúvel em água? Justifique a sua resposta.
Questão 29. Durante a realização de um estudo de corrosão, foi montado um sistema constituído por um elemento
galvânico com as seguintes características:

I. Anodo de ferro e catodo de platina;


II. Área de exposição ao meio corrosivo de ambos os eletrodos igual a 100,0 cm2;
III. Circuito eletrolítico mantido por ponte salina;
IV. Eletrodos interconectados por fio de cobre;
V. Eletrólito formado por solução aquosa ácida, livre de oxigênio atmosférico.

Considerando que ocorre perda de massa do eletrodo de ferro, calcule a corrente de corrosão (em ampère)
equivalente ao fluxo de elétrons no sistema, decorrente do processo de dissolução metálica, se esse metal
apresentar uma taxa de corrosão uniforme de 350 mdd.
mg
Dado: mdd = (miligrama por decímetro quadrado por dia, de ferro metálico corroído)
dm2 ⋅ dia

Questão 30. A reação de combustão 2SO2 (g) + O2(g) → 2SO3(g) é lenta e pode ser representada pela figura
abaixo:
Energia (Kcal/mol)

Caminho da reação

Esta mesma reação pode ser catalisada pelo NO2(g) em duas etapas, sendo que a primeira é bem mais lenta que a
segunda. Numa mesma figura, esboce o perfil da curva da reação não-catalisada e da reação catalisada pelo
NO2(g).
CONSTANTES
23 −1
Constante de Avogadro = 6,02 x 10 mol
4 −1 4 −1 4 −1 −1
Constante de Faraday (F) = 9,65 x 10 C mol = 9,65 x 10 A s mol = 9,65 x 10 J V mol
Volume molar de gás ideal = 22,4 L (CNTP)
−19
Carga elementar = 1,602 x 10 C
−2 −1 −1 −1 −1 −1 −1 −1 −1
Constante dos gases (R) = 8,21 x 10 atm L K mol = 8,31 J K mol = 62,4 mmHg L K mol = 1,98 cal K mol
−2
Constante gravitacional (g) = 9,81 m s

DEFINIÇÕES
−2
Pressão de 1 atm = 760 mmHg = 101325 N m = 760 Torr
−2
1 N = 1 kg m s
Condições normais de temperatura e pressão (CNTP): 0 oC e 760 mmHg
Condições ambientes: 25 oC e 1 atm.
−1
Condições-padrão: 25 oC, 1 atm, concentração das soluções: 1 mol L (rigorosamente: atividade unitária das espécies), sólido com
estrutura cristalina mais estável nas condições de pressão e temperatura em questão.
(s) ou (c) = sólido cristalino; (l) ou ( A ) = líquido; (g) = gás; (aq) = aquoso; (graf) = grafite; (CM) = circuito metálico;
−1
(conc) = concentrado; (ua) = unidades arbitrárias; [A] = concentração da espécie química A em mol L .

MASSAS MOLARES

Elemento Químico Número Atômico Massa Molar Elemento Químico Número Atômico Massa Molar
(g mol−1) (g mol−1)
H 1 1,01 Fe 26 55,85
He 2 4,00 Ni 28 58,69
Li 3 6,94 Cu 29 63,55
C 6 12,01 Zn 30 65,40
N 7 14,01 Ge 32 72,64
O 8 16,00 As 33 74,92
Ne 10 20,18 Br 35 79,90
Na 11 22,99 Kr 36 83,80
Mg 12 24,31 Ag 47 107,87
Al 13 26,98 Cd 48 112,41
Si 14 28,09 Sn 50 118,71
S 16 32,07 I 53 126,90
Cl 17 35,45 Xe 54 131,29
Ar 18 39,95 Cs 55 132,91
K 19 39,10 Ba 56 137,33
Ca 20 40,08 Pt 78 195,08
Cr 24 52,00 Pb 82 207,2
Mn 25 54,94 Ra 86 222

Questão 1. Uma mistura sólida é composta de carbonato de sódio e bicarbonato de sódio. A dissolução
completa de 2,0 g dessa mistura requer 60,0 mL de uma solução aquosa 0,5 mol L─1 de HCl. Assinale a opção
que apresenta a massa de cada um dos componentes desta mistura sólida.

A ( ) m Na 2 CO3 = 0, 4 g ; m NaHCO3 = 1, 6 g B ( ) m Na 2 CO3 = 0, 7 g ; m NaHCO3 = 1,3g


C ( ) m Na 2 CO3 = 0,9 g ; m NaHCO3 = 1,1g D ( ) m Na 2 CO3 = 1,1g ; m NaHCO3 = 0,9 g
E ( ) m Na 2 CO3 = 1,3g ; m NaHCO3 = 0, 7 g

Questão 2. No ciclo de Carnot, que trata do rendimento de uma máquina térmica ideal, estão presentes as
seguintes transformações:

A( ) duas adiabáticas e duas isobáricas.


B( ) duas adiabáticas e duas isocóricas.
C( ) duas adiabáticas e duas isotérmicas.
D( ) duas isobáricas e duas isocóricas.
E( ) duas isocóricas e duas isotérmicas.
Questão 3. Suponha que um metal alcalino terroso se desintegre radioativamente emitindo uma partícula alfa.
Após três desintegrações sucessivas, em qual grupo (família) da tabela periódica deve-se encontrar o elemento
resultante deste processo?

A ( ) 13 (III A) B ( ) 14 (IV A) C ( ) 15 (V A) D ( ) 16 (VI A) E ( ) 17 (VII A)

Questão 4. Um estudante mergulhou uma placa de um metal puro em água pura isenta de ar, a 25 °C, contida
em um béquer. Após certo tempo, ele observou a liberação de bolhas de gás e a formação de um precipitado.
Com base nessas informações, assinale a opção que apresenta o metal constituinte da placa.

A ( ) Cádmio B ( ) Chumbo C ( ) Ferro D ( ) Magnésio E ( ) Níquel

Questão 5. Qual o gráfico que apresenta a curva que melhor representa o decaimento de uma amostra contendo
10,0 g de um material radioativo ao longo dos anos?

A( ) 10,0
B( ) 10,00
10,
C( ) 10,0

Massa (g)
Massa (g)

Massa (g)

Tempo (anos) Tempo (anos) Tempo (anos)

D( ) 10,0 E( ) 10,0
Massa (g)

Massa (g)

Tempo (anos) Tempo (anos)

Questão 6. Num experimento, um estudante verificou ser a mesma a temperatura de fusão de várias amostras
de um mesmo material no estado sólido e também que esta temperatura se manteve constante até a fusão
completa. Considere que o material sólido tenha sido classificado como:

I. Substância simples pura III. Mistura homogênea eutética


II. Substância composta pura IV. Mistura heterogênea

Então, das classificações acima, está(ão) ERRADA(S)

A ( ) apenas I e II. B ( ) apenas II e III. C ( ) apenas III.


D ( ) apenas III e IV. E ( ) apenas IV.

Questão 7. Assinale a afirmação CORRETA a respeito do ponto de ebulição normal (PE) de algumas
substâncias.

A( ) O 1-propanol tem menor PE do que o etanol.


B( ) O etanol tem menor PE do que o éter metílico.
C( ) O n-heptano tem menor PE do que o n-hexano.
D( ) A trimetilamina tem menor PE do que a propilamina.
E( ) A dimetilamina tem menor PE do que a trimetilamina.
Questão 8. O diagrama temperatura (T) versus volume (V) representa
hipoteticamente as transformações pelas quais um gás ideal no estado 1 pode atingir 3
T
o estado 3. Sendo ΔU a variação de energia interna e q a quantidade de calor
trocado com a vizinhança, assinale a opção com a afirmação ERRADA em relação
às transformações termodinâmicas representadas no diagrama.
1 2
A ( ) ΔU12 = q 12 B ( ) ΔU13 = ΔU 23
C ( ) ΔU 23 = q 23 D ( ) ΔU 23 > ΔU12 V

E ( ) q 23 > 0

Questão 9. Considere os átomos hipotéticos neutros V, X, Y e Z no estado gasoso. Quando tais átomos
recebem um elétron cada um, as configurações eletrônicas no estado fundamental de seus respectivos ânions
são dadas por:

V − (g) : [gás nobre] ns 2 np6 nd10 (n + 1)s 2 (n + 1) p6


X − (g) : [gás nobre] ns 2 np6
Y − (g) : [gás nobre] ns 2 np6 nd10 (n + 1)s 2 (n + 1) p3
Z− (g) : [gás nobre] ns 2 np3

Nas configurações acima, [gás nobre] representa a configuração eletrônica no diagrama de Linus Pauling para o
mesmo gás nobre, e n é o mesmo número quântico principal para todos os ânions. Baseado nessas informações,
é CORRETO afirmar que

A( ) o átomo neutro V deve ter a maior energia de ionização entre eles.


B( ) o átomo neutro Y deve ter a maior energia de ionização entre eles.
C( ) o átomo neutro V deve ter maior afinidade eletrônica do que o átomo neutro X.
D( ) o átomo neutro Z deve ter maior afinidade eletrônica do que o átomo neutro X.
E( ) o átomo neutro Z deve ter maior afinidade eletrônica do que o átomo neutro Y.

Questão 10. Considere a reação de dissociação do N 2O4 (g) representada pela seguinte equação:
N 2O4 (g) U 2 NO2 (g)
Assinale a opção com a equação CORRETA que relaciona a fração percentual (α ) de N 2O4 (g) dissociado
com a pressão total do sistema (P) e com a constante de equilíbrio em termos de pressão (Kp).

Kp 4P + K p Kp
A( ) α = B( ) α = C( ) α =
4P + K p Kp 2P + K p
2P + K p Kp
D( ) α = E( ) α =
Kp 2+P

Questão 11. Considere a reação química representada pela seguinte equação:


4 NO 2 ( g ) + O 2 (g) → 2N 2O5 (g)
Num determinado instante de tempo t da reação, verifica-se que o oxigênio está sendo consumido a uma
velocidade de 2, 4 x10−2 mol L−1 s −1. Nesse tempo t, a velocidade de consumo de NO2 será de

A ( ) 6, 0 x10−3 mol L−1 s −1. B ( ) 1, 2 x10−2 mol L−1 s −1.


C ( ) 2, 4 x10−2 mol L−1 s −1. D ( ) 4,8 x10−2 mol L−1 s −1.
E ( ) 9, 6 x10−2 mol L−1 s −1.
Questão 12. O acidente nuclear ocorrido em Chernobyl (Ucrânia), em abril de 1986, provocou a emissão
radioativa predominantemente de Iodo-131 e Césio-137. Assinale a opção CORRETA que melhor apresenta os
respectivos períodos de tempo para que a radioatividade provocada por esses dois elementos radioativos decaia
para 1% dos seus respectivos valores iniciais. Considere o tempo de meia-vida do Iodo-131 igual a 8,1 dias e do
Césio-137 igual a 30 anos. Dados: ln 100 = 4,6 ; ln 2 = 0,69.

A ( ) 45 dias e 189 anos. B ( ) 54 dias e 201 anos.


C ( ) 61 dias e 235 anos. D ( ) 68 dias e 274 anos.
E ( ) 74 dias e 296 anos.

Questão 13. Assumindo um comportamento ideal dos gases, assinale a opção com a afirmação CORRETA.

A ( ) De acordo com a Lei de Charles, o volume de um gás torna-se maior quanto menor for a sua
temperatura.
B ( ) Numa mistura de gases contendo somente moléculas de oxigênio e nitrogênio, a velocidade média das
moléculas de oxigênio é menor do que as de nitrogênio.
C ( ) Mantendo-se a pressão constante, ao aquecer um mol de gás nitrogênio sua densidade irá aumentar.
D ( ) Volumes iguais dos gases metano e dióxido de carbono, nas mesmas condições de temperatura e
pressão, apresentam as mesmas densidades.
E ( ) Comprimindo-se um gás a temperatura constante, sua densidade deve diminuir.

Questão 14. Um estudante imergiu a extremidade de um fio de níquel-crômio limpo em uma solução aquosa de
ácido clorídrico e, a seguir, colocou esta extremidade em contato com uma amostra de um sal iônico puro. Em
seguida, expôs esta extremidade à chama azulada de um bico de Bunsen, observando uma coloração amarela na
chama. Assinale a opção que contém o elemento químico responsável pela coloração amarelada observada.

A ( ) Bário. B ( ) Cobre. C ( ) Lítio. D ( ) Potássio. E ( ) Sódio.

Questão 15. Considere os seguintes sais:

I. Al(NO3 )3 II. NaCl III. ZnCl2 IV. CaCl2

Assinale a opção que apresenta o(s) sal(is) que causa(m) a desestabilização de uma suspensão coloidal estável
de sulfeto de arsênio (As2S3) em água.

A ( ) Nenhum dos sais relacionados. B ( ) Apenas o sal I.


C ( ) Apenas os sais I e II. D ( ) Apenas os sais II, III e IV.
E ( ) Todos os sais.

Questão 16. Uma solução aquosa de um ácido fraco monoprótico é mantida à temperatura de 25 °C. Na
condição de equilíbrio, este ácido está 2,0 % dissociado. Assinale a opção CORRETA que apresenta,
respectivamente, os valores numéricos do pH e da concentração molar (expressa em mol L─1) do íon hidroxila
nesta solução aquosa. Dados: pKa (25 °C) = 4,0 ; log 5 = 0,7.

A ( ) 0, 7 e 5, 0 x10−14 B ( ) 1, 0 e 1, 0 x10−13
C ( ) 1, 7 e 5, 0 x10−13 D ( ) 2,3 e 2, 0 x10−12
E ( ) 4, 0 e 1, 0 x10−10

Questão 17. Foi observada a reação entre um composto X e uma solução aquosa de permanganato de potássio,
a quente, ocorrendo o aumento do pH da solução e a formação de um composto Y sólido. Após a separação do
composto Y e a neutralização da solução resultante, verificou-se a formação de um composto Z pouco solúvel
em água. Assinale a opção que melhor representa o grupo funcional do composto orgânico X.

A ( ) álcool B ( ) amida C ( ) amina D ( ) éster E ( ) éter


Questão 18. Nos gráficos abaixo, cada eixo representa uma propriedade termodinâmica de um gás que se
comporta idealmente.

I II III

Com relação a estes gráficos, é CORRETO afirmar que

A( ) I pode representar a curva de pressão versus volume.


B( ) II pode representar a curva de pressão versus inverso do volume.
C( ) II pode representar a curva de capacidade calorífica versus temperatura.
D( ) III pode representar a curva de energia interna versus temperatura.
E( ) III pode representar a curva de entalpia versus o produto da pressão pelo volume.

Questão 19. A 20 °C, a pressão de vapor da água em equilíbrio com uma solução aquosa de açúcar é igual a
16,34 mmHg. Sabendo que a 20 °C a pressão de vapor da água pura é igual a 17,54 mmHg, assinale a opção
com a concentração CORRETA da solução aquosa de açúcar.

A ( ) 7% (m/m) B ( ) 93% (m/m)


1
C ( ) 0,93 mol L─ D ( ) A fração molar do açúcar é igual a 0,07
E ( ) A fração molar do açúcar é igual a 0,93

Questão 20. Um elemento galvânico é constituído pelos eletrodos abaixo especificados, ligados por uma ponte
salina e conectados a um voltímetro de alta impedância.

Eletrodo I: fio de platina em contato com 500 mL de solução aquosa 0,010 mol L─1 de hidróxido de potássio;
Eletrodo II: fio de platina em contato com 180 mL de solução aquosa 0,225 mol L─1 de ácido perclórico
adicionado a 320 mL de solução aquosa 0,125 mol L─1 de hidróxido de sódio.

Admite-se que a temperatura desse sistema eletroquímico é mantida constante e igual a 25 °C e que a pressão
parcial do oxigênio gasoso (PO2 ) dissolvido é igual a 1 atm. Assinale a opção CORRETA com o valor
calculado na escala do eletrodo padrão de hidrogênio (EPH) da força eletromotriz, em volt, desse elemento
galvânico. Dados: E oO / H O = 1, 23V (EPH) ; E o − = 0, 40 V (EPH)
2 2 O 2 / OH

A ( ) 1,17 B ( ) 0,89 C ( ) 0,75 D ( ) 0,53 E ( ) 0,46

AS QUESTÕES DISSERTATIVAS, NUMERADAS DE 21 A 30, DEVEM SER RESPONDIDAS NO


CADERNO DE SOLUÇÕES.

Questão 21. Escreva a equação química balanceada da combustão completa do iso-octano com o ar
atmosférico. Considere que o ar é seco e composto por 21% de oxigênio gasoso e 79% de nitrogênio gasoso.

Questão 22. São fornecidas as seguintes informações relativas aos cinco compostos amínicos: A, B, C, D e E.
Os compostos A e B são muito solúveis em água, enquanto que os compostos C, D, e E são pouco solúveis. Os
valores das constantes de basicidade dos compostos A, B, C, D e E são, respectivamente, 1,0 x 10─3; 4,5 x 10─4;
2,6 x 10─10; 3,0 x 10─12 e 6,0 x 10─15.
Atribua corretamente os dados experimentais apresentados aos seguintes compostos:
2-nitroanilina, 2-metilanilina, 2-bromoanilina, metilamina e dietilamina.
Justifique a sua resposta.
Questão 23. A 25 °C, realizam-se estes dois experimentos (Exp I e Exp II) de titulação ácido-base medindo-se
o pH da solução aquosa em função do volume da base adicionada:

Exp I: Titulação de 50 mL de ácido clorídrico 0,10 mol L─1 com hidróxido de sódio 0,10 mol L─1.
Exp II: Titulação de 50 mL de ácido acético 0,10 mol L─1 com hidróxido de sódio 0,10 mol L─1.

a) Esboce em um mesmo gráfico (pH versus volume de hidróxido de sódio) a curva que representa a
titulação do Exp I e a curva que representa a titulação do Exp II. Deixe claro no gráfico os valores
aproximados do pH nos pontos de equivalência.
b) O volume da base correspondente ao ponto de equivalência de uma titulação ácido-base pode ser
determinado experimentalmente observando-se o ponto de viragem de um indicador. Em laboratório,
dispõem-se das soluções aquosas do ácido e da base devidamente preparados nas concentrações
propostas, de indicador, de água destilada e dos seguintes instrumentos: balão volumétrico, bico de
Bunsen, bureta, cronômetro, dessecador, erlenmeyer, funil, kitassato, pipeta volumétrica, termômetro e
tubo de ensaio. Desses instrumentos, cite os três mais adequados para a realização desse experimento.

Questão 24. Um elemento galvânico é constituído por uma placa de ferro e por uma placa de estanho, de
mesmas dimensões, imersas em uma solução aquosa 0,10 mol L─1 de ácido cítrico. Considere que esta solução:
contém íons ferrosos e estanosos; é ajustada para pH = 2; é isenta de oxigênio; e é mantida nas condições
ambientes. Sabendo-se que o ânion citrato reage quimicamente com o cátion Sn2+(aq), diminuindo o valor do
potencial de eletrodo do estanho, determine o valor numérico da relação entre as concentrações dos cátions
( )
Sn2+(aq) e Fe2+(aq), [Sn 2+ ] [Fe 2+ ] , a partir do qual o estanho passa a se comportar como o anodo do par
galvânico.
Dados: Potenciais de eletrodo em relação ao eletrodo padrão de hidrogênio nas condições-padrão:
E o Fe2 + / Fe = − 0, 44 V ; E oSn 2 + /Sn = − 0,14 V

Questão 25.
a) Considerando que a pressão osmótica da sacarose (C12H 22O11) a 25 °C é igual a 15 atm, calcule a
massa de sacarose necessária para preparar 1,0 L de sua solução aquosa a temperatura ambiente.
b) Calcule a temperatura do ponto de congelamento de uma solução contendo 5,0 g de glicose (C6H12O6 )
em 25 g de água. Sabe-se que a constante do ponto de congelamento da água é igual a 1,86 oC kg mol─1.
c) Determine a fração molar de hidróxido de sódio em uma solução aquosa contendo 50% em massa desta
espécie.

Questão 26. São dadas as seguintes informações:

I. O polietileno é estável até aproximadamente 340 °C. Acima de 350 °C ele entra em combustão.
II. Para reduzir ou retardar a propagação de chama em casos de incêndio, são adicionados retardantes de
chama à formulação dos polímeros.
III. O Al(OH)3 pode ser usado como retardante de chama. A aproximadamente 220 °C, ele se decompõe,
segundo a reação 2 Al(OH)3 (s) → Al2O3 (s) + 3H 2O(g) , cuja variação de entalpia (∆H) envolvida é
igual a 1170 J g─1.
IV. Os três requisitos de combustão de um polímero são: calor de combustão, combustível e oxigênio. Os
retardantes de chama interferem no fornecimento de um ou mais desses requisitos.

Se Al(OH)3 for adicionado a polietileno, cite um dos requisitos de combustão que será influenciado por cada
um dos parâmetros abaixo quando a temperatura próxima ao polietileno atingir 350 °C. Justifique
resumidamente sua resposta.

a) Formação de Al2O3(s)
b) Formação de H2O(g)
c) ∆H de decomposição do Al(OH)3
Questão 27. Sabendo que a constante de dissociação do hidróxido de amônio e a do ácido cianídrico em água
são, respectivamente, K b = 1, 76 x10−5 (pK b = 4, 75) e Ka = 6, 20 x10−10 (pK a = 9, 21) , determine a constante de
hidrólise e o valor do pH de uma solução aquosa 0,1 mol L─1 de cianeto de amônio.

Questão 28. Considere duas reações químicas (I e II) envolvendo um reagente X. A primeira (I) é de primeira
ordem em relação a X e tem tempo de meia-vida igual a 50 s. A segunda (II) é de segunda ordem em relação a
X e tem tempo de meia-vida igual à metade da primeira reação. Considere que a concentração inicial de X nas
duas reações é igual a 1,00 mol L─1. Em um gráfico de concentração de X (mol L─1) versus tempo (de 0 até
200 s), em escala, trace as curvas de consumo de X para as duas reações. Indique com I a curva que representa
a reação de primeira ordem e, com II, a que representa a reação de segunda ordem.

Questão 29. Um tanque de estocagem de produtos químicos foi revestido internamente com níquel puro para
resistir ao efeito corrosivo de uma solução aquosa ácida contida em seu interior. Para manter o líquido
aquecido, foi acoplado junto ao tanque um conjunto de resistores elétricos alimentados por um gerador de
corrente contínua. Entretanto, uma falha no isolamento elétrico do circuito dos resistores promoveu a
eletrificação do tanque, ocasionando um fluxo de corrente residual de intensidade suficiente para desencadear o
processo de corrosão eletrolítica do revestimento metálico.
Admitindo-se que a superfície do tanque é constituída por uma monocamada de níquel com densidade atômica
igual a 1,61 x 1019 átomos m─2 e que a área superficial do tanque exposta à solução ácida é de 5,0 m2, calcule:

a) a massa, expressa em gramas, de átomos de níquel que constituem a monocamada atômica do revestimento
metálico.
b) o tempo necessário, expresso em segundos, para que a massa de níquel da monocamada atômica seja
consumida no processo de dissolução anódica pela passagem da densidade de corrente de corrosão de
7, 0 μA cm −2 .

Questão 30. É descrita uma seqüência de várias etapas experimentais com suas respectivas observações:

I. Dissolução completa de um fio de cobre em água de bromo em excesso com formação de uma solução
azulada A.
II. Evaporação completa da solução A e formação de um sólido marrom B.
III. Aquecimento do sólido B a 500 °C, com formação de um sólido branco de CuBr e um gás marrom C.
IV. Dissolução de CuBr em uma solução aquosa concentrada de ácido nítrico, formando uma nova solução
azulada D e liberação de dois gases: C e E.
V. Evaporação da solução azulada D com formação de um sólido preto F e liberação de dois gases: E e G.
VI. Reação a quente do sólido F com hidrogênio gasoso e na ausência de ar, formando um sólido
avermelhado H e liberando água.

Baseando-se nesta descrição, apresente as fórmulas moleculares das substâncias B, C, E, F, G e H.


CONSTANTES
23 −1
Constante de Avogadro = 6,02 x 10 mol
4 −1 4 −1 4 −1 −1
Constante de Faraday (F) = 9,65 x 10 C mol = 9,65 x 10 A s mol = 9,65 x 10 J V mol
Volume molar de gás ideal = 22,4 L (CNTP)
−19
Carga elementar = 1,602 x 10 C
−2 −1 −1 −1 −1 −1 −1 −1 −1
Constante dos gases (R) = 8,21 x 10 atm L K mol = 8,31 J K mol = 62,4 mmHg L K mol = 1,98 cal K mol
−2
Constante gravitacional (g) = 9,81 m s

DEFINIÇÕES
−2
Pressão de 1 atm = 760 mmHg = 101 325 N m = 760 Torr
2 −2
1J = 1 N m = 1 kg m s
o
Condições normais de temperatura e pressão (CNTP): 0 C e 760 mmHg
o
Condições ambientes: 25 C e 1 atm.
o −1
Condições-padrão: 25 C, 1 atm, concentração das soluções: 1 mol L (rigorosamente: atividade unitária das espécies), sólido com
estrutura cristalina mais estável nas condições de pressão e temperatura em questão.
(s) = sólido. ( A ) = líquido. (g) = gás. (aq) = aquoso. (CM) = circuito metálico. (conc) = concentrado.
−1
(ua) = unidades arbitrárias. [A] = concentração da espécie química A em mol L .

MASSAS MOLARES

Elemento Químico Número Atômico Massa Molar Elemento Químico Número Atômico Massa Molar
−1
(g mol ) (g mol−1)
H 1 1,01 Cr 24 52,00
B 5 10,81 Fe 26 55,85
C 6 12,01 Ni 28 58,69
N 7 14,01 Cu 29 63,55
O 8 16,00 Zn 30 65,40
Na 11 22,99 Sr 38 87,62
P 15 30,97 Ag 47 107,87
S 16 32,07 I 53 126,90
Cl 17 35,45 W 74 183,84
Ar 18 39,95 Pt 78 195,08
K 19 39,10 Au 79 196,97
Ca 20 40,08 Pb 82 207,2

Questão 1. A figura ao lado apresenta a curva de aquecimento de 100 g de


uma substância pura genérica no estado sólido. Sabe-se que calor é fornecido 120 S
–1
a uma velocidade constante de 500 cal min . Admite-se que não há perda de 80 T
T/°C

calor para o meio ambiente, que a pressão é de 1 atm durante toda a 40


transformação e que a substância sólida apresenta apenas uma fase cristalina. Q
0
Considere que sejam feitas as seguintes afirmações em relação aos estágios R
-40 P
de aquecimento descritos na figura: 0 40 80 120 160
Tempo/min
I. No segmento PQ ocorre aumento da energia cinética das moléculas.
II. No segmento QR ocorre aumento da energia potencial.
III. O segmento QR é menor que o segmento ST porque o calor de fusão da substância é menor que o seu
calor de vaporização.
IV. O segmento RS tem inclinação menor que o segmento PQ porque o calor específico do sólido é maior que
o calor específico do líquido.

Das afirmações acima, está(ão) ERRADA(S):

A ( ) apenas I. B ( ) apenas I, II e III. C ( ) apenas II e IV.


D ( ) apenas III. E ( ) apenas IV.
Questão 2. Historicamente, a teoria atômica recebeu várias contribuições de cientistas.
Assinale a opção que apresenta, na ordem cronológica CORRETA, os nomes de cientistas que são apontados
como autores de modelos atômicos.

A( ) Dalton, Thomson, Rutherford e Bohr.


B( ) Thomson, Millikan, Dalton e Rutherford.
C( ) Avogadro, Thomson, Bohr e Rutherford.
D( ) Lavoisier, Proust, Gay-Lussac e Thomson.
E( ) Rutherford, Dalton, Bohr e Avogadro.

Questão 3. HCA (g) é borbulhado e dissolvido em um solvente X. A solução resultante é não-condutora em


relação à corrente elétrica. O solvente X deve ser necessariamente

A ( ) polar. B ( ) não-polar. C ( ) hidrofílico.


D ( ) mais ácido que HCA . E ( ) menos ácido que HCA .

–1
Questão 4. Uma solução aquosa de HCA 0,1 mol L foi titulada com uma
–1
solução aquosa de Na OH 0,1 mol L . A figura ao lado apresenta a curva de

Condutância
T
titulação obtida em relação à condutância da solução de HCA em função do R
volume de Na OH adicionado. S

Com base nas informações apresentadas nesta figura, assinale a opção Volume de NaOH
ERRADA.

+ − +
A ( ) Os íons responsáveis pela condutância da solução no ponto R são: H , CA e Na .
+ −
B ( ) Os íons responsáveis pela condutância da solução no ponto S são: Na e CA .
C ( ) A condutância da solução no ponto R é maior que no ponto S porque a mobilidade iônica dos íons
presentes em R é maior que a dos íons presentes em S.

D ( ) A condutância da solução em T é maior que em S porque os íons OH têm maior mobilidade iônica que

os íons CA .
E ( ) No ponto S, a solução apresenta neutralidade de cargas, no R, predominância de cargas positivas e, no
T, de cargas negativas.

Questão 5. Uma barra de ferro e um fio de platina, conectados eletricamente a um voltímetro de alta
–1
impedância, são parcialmente imersos em uma mistura de soluções aquosas de FeSO4 (1,0 mol L ) e HCA
isenta de oxigênio. Um fluxo de gás hidrogênio é mantido constante sobre a parte imersa da superfície da
platina, com pressão nominal (PH2 ) de 1,0 atm, e a força eletromotriz medida a 25 °C é igual a 0,292 V.
Considerando-se que ambos os metais são quimicamente puros e que a platina é o polo positivo do elemento
galvânico formado, assinale a opção CORRETA que apresenta o valor calculado do pH desse meio aquoso.
Dados: E o H + /H = 0, 000 V; E o Fe2 + /Fe0 = − 0, 440V
2

A ( ) 0,75 B ( ) 1,50 C ( ) 1,75 D ( ) 2,50 E ( ) 3,25

Questão 6. A seguinte reação não-balanceada e incompleta ocorre em meio ácido:

(Cr2O7 ) −2 + (C2O 4 ) −2 → Cr 3+ + CO 2

A soma dos coeficientes estequiométricos da reação completa e balanceada é igual a

A ( ) 11. B ( ) 22. C ( ) 33. D ( ) 44. E ( ) 55.


Questão 7. Considere os seguintes líquidos, todos a 25 °C:

I. Cu(NO3 ) 2 (aq) IV. CH3 (CH 2 ) 16 CH 2OH(A)


II. CS2 (A) V. HCA(aq)
III. CH3CO 2 H(aq) VI. C6 H 6 (A )

Assinale a opção que indica o(s) líquido(s) solúvel(eis) em tetracloreto de carbono.

A ( ) Apenas I, III e V B ( ) Apenas II, IV e VI C ( ) Apenas III


D ( ) Apenas IV E ( ) Apenas V

Questão 8. Considere o seguinte mecanismo de reação genérica:

A 4+ + B2+ → A3+ + B3+ (etapa lenta)


A 4+ + B3+ → A3+ + B4+ (etapa rápida)
C + + B4+ → C3+ + B2+ (etapa rápida)

Com relação a este mecanismo, assinale a opção ERRADA.

A ( ) A reação global é representada pela equação C + + 2 A 4+ → C3+ + 2 A3+ .


B( ) B2+ é catalisador.
C( ) B3+ e B4+ são intermediários da reação.
D( ) A lei de velocidade é descrita pela equação v = k[C+ ][A 4+ ] .
E( ) A reação é de segunda ordem.

Questão 9. A 25 °C e 1 atm, uma solução de água pura contendo algumas gotas de solução alcoólica de
indicador ácido-base azul de bromotimol apresenta coloração azulada. Nestas condições, certa quantidade de
uma substância no estado sólido é adicionada e a solução torna-se amarelada. Assinale a opção que apresenta a
substância sólida adicionada.

A ( ) Iodo. B ( ) Sacarose. C ( ) Gelo seco.


D ( ) Nitrato de prata. E ( ) Cloreto de sódio.

Questão 10. Em cinco béqueres foram adicionados 50 mL de uma solução de referência, que consiste de uma
solução aquosa saturada em cloreto de prata, contendo corpo de fundo, a 25 °C e 1 atm. A cada béquer, foram
adicionados 50 mL de uma solução aquosa diluída diferente, dentre as seguintes:

I. Solução de cloreto de sódio a 25 °C.


II. Solução de Glicose a 25 °C.
III. Solução de Iodeto de sódio a 25 °C.
IV. Solução de Nitrato de prata a 25 °C.
V. Solução de Sacarose a 50 °C.

Considere que o corpo de fundo permanece em contato com as soluções após rápida homogeneização das
misturas aquosas e que não ocorre formação de óxido de prata sólido. Nestas condições, assinale a opção que
indica a(s) solução(ões), dentre as acima relacionadas, que altera(m) a constante de equilíbrio da solução de
referência.

A ( ) Apenas I, III e IV B ( ) Apenas I e IV C ( ) Apenas II e V


D ( ) Apenas III E ( ) Apenas V
Questão 11. A 25 °C e 1 atm, uma amostra de 1,0 L de água pura foi saturada com oxigênio gasoso (O2) e o
sistema foi mantido em equilíbrio nessas condições. Admitindo-se comportamento ideal para o O2 e sabendo-se
–3 – –1
que a constante da Lei de Henry para esse gás dissolvido em água é igual a 1,3 x 10 mol L 1 atm , nas
condições do experimento, assinale a opção CORRETA que exprime o valor calculado do volume, em L, de O2
solubilizado nessa amostra.
–3 –3 –3
A ( ) 1,3 x 10 B ( ) 2,6 x 10 C ( ) 3,9 x 10
–2 –2
D ( ) 1,6 x 10 E ( ) 3,2 x 10
3
Questão 12. Um vaso de pressão com volume interno de 250 cm contém gás nitrogênio (N2) quimicamente
puro, submetido à temperatura constante de 250 °C e pressão total de 2,0 atm. Assumindo que o N2 se comporta
como gás ideal, assinale a opção CORRETA que apresenta os respectivos valores numéricos do número de
–3
moléculas e da massa específica, em kg m , desse gás quando exposto às condições de pressão e temperatura
apresentadas.
21 21 21
A ( ) 3,7 x 10 e 1,1 B ( ) 4,2 x 10 e 1,4 C ( ) 5,9 x 10 e 1,4
21 21
D ( ) 7,2 x 10 e 1,3 E ( ) 8,7 x 10 e 1,3

Questão 13. Um recipiente contendo gás hidrogênio (H2) é mantido à temperatura constante de 0 °C.
Assumindo que, nessa condição, o H2 é um gás ideal e sabendo-se que a velocidade média das moléculas desse
3 –1
gás, nessa temperatura, é de 1,85 x 10 m s , assinale a alternativa CORRETA que apresenta o valor calculado
da energia cinética média, em J, de uma única molécula de H2.
–24 –24 –21
A ( ) 3,1 x 10 B ( ) 5,7 x 10 C ( ) 3,1 x 10
–21 –18
D ( ) 5,7 x 10 E ( ) 2,8 x 10

Questão 14. Assinale a opção que apresenta a afirmação CORRETA sobre uma reação genérica de ordem zero
em relação ao reagente X.

A( ) A velocidade inicial de X é maior que sua velocidade média.


B( ) A velocidade inicial de X varia com a concentração inicial de X.
C( ) A velocidade de consumo de X permanece constante durante a reação.
D( ) O gráfico do logaritmo natural de X versus o inverso do tempo é representado por uma reta.
E( ) O gráfico da concentração de X versus tempo é representado por uma curva exponencial decrescente.

Questão 15. Uma solução aquosa saturada em fosfato de estrôncio [Sr3(PO4)2] está em equilíbrio químico à
–7 –1
temperatura de 25 °C, e a concentração de equilíbrio do íon estrôncio, nesse sistema, é de 7,5 x 10 mol L .
Considerando-se que ambos os reagentes (água e sal inorgânico) são quimicamente puros, assinale a alternativa
CORRETA com o valor do pK PS(25°C) do Sr3(PO4)2.
Dado: KPS = constante do produto de solubilidade.

A ( ) 7,0 B ( ) 13,0 C ( ) 25,0 D ( ) 31,0 E ( ) 35,0

−1
Questão 16. Sabe-se que a 25 °C as entalpias de combustão (em kJ mol ) de grafita, gás hidrogênio e gás
metano são, respectivamente: –393,5; –285,9 e –890,5. Assinale a alternativa que apresenta o valor CORRETO
da entalpia da seguinte reação:

C (grafita) + 2H 2 (g) → CH 4 (g)

A ( ) − 211,1 kJ mol−1 B ( ) − 74,8 kJ mol−1 C ( ) 74,8 kJ mol−1


D ( ) 136,3 kJ mol−1 E ( ) 211,1 kJ mol−1
Questão 17. Uma lâmpada incandescente comum consiste de um bulbo de vidro preenchido com um gás e de
um filamento metálico que se aquece e emite luz quando percorrido por corrente elétrica.
Assinale a opção com a afirmação ERRADA a respeito de características que o filamento metálico deve
apresentar para o funcionamento adequado da lâmpada.

A( ) O filamento deve ser feito com um metal de elevado ponto de fusão.


B( ) O filamento deve ser feito com um metal de elevada pressão de vapor.
C( ) O filamento deve apresentar resistência à passagem de corrente elétrica.
D( ) O filamento deve ser feito com um metal que não reaja com o gás contido no bulbo.
E( ) O filamento deve ser feito com um metal dúctil para permitir a produção de fios finos.

Questão 18. Em um processo de eletrodeposição de níquel, empregou-se um eletrodo ativo de níquel e um


eletrodo de cobre, ambos parcialmente imersos em uma solução aquosa contendo sais de níquel (cloreto e
sulfato) dissolvidos, sendo este eletrólito tamponado com ácido bórico. No decorrer do processo, conduzido à
temperatura de 55 °C e pressão de 1 atm, níquel metálico depositou-se sobre a superfície do eletrodo de cobre.
Considere que as seguintes afirmações sejam feitas:

I. Ocorre formação de gás cloro no eletrodo de cobre.


II. A concentração de íons cobre aumenta na solução eletrolítica.
III. Ocorre formação de hidrogênio gasoso no eletrodo de níquel.
IV. O ácido bórico promove a precipitação de níquel na forma de produto insolúvel no meio aquoso.

Com relação ao processo de eletrodeposição acima descrito, assinale a opção CORRETA.

A( ) Todas as afirmações são verdadeiras.


B( ) Apenas a afirmação IV é verdadeira.
C( ) Apenas a afirmação III é falsa.
D( ) Apenas as afirmações II e IV são falsas.
E( ) Todas as afirmações são falsas.

Questão 19. Considere duas reações químicas, mantidas à temperatura e pressão ambientes, descritas pelas
equações abaixo:

I. H 2 (g) + 1 2 O2 (g) → H 2O(g) ΙΙ. H 2 (g) + 1 2 O 2 (g) → H 2O(A)

Assinale a opção que apresenta a afirmação ERRADA sobre estas reações.

A( ) As reações I e II são exotérmicas.


B( ) Na reação I, o valor, em módulo, da variação de entalpia é menor que o da variação de energia interna.
C( ) O valor, em módulo, da variação de energia interna da reação I é menor que o da reação II.
D( ) O valor, em módulo, da variação de entalpia da reação I é menor que o da reação II.
E( ) A capacidade calorífica do produto da reação I é menor que a do produto da reação II.

Questão 20. Considere o composto aromático do tipo C6H5Y, em que Y representa um grupo funcional ligado
ao anel.
Assinale a opção ERRADA com relação ao(s) produto(s) preferencialmente formado(s) durante a reação de
nitração deste tipo de composto nas condições experimentais apropriadas.

A( ) Se Y representar o grupo –CH3, o produto formado será o m–nitrotolueno.


B( ) Se Y representar o grupo –COOH, o produto formado será o ácido m–nitro benzóico.
C( ) Se Y representar o grupo –NH2, os produtos formados serão o–nitroanilina e p–nitroanilina.
D( ) Se Y representar o grupo –NO2, o produto formado será o 1,3–dinitrobenzeno.
E( ) Se Y representar o grupo –OH, os produtos formados serão o–nitrofenol e p–nitrofenol.
AS QUESTÕES DISSERTATIVAS, NUMERADAS DE 21 A 30, DEVEM SER RESPONDIDAS NO
CADERNO DE SOLUÇÕES.
AS QUESTÕES NUMÉRICAS DEVEM SER DESENVOLVIDAS ATÉ O FINAL E O VALOR
ABSOLUTO DO RESULTADO DEVE SER APRESENTADO.

Questão 21. Determine o valor aproximado do pH no ponto de equivalência, quando se titula 25,0 mL de ácido
–1 –1 –5
acético 0,1000 mol L com hidróxido de sódio 0,1000 mol L . Sabe-se que log 2 = 0,3 e Ka = 1,8 x 10 .

Questão 22. Proponha um método de obtenção de sulfato de cobre anidro a partir de uma reação de
neutralização. Expresse as etapas para a sua obtenção por meio de equações químicas, indicando as condições
necessárias para que cada etapa seja realizada.

Questão 23. A nitroglicerina, C3H5(ONO2)3(A), é um óleo denso que detona se aquecido a 218 °C ou quando é
submetido a um choque mecânico. Escreva a equação que representa a reação química do processo, sabendo
que a reação de decomposição é completa, e explique porque a molécula é explosiva.

Questão 24. Foram realizadas duas experiências com dois ovos de galinha. Inicialmente, ambos foram imersos
em vinagre até a dissolução total da casca, que pode ser considerada constituída prioritariamente por carbonato
de cálcio. Os ovos envoltos apenas em suas membranas foram cuidadosamente retirados do vinagre e deixados
secar por um breve período. A seguir, um ovo foi imerso em água pura e, o outro, numa solução saturada de
sacarose, sendo ambos assim mantidos até se observar variação volumétrica de cada ovo.

a) Escreva a equação química balanceada que descreve a reação de dissolução da casca de ovo.
b) O volume dos ovos imersos nos líquidos deve aumentar ou diminuir? Explique sucintamente por que estas
variações volumétricas ocorrem.

Questão 25. Considere a curva de variação da energia potencial das


espécies A, B, C, D e E, envolvidas em uma reação química genérica, em
função do caminho da reação, apresentada na figura ao lado. Suponha que a
reação tenha sido acompanhada experimentalmente, medindo-se as
Energia

A+B
concentrações de A, B e C em função do tempo.

a) Proponha um mecanismo de reação para o processo descrito na figura, C D+E


indicando a reação global.
b) Indique a etapa lenta do processo e escreva a lei de velocidade da Caminho da reação
reação.
c) Baseado na sua resposta ao item b) e conhecendo as concentrações de A, B e C em função do tempo,
explique como determinar a constante de velocidade desta reação.

Questão 26. Dada a fórmula molecular C3H4CA2, apresente as fórmulas estruturais dos compostos de cadeia
aberta que apresentam isomeria geométrica e dê seus respectivos nomes.

Questão 27. Considere que certa solução aquosa preparada recentemente contém nitratos dos seguintes cátions:
Pb 2+ , Cu 2+ , Fe 2+ e Ag + .
Descreva um procedimento experimental para separar esses íons, supondo que você dispõe de placas polidas
dos seguintes metais puros: zinco, cobre, ferro, prata, chumbo e ouro e os instrumentos de vidro adequados.
Descreva cada etapa experimental e apresente todas as equações químicas balanceadas.
Dados:
E o Zn 2+ / Zn = − 0, 76 V E o Fe2+ / Fe = − 0, 44 V

E o Pb2 + / Pb = − 0,13V E oCu 2+ / Cu = 0,34 V

E o Ag + / Ag = 0,80 V E o Au 3+ / Au = 1, 40 V
Questão 28. Considere que as reações químicas representadas pelas equações não balanceadas abaixo ocorram
em condições experimentais apropriadas e que as espécies A, B, C, D, E e F representam os produtos destas
reações.

C6 H 6 O CH3 + HI → A + B
CH3 CH 2 I + Na → C + D
CH3 CH 2 I + CH3 CH 2 C ≡ C Na → E + F

Apresente as equações químicas balanceadas e os respectivos produtos.

Questão 29. Uma chapa metálica de cobre recoberta com uma camada passiva de óxido de cobre (I) é imersa
em um recipiente de vidro contendo água destilada acidificada (pH = 4) e gás oxigênio (O2) dissolvido, sendo a
temperatura e a pressão deste sistema iguais a 25 °C e 1 atm, respectivamente. Admitindo-se que a
–6 –1
concentração inicial de equilíbrio dos íons de cobre (II) na solução aquosa é de 10 mol L e, considerando
que, nessas condições, a camada de óxido que envolve o metal pode ser dissolvida:

a) Escreva a equação química balanceada da reação que representa o processo de corrosão do Cu2O(s) no
referido meio líquido com o O2(g) dissolvido.
b) Determine o valor numérico da pressão de oxigênio, expresso em atm, a partir do qual o Cu2O(s) apresenta
tendência termodinâmica de sofrer corrosão espontânea no meio descrito acima.
Dados: E oCu 2+ / Cu O = 0, 20 V; E oO2 / H 2O = 1, 23V
2

Questão 30. Cobre metálico exposto à atmosfera ambiente úmida sofre corrosão, com formação de cuprita
(Cu2O) sobre a sua superfície. Este fato é comprovado em laboratório com a aplicação de corrente elétrica,
proveniente de um gerador de corrente contínua, em um eletrodo de cobre (isento de óxido) imerso numa
solução aquosa neutra de cloreto de potássio (pH = 7) contendo oxigênio gasoso (O2) dissolvido. Considere que
esse procedimento é realizado nas seguintes condições:

I. Eletrodos metálicos empregados: catodo de platina e anodo de cobre.


2
II. Área imersa do anodo: 350,0 cm .
–2
III. Densidade de corrente aplicada: 10,0 μA cm .
IV. Tempo de eletrólise: 50 s.

Baseado no procedimento experimental acima descrito:

a) Escreva as equações químicas balanceadas que representam as reações envolvidas na formação da cuprita
sobre cobre metálico.
b) Calcule o valor numérico da massa de cuprita, expressa em g, formada sobre a superfície do anodo.
–3
c) Sabendo que a massa específica média da cuprita é igual a 6,0 g cm , calcule o valor numérico da
espessura média, expressa em μm, desse óxido formado durante a eletrólise.
CONSTANTES

23 −1
Constante de Avogadro = 6, 02 x 10 mol
−1 −1 −1 −1
= 9, 65 x 10 A s mol = 9, 65 x 10 J V mol
4 4 4
Constante de Faraday (F) = 9, 65 x 10 C mol
Volume molar de gás ideal = 22, 4 L (CNTP)
−19
Carga elementar = 1, 602 x 10 C
−2 −1 −1 −1 −1 −1 −1 −1 −1
Constante dos gases (R) = 8, 21 x 10 atm L K mol = 8, 31 J K mol = 1, 98 cal K mol = 62, 4 mm Hg L K mol
−2
Constante gravitacional (g) = 9,81 m s
DEFINIÇÕES

−2
Pressão de 1 atm = 760 mmHg = 101 325 N m = 760 Torr
−2
1 J = 1 N m = 1 kg m s
2

Condições normais de temperatura e pressão (CNTP): 0 º C e 760 mmHg


Condições ambientes: 25 º C e 1 atm
−1
Condições-padrão: 25 º C e 1 atm ; concentração das soluções = 1 mol L (rigorosamente: atividade unitária das espécies); sólido
com estrutura cristalina mais estável nas condições de pressão e temperatura em questão.
(s) = sólido . (l ) = líquido . (g ) = gás . (aq) = aquoso . (CM) = circuito metálico . (conc) = concentrado .
(ua) = unidades arbitrárias . [ A ] = concentração da espécie química A em mol L .
−1

MASSAS MOLARES

Massa Molar Massa Molar


Elemento Químico Número Atômico -1 Elemento Químico Número Atômico -1
(g mol ) (g mol )

H 1 1,01 Mn 25 54,94
Li 3 6,94 Fe 26 55,85
C 6 12,01 Co 27 58,93
N 7 14,01 Cu 29 63,55
O 8 16,00 Zn 30 65,39
F 9 19,00 As 33 74,92
Ne 10 20,18 Br 35 79,90
Na 11 22,99 Mo 42 95,94
Mg 12 24,30 Sb 51 121,76
Al 13 26,98 I 53 126,90
Si 14 28,08 Ba 56 137,33
S 16 32,07 Pt 78 195,08
Cl 17 35,45 Au 79 196,97
Ca 20 40,08 Hg 80 200,59

Questão 1. A solução aquosa 6% em massa de água oxigenada (H 2O 2 ) é geralmente empregada como agente
branqueador para tecidos e cabelos. Pode-se afirmar que a concentração aproximada dessa solução aquosa, expressa em
volumes, é

A ( ) 24. B ( ) 20. C ( ) 12. D ( ) 10. E ( ) 6.

Questão 2. Assinale a opção que apresenta o ácido mais forte, considerando que todos se encontram nas mesmas
condições de concentração, temperatura e pressão.

A( ) CH 3COOH B( ) CH 3CH 2 COOH C( ) ( CH3 )3 CCOOH


D( ) ClCH 2COOH E( ) Cl 3CCOOH
Questão 3. A 25 °C, três frascos (I, II e III) contêm, respectivamente, soluções aquosas 0,10 mol L−1 em acetato de
sódio, em cloreto de sódio e em nitrito de sódio.
Assinale a opção que apresenta a ordem crescente CORRETA de valores de pH x (x = I, II e III) dessas soluções,
sabendo que as constantes de dissociação (K) , a 25 °C, dos ácidos clorídrico (HCl) , nitroso (HNO 2 ) e acético
(CH 3COOH) , apresentam a seguinte relação:
K HCl > K HNO2 > K CH COOH
3

A( ) pH I < pH II < pH III B( ) pH I < pH III < pH II


C( ) pH II < pH I < pH III D( ) pH II < pH III < pH I
E( ) pH III < pH II < pH I

Questão 4. A 25 °C, as massas específicas do etanol e da água, ambos puros, são 0,8 g cm −3 e
1, 0 g cm −3 ,
respectivamente. Adicionando 72 g de água pura a 928 g de etanol puro, obteve-se uma solução com 1208 cm3 de
volume.
Assinale a opção que expressa a concentração desta solução em graus Gay-Lussac (°GL).

A( ) 98 B( ) 96 C( ) 94 D( ) 93 E( ) 72

Questão 5. Considere a energia liberada em

I. combustão completa (estequiométrica) do octano e em


II. célula de combustível de hidrogênio e oxigênio.

Assinale a opção que apresenta a razão CORRETA entre a quantidade de energia liberada por átomo de hidrogênio na
combustão do octano e na célula de combustível.
Dados: Energias de ligação, em kJ mol−1 :

C–C 347 H–H 436


C–H 413 H–O 464
C=O 803 O=O 498

A( ) 0,280 B( ) 1,18 C( ) 2,35 D( ) 10,5 E( ) 21,0

Questão 6. Em um experimento eletrolítico, uma corrente elétrica circula através de duas células durante 5 horas. Cada
célula contém condutores eletrônicos de platina. A primeira célula contém solução aquosa de íons Au 3+ enquanto que,
na segunda célula, está presente uma solução aquosa de íons Cu 2+ .
Sabendo que 9,85 g de ouro puro foram depositados na primeira célula, assinale a opção que corresponde à massa de
cobre, em gramas, depositada na segunda célula eletrolítica.

A ( ) 2,4 B ( ) 3,6 C ( ) 4,8 D ( ) 6,0 E ( ) 7,2

Questão 7. A combustão de um composto X na presença de ar atmosférico ocorre com a formação de fuligem.


Dos compostos abaixo, assinale a opção que contém o composto X que apresenta a maior tendência de combustão
fuliginosa.

A( ) C6 H 6 B( ) C2H5OH C( ) CH 4
D( ) CH3 (CH 2 )6 CH3 E( ) CH3OH
Questão 8. Nas condições ambientes, assinale a opção que contém apenas óxidos neutros.

A( ) NO 2 , CO e Al2O3 B( ) N 2O , NO e CO
C( ) N 2O , NO e NO 2 D( ) SiO 2 , CO 2 e Al2O3
E( ) SiO 2 , CO 2 e CO

Questão 9. Assinale a opção que apresenta a fórmula molecular do polímero que pode conduzir corrente elétrica.

A( ) CH2 CH2 B( ) CH CH C( ) CF2 CF2


n n n

D( ) CHCH3 CH2 E( ) CHOH CH2


n n

Questão 10. São descritos abaixo dois experimentos, I e II, nos quais há sublimação completa de uma mesma
quantidade de dióxido de carbono no estado sólido a 25 °C:
I - O processo é realizado em um recipiente hermeticamente fechado, de paredes rígidas e indeformáveis.
II - O processo é realizado em cilindro provido de um pistão, cuja massa é desprezível e se desloca sem atrito.
A respeito da variação da energia interna do sistema (ΔU) , calor (q) e trabalho (w), nos experimentos I e II, assinale
a opção que contém a afirmação ERRADA.

A( ) qI > 0 B( ) w II > w I C( ) ΔU I > ΔU II


D( ) w II ≠ 0 E( ) ΔU II = q II

Questão 11. Assinale a opção CORRETA que apresenta o potencial de equilíbrio do eletrodo Al3+ / Al , em volt, na
escala do eletrodo de referência de cobre-sulfato de cobre, à temperatura de 25 °C, calculado para uma concentração do
íon alumínio de 10−3 mol L−1 .
Dados: Potenciais de eletrodo padrão do cobre-sulfato de cobre (E o CuSO4 /Cu ) e do alumínio (E o Al3+ /Al ) , na escala do
eletrodo de hidrogênio, nas condições-padrão:

E o CuSO4 /Cu = 0,310 V


E o Al3+ /Al = − 1,67 V

A ( ) − 1, 23 B ( ) − 1,36 C ( ) − 1, 42 D ( ) − 1,98 E ( ) − 2, 04

Questão 12. Em um experimento de laboratório, cloreto de alumínio, cloreto de zinco e carbonato de sódio são
dissolvidos, individualmente, em três recipientes separados contendo água neutra aerada com pH = 7. Uma placa de
ferro metálico é imersa em cada um dos recipientes, que são mantidos à temperatura de 25 °C.
Admitindo-se as condições experimentais apresentadas acima, são feitas as seguintes afirmações em relação à
influência da hidrólise dos sais na velocidade de corrosão das placas metálicas:

I. O cátion alumínio hidratado forma soluções aquosas que aceleram a corrosão do ferro.
II. As soluções aquosas produzidas pela hidrólise do ânion carbonato inibem a corrosão do ferro.
III. A corrosão do ferro é inibida pela solução aquosa formada no processo de hidrólise do cátion zinco hidratado.

Das afirmações acima, está(ão) CORRETA(S) apenas

A ( ) I e II. B ( ) I e III. C ( ) II. D ( ) II e III. E ( ) III.


Questão 13. A reação catalisada do triacilglicerol com um álcool (metanol ou etanol) produz glicerol
(1, 2,3 − propanotriol) e uma mistura de ésteres alquílicos de ácidos graxos de cadeia longa, mais conhecido como
biodiesel. Essa reação de transesterificação envolve o equilíbrio representado pela seguinte equação química
balanceada:

H 2C − OCOR ' H 2C − OH R OCOR '


+
NaOH
HC − OCOR '' + 3R − OH HC − OH + R OCOR ''
+
H 2C − OCOR ''' H 2C − OH R OCOR '''
triacilglicerol álcool glicerol mistura de ésteres alquílicos
(biodiesel)

em que: R ', R ", R ''' = cadeias carbônicas dos ácidos graxos e R = grupo alquil do álcool reagente.

A respeito da produção do biodiesel pelo processo de transesterificação, são feitas as seguintes afirmações:

I. O hidróxido de sódio é dissolvido completamente e reage com o agente transesterificante para produzir água e o
íon alcóxido.
II. Na transesterificação catalisada por álcali, os reagentes empregados nesse processo devem ser substancialmente
anidros para prevenir a formação de sabões.
III. Na reação de produção do biodiesel pela rota etílica, com catalisador alcalino, o alcóxido formado inibe a reação
de saponificação.

Das afirmações acima, está(ão) CORRETA(S) apenas

A ( ) I e II. B ( ) I e III. C ( ) II. D ( ) II e III. E ( ) III.

Questão 14. Um sistema em equilíbrio é composto por n 0 mol de um gás ideal a pressão P0 , volume V0 , temperatura
T0 e energia interna U 0 . Partindo sempre deste sistema em equilíbrio, são realizados isoladamente os seguintes
processos:

I. Processo isobárico de T0 até T0 2 .


II. Processo isobárico de V0 até 2V0 .
III. Processo isocórico de P0 até P0 2 .
IV. Processo isocórico de T0 até 2T0 .
V. Processo isotérmico de P0 até P0 2 .
VI. Processo isotérmico de V0 até V0 2 .

Admitindo que uma nova condição de equilíbrio para esse sistema seja atingida em cada processo x
(x = I, II, III, IV, V e VI) , assinale a opção que contém a informação ERRADA.

A ( ) U V = U VI 2 B ( ) U VI = U 0 C ( ) PIV = PVI D ( ) TII = 4TIII E ( ) VI = VV 4

Questão 15. Quando aquecido ao ar, 1,65 g de um determinado elemento X forma 2,29 g de um óxido de fórmula
X 3O 4 . Das alternativas abaixo, assinale a opção que identifica o elemento X .

A( ) Antimônio B( ) Arsênio C( ) Ouro


D( ) Manganês E( ) Molibdênio
Questão 16. Assinale a opção que apresenta a ordem crescente ERRADA de solubilidade em água das substâncias
abaixo, nas condições ambientes.

A( ) C5 H12 < C5 H11Cl < C5 H11OH B( ) C5 H11OH < C4 H 9 OH < C3H 7 OH


C( ) CH 4 < C2H6 < C2H 4O D( ) CCl 2 F2 < CClF3 < CF4
E( ) N 2 < O 2 < NO

Questão 17. Considere as seguintes afirmações:

I. Um coloide é formado por uma fase dispersa e outra dispersante, ambas no estado gasoso.
II. As ligações químicas em cerâmicas podem ser do tipo covalente ou iônica.
III. Cristal líquido apresenta uma ou mais fases organizadas acima do ponto de fusão do sólido correspondente.

Então, das afirmações acima, está(ão) CORRETA(S)

A ( ) apenas I. B ( ) apenas I e II. C ( ) apenas II.


D ( ) apenas II e III. E ( ) apenas III.

Questão 18. Assinale a opção que apresenta a relação ERRADA a respeito do comprimento de ligação (R ) entre
pares de moléculas (neutras, cátions ou ânions), todas no estado gasoso.

A( ) R CO em CO < R CO em CO 2 B( ) R NO em NO + < R NO em NO −
C( ) R NO em NO 2 − < R NO em NO2 + D( ) R NN em N 2 F2 < R NN em N 2 F4
2−
E( ) R SO em SO3 < R SO em SO3

Questão 19. A figura mostra o perfil reacional da


decomposição de um composto X por dois caminhos
reacionais diferentes, I e II .
Baseado nas informações apresentadas nessa figura,
assinale a opção ERRADA.

A( ) O caminho reacional II envolve duas etapas.


B( ) A quantidade de energia liberada pelo caminho
reacional I é igual à do caminho reacional II .
C( ) O composto K é um intermediário no processo
reacional pelo caminho II .
D( ) O caminho reacional I mostra que a
decomposição de X é de primeira ordem.
E( ) O caminho reacional II refere-se à reação
catalisada.

Questão 20. Considere dois cilindros idênticos (C1 e C2) , de paredes rígidas e indeformáveis, inicialmente
evacuados. Os cilindros C1 e C2 são preenchidos, respectivamente, com O 2 (g) e Ne (g) até atingirem a pressão
de 0,5 atm e temperatura de 50 ºC. Supondo comportamento ideal dos gases, são feitas as seguintes afirmações:

I. O cilindro C1 contém maior quantidade de matéria que o cilindro C2 .


II. A velocidade média das moléculas no cilindro C1 é maior que no cilindro C2 .
III. A densidade do gás no cilindro C1 é maior que a densidade do gás no cilindro C2 .
IV. A distribuição de velocidades das moléculas contidas no cilindro C1 é maior que a das contidas no cilindro C2 .

Assinale a opção que apresenta a(s) afirmação(ões) CORRETA(S).

A( ) Apenas I e III. B( ) Apenas I e IV. C( ) Apenas II.


D( ) Apenas II e IV. E( ) Apenas III.
AS QUESTÕES DISSERTATIVAS, NUMERADAS DE 21 A 30, DEVEM SER RESPONDIDAS NO
CADERNO DE SOLUÇÕES.
AS QUESTÕES NUMÉRICAS DEVEM SER DESENVOLVIDAS ATÉ O FINAL, COM APRESENTAÇÃO
DO VALOR ABSOLUTO DO RESULTADO.
βC
Questão 21. A velocidade de uma reação química é dada pela seguinte equação: v = ; em que β e α
1+ αC
são constantes e C , a concentração do reagente.
Calcule o valor do produto α C quando a velocidade da reação atinge 90% do seu valor limite, o que ocorre quando
α C >> 1.

Questão 22. Determine a constante de equilíbrio, a 25 °C e 1,0 atm, da reação representada pela seguinte equação
química:
2 MnO −4 (aq) + 3Mn 2+ (aq) + 2 H 2O (l) 5MnO 2 (s) + 4H + (aq)

São dadas as semiequações químicas e seus respectivos potenciais elétricos na escala do eletrodo de hidrogênio, nas
condições-padrão:

2 MnO 4− (aq) + 8H + (aq) + 6e − 2 MnO 2 (s) + 4H 2O (l) ; E o MnO− MnO2


= 1,70 V
4

3MnO 2 (s) + 12 H + (aq) + 6e− 3Mn 2+ (aq) + 6 H 2O (l) ; E o MnO Mn 2 +


= 1, 23 V
2

Questão 23. Para cada conjunto de substâncias, escolha aquela que apresenta a propriedade indicada em cada caso.
Justifique sua resposta.

a) Entre acetona, ácido acético e ácido benzóico, qual deve apresentar a maior entalpia de vaporização?
b) Entre hidrogênio, metano e monóxido de carbono, qual deve apresentar o menor ponto de congelamento?
c) Entre flúor, cloro e bromo, qual deve apresentar maior ponto de ebulição?
d) Entre acetona, água e etanol, qual deve apresentar menor pressão de vapor nas condições ambientes?
e) Entre éter, etanol e etilenoglicol, qual deve apresentar maior viscosidade nas condições ambientes?

Questão 24. A reação química hipotética representada pela seguinte equação: 2 AB2C ⎯⎯
k
→ 2 AB2 + C 2
foi acompanhada experimentalmente, medindo-se as concentrações das espécies [ AB2C] , [ AB2 ] e [C2 ] em
função do tempo. A partir destas informações experimentais, foram determinadas a constante de velocidade (k) e a
lei de velocidade da reação.
Com base nessa lei de velocidade, o mecanismo abaixo foi proposto e aceito:

Mecanismo: AB2C ⎯⎯
k1
→ AB2 + C lenta
AB2C + C ⎯⎯→ AB2 + C2
k2
rápida

Explique como foi possível determinar a constante de velocidade (k) .

Questão 25. Em um frasco de vidro, uma certa quantidade de Ba(OH) 2 .8H 2O (s) é adicionada a uma quantidade,
em excesso, de NH 4 NO3 (s) , ambos pulverizados. Quando os dois reagentes são misturados, observa-se a ocorrência
de uma reação química. Imediatamente após a reação, o frasco é colocado sobre um bloco de madeira umedecido,
permanecendo aderido a ele por um certo período de tempo.
Escreva a equação química balanceada que representa a reação observada. Explique por que o frasco ficou aderido ao
bloco de madeira, sabendo que o processo de dissolução em água do NH 4 NO3 (s) é endotérmico.
Questão 26. Escreva as fórmulas estruturais das substâncias A, B, C, D, E e F apresentadas nas seguintes
equações químicas:
CH 3CH 2CH 2 Br + CN − → A + B
+
A + H 2O ⎯⎯→
H
C + D
LiAlH 4
A ⎯⎯⎯⎯ → E
A ⎯⎯⎯⎯
CH3MgBr
→ F

Questão 27. O dióxido de carbono representa, em média, 0,037% da composição volumétrica do ar seco atmosférico,
nas condições ambientes. Esse gás, dissolvido em água, sofre um processo de hidratação para formar um ácido
diprótico, que se ioniza parcialmente no líquido.
Admitindo-se que água pura seja exposta a CO 2 (g) atmosférico, nas condições ambientes, e sabendo que o equilíbrio
entre as fases gasosa e líquida desse gás é descrito pela lei de Henry, calcule:

a) a solubilidade do CO 2 (aq) , expressa em mg L−1 , nas condições especificadas acima, sabendo que a constante da
lei de Henry para CO 2 gasoso dissolvido em água a 25 ºC é 3, 4 x 10−2 mol L−1 atm −1 .
b) a concentração molar do ânion bicarbonato, expressa em mol L−1 , sabendo que a constante de dissociação ácida
para o primeiro equilíbrio de ionização do ácido diprótico a 25 ºC é 4, 4 x 10−7 .

Questão 28. Em um processo hidrometalúrgico, conduzido nas condições ambientes, o mineral calcopirita (CuFeS2 )
é lixiviado em solução aquosa de sulfato férrico. Durante o processo, o sulfato férrico é regenerado a partir da adição
de ácido sulfúrico e oxigênio gasoso a essa solução aquosa.
Sabendo que a calcopirita é um semicondutor que sofre corrosão eletroquímica em meios aquosos oxidantes e,
admitindo-se que esse mineral, empregado no processo de lixiviação, é quimicamente puro, escreva as equações
químicas balanceadas das reações que representam:

a) a etapa de lixiviação de CuFeS2 (s) com sulfato férrico aquoso.


b) a etapa de regeneração da quantidade exata de matéria total do sulfato férrico consumido no processo de lixiviação
da etapa “a”, com adição de solução aquosa diluída de ácido sulfúrico e injeção de gás oxigênio.
c) a reação global do processo de lixiviação da calcopirita, considerando-se as etapas “a” e “b” acima.

Questão 29. O produto de solubilidade em água, a 25 °C, do sal hipotético M(IO3 ) 2 é 7, 2 x 10−9 .
Calcule a solubilidade molar desse sal em uma solução aquosa 2,0 x 10−2 mol L−1 de M(NO3 ) 2 .

Questão 30. Estima-se que a exposição a 16 mg m −3 de vapor de mercúrio por um período de 10 min seja letal para
um ser humano. Um termômetro de mercúrio foi quebrado e todo o seu conteúdo foi espalhado em uma sala fechada de
10 m de largura, 10 m de profundidade e 3 m de altura, mantida a 25 °C.
Calcule a concentração de vapor de mercúrio na sala após o estabelecimento do equilíbrio Hg (l) Hg (g) ,
sabendo que a pressão de vapor do mercúrio a 25 °C é 3 x 10−6 atm , e verifique se a concentração de vapor do
mercúrio na sala será letal para um ser humano que permaneça em seu interior por 10 min.
CONSTANTES

23 −1
Constante de Avogadro = 6, 02 x 10 mol
−1 −1 −1 −1
Constante de Faraday (F) == =
4 4 4
9, 65 x 10 C mol 9, 65 x 10 A s mol 9, 65 x 10 J V mol
Volume molar de gás ideal = 22, 4 L (CNTP)
−19
Carga elementar = 1, 602 x 10 C
−2 −1 −1 −1 −1 −1 −1 −1 −1
Constante dos gases (R) = 8, 21 x 10 atm L K mol = 8, 31 J K mol = 1, 98 cal K mol = 62, 4 mm Hg L K mol
−2
Constante gravitacional (g) = 9,81 m s
DEFINIÇÕES

−2
=
Pressão de 1 atm =
760 mmHg 101=
325 N m 760 Torr
−2
= 1=
2
1J Nm 1 kg m s
Condições normais de temperatura e pressão (CNTP): 0 º C e 760 mmHg
Condições ambientes: 25 º C e 1 atm
−1
Condições-padrão: 25 º C e 1 atm ; concentração das soluções = 1 mol L (rigorosamente: atividade unitária das espécies); sólido
com estrutura cristalina mais estável nas condições de pressão e temperatura em questão.
(s) = sólido . ( ) = líquido . (g) = gás . (aq) = aquoso . (CM) = circuito metálico . (conc) = concentrado .
(ua) = unidades arbitrárias . [ A ] = concentração da espécie química A em mol L .
−1

MASSAS MOLARES

Elemento Número Massa Molar Elemento Número Massa Molar


Químico Atômico −1 Químico Atômico −1
(g.mol ) (g.mol )
H 1 1,01 S 16 32,07
Li 3 6,94 Cl 17 35,45
C 6 12,01 K 19 39,10
N 7 14,01 Ca 20 40,08
O 8 16,00 Mn 25 54,94
F 9 19,00 As 33 74,92
Na 11 22,99 Br 35 79,90
Mg 12 24,30 Ag 47 107,90
Al 13 26,98 I 53 126,90
Si 14 28,08 Pt 78 195,08
P 15 30,97 Hg 80 200,59

Questão 1. Uma amostra de 2×10−2 g de um determinado composto orgânico é dissolvida em 300 mL de água
a 25 °C, resultando numa solução de pressão osmótica 0,027 atm. Pode-se afirmar, então, que o composto
orgânico é o(a)

A ( ) ácido etanoico (ácido acético). B ( ) 1,2-etanodiol (etileno glicol).


C ( ) etanol (álcool etílico). D ( ) metanodiamida (ureia).
E ( ) tri-fluor-carbono.

Questão 2. Considere as seguintes afirmações:

I. Aldeídos podem ser oxidados a ácidos carboxílicos.


II. Alcanos reagem com haletos de hidrogênio.
III. Aminas formam sais quando reagem com ácidos.
IV. Alcenos reagem com álcoois para formar ésteres.

Das afirmações acima, está(ão) CORRETA(S) apenas

A ( ) I. B ( ) I e III. C ( ) II. D ( ) II e IV. E ( ) IV.


Questão 3. A reação de sulfonação do naftaleno ocorre por substituição eletrofílica nas posições α e β do
composto orgânico, de acordo com o diagrama de coordenada
de reação a 50 °C.

Com base neste diagrama, são feitas as seguintes afirmações:

I. A reação de sulfonação do naftaleno é endotérmica.


II. A posição α do naftaleno é mais reativa do que a de β.
III. O isômero β é mais estável que o isômero α.

Das afirmações acima, está(ão) CORRETA(S) apenas

A ( ) I. B ( ) I e II. C ( ) II. D ( ) II e III. E ( ) III.

Questão 4. Assinale a opção que corresponde, aproximadamente, ao produto de solubilidade do AgCl (c) em
água nas condições-padrão, sendo dados:
Ag + (aq ) + e −  Ag (c); E ο = 0,799 V e AgCl (c) + e −  Ag (c) + Cl − (aq ); E ο = 0,222 V ,
em que E ο é o potencial do eletrodo em relação ao eletrodo padrão de hidrogênio nas condições-padrão.

A ( ) 1×10−18 B ( ) 1×10−10 C ( ) 1×10−5 D ( ) 1×105 E ( ) 1×1010

Questão 5. Considere as seguintes misturas (soluto/solvente) na concentração de 10 % em mol de soluto:

I. acetona/clorofórmio II. água/etanol III. água/metanol


IV. benzeno/tolueno V. n-hexano/n-heptano

Assinale a opção que apresenta a(s) mistura(s) para a(s) qual(is) a pressão de vapor do solvente na mistura é
aproximadamente igual à sua pressão de vapor quando puro multiplicada pela sua respectiva fração molar.

A ( ) Apenas I B ( ) Apenas I, II e III C ( ) Apenas II e III


D ( ) Apenas IV e V E ( ) Apenas V

Questão 6. Considere que a reação hipotética representada pela equação química X + Y → Z ocorra em três
condições diferentes (a, b e c), na mesma temperatura, pressão e composição total (número de moléculas de
X+Y), a saber:

a- O número de moléculas de X é igual ao número de moléculas de Y.


b- O número de moléculas de X é 1/3 do número de moléculas de Y.
c- O número de moléculas de Y é 1/3 do número de moléculas de X.

Baseando nestas informações, considere que sejam feitas as seguintes afirmações:

I. Se a lei de velocidade da reação for v = k [ X ].[Y ]2 , então v c < v a < vb .


II. Se a lei de velocidade da reação for v = k [ X ].[Y ] , então v=
b vc < va .
III. Se a lei de velocidade da reação for v = k [ X ] , então t1 2( c ) > t1 2(b ) > t1 2( a ) , em que t1 2 = tempo de meia-vida.

Das afirmações acima, está(ão) CORRETA(S) apenas

A ( ) I. B ( ) I e II. C ( ) II. D ( ) II e III. E ( ) III.


Questão 7. Considere os seguintes potenciais de eletrodo em relação ao eletrodo padrão de hidrogênio nas
condições-padrão ( E ο ) : E οM3+ M 2+ = 0,80 V e E οM 2+ Mο = − 0, 20 V. Assinale a opção que apresenta o valor, em V,
de E οM3+ Mο
.

A ( ) – 0,33 B ( ) – 0,13 C ( ) + 0,13 D ( ) + 0,33 E ( ) + 1,00

Questão 8. Considere as seguintes afirmações a respeito dos haletos de hidrogênio HF , HCl , HBr e HI :

I. A temperatura de ebulição do HI é maior do que a dos demais.


II. À exceção do HF , os haletos de hidrogênio dissociam-se completamente em água.
III. Quando dissolvidos em ácido acético glacial puro, todos se comportam como ácidos, conforme a seguinte
ordem de força ácida: HI > HBr > HCl >> HF .

Das afirmações acima, está(ão) CORRETA(S) apenas

A ( ) I. B ( ) I e II. C ( ) II. D ( ) II e III. E ( ) III.

Questão 9. Considere volumes iguais dos gases NH 3 , CH 4 e O2 nas CNTP. Assinale a opção que apresenta o(s)
gás(es) que se comporta(m) idealmente.

A ( ) Apenas NH 3 B ( ) Apenas CH 4 C ( ) Apenas O2


D ( ) Apenas NH 3 e CH 4 E ( ) Apenas CH 4 e O2

Questão 10. A 25 °C, a força eletromotriz da seguinte célula eletroquímica é de 0,45 V:


Pt ( s ) | H 2 (g, 1 atm) | H + ( x mol.L−1 ) || KCl (0,1 mol.L−1 ) | Hg 2Cl2 ( s ) | Hg ( l ) Pt ( s ) .
Sendo o potencial do eletrodo de calomelano – KCl (0,1 mol.L−1 ) | Hg 2Cl2 ( s ) | Hg ( l ) – nas condições-padrão
igual a 0,28 V e x o valor numérico da concentração dos íons H+, assinale a opção com o valor aproximado do
pH da solução.

A ( ) 1,0 B ( ) 1,4 C ( ) 2,9 D ( ) 5,1 E ( ) 7,5

Questão 11. São feitas as seguintes afirmações a respeito dos produtos formados preferencialmente em
eletrodos eletroquimicamente inertes durante a eletrólise de sais inorgânicos fundidos ou de soluções aquosas
de sais inorgânicos:

I. Em CaCl2 (l ) há formação de Ca ( s ) no catodo.


II. Na solução aquosa 1×10−3 mol.L−1 em Na2 SO4 há aumento do pH ao redor do anodo.
III. Na solução aquosa 1mol.L−1 em AgNO3 há formação de O2 ( g ) no anodo.
IV. Em NaBr (l ) há formação de Br2 (l ) no anodo.

Das afirmações acima, está(ão) ERRADA(S) apenas

A ( ) I e II. B ( ) I e III. C ( ) II. D ( ) III. E ( ) IV.


Questão 12. São feitas as seguintes afirmações em relação à isomeria de compostos orgânicos:

I. O 2-cloro-butano apresenta dois isômeros óticos.


II. O n-butano apresenta isômeros conformacionais.
III. O metil-ciclo-propano e o ciclo-butano são isômeros estruturais.
IV. O alceno de fórmula molecular C4 H 8 apresenta um total de três isômeros.
V. O alcano de fórmula molecular C5 H12 apresenta um total de dois isômeros.

Das afirmações acima, está(ão) CORRETA(S) apenas

A ( ) I, II e III. B ( ) I e IV. C ( ) II e III.


D ( ) III, IV e V. E ( ) IV e V.

Questão 13. Considere as reações representadas pelas seguintes equações químicas:

I. C ( s ) + 2 H 2 ( g ) → CH 4 ( g ) II. N 2O( g ) → N 2 ( g ) + 1 2 O2 ( g )
III. 2 NI 3 ( s ) → N 2 ( g ) + 3I 2 ( g ) IV. 2O3 ( g ) → 3O2 ( g )

Assinale a opção que apresenta a(s) reação(ões) química(s) na(s) qual(is) há uma variação negativa de entropia.

A ( ) Apenas I B ( ) Apenas II e IV C ( ) Apenas II e III e IV


D ( ) Apenas III E ( ) Apenas IV

Questão 14. Assinale a opção que indica o polímero da borracha natural.

A ( ) Poliestireno B ( ) Poliisopreno C ( ) Poli (metacrilato de metila)


D ( ) Polipropileno E ( ) Poliuretano

Questão 15. Assinale a opção que apresenta os compostos nitrogenados em ordem crescente de número de
oxidação do átomo de nitrogênio.

A ( ) N 2 H 4 < K 2 N 2O2 < NaNH 2 < NI 3 < Na2 NO2 B ( ) K 2 N 2O2 < Na2 NO2 < NI 3 < NaNH 2 < N 2 H 4
C ( ) NaNH 2 < N 2 H 4 < K 2 N 2O2 < Na2 NO2 < NI 3 D ( ) NI 3 < NaNH 2 < Na2 NO2 < N 2 H 4 < K 2 N 2O2
E ( ) Na2 NO2 < NI 3 < N 2 H 4 < K 2 N 2O2 < NaNH 2

Questão 16. A figura representa a curva de aquecimento de uma amostra, em que S, L e G significam,
respectivamente, sólido, líquido e gasoso. Com base nas informações da figura é CORRETO afirmar que a
amostra consiste em uma

A( ) substância pura.
B( ) mistura coloidal.
C( ) mistura heterogênea.
D( ) mistura homogênea azeotrópica.
E( ) mistura homogênea eutética.
Questão 17. Considere os seguintes pares de moléculas:

I. LiCl e KCl II. AlCl3 e PCl3 III. NCl3 e AsCl3

Assinale a opção com as três moléculas que, cada uma no seu respectivo par, apresentam ligações com o maior
caráter covalente.

A ( ) LiCl , AlCl3 e NCl3 B ( ) LiCl , PCl3 e NCl3 C ( ) KCl , AlCl3 e AsCl3


D ( ) KCl , PCl3 e NCl3 E ( ) KCl , AlCl3 e NCl3

Questão 18. São descritos três experimentos (I, II e III) utilizando-se em cada um 30 mL de uma solução
aquosa saturada, com corpo de fundo de cloreto de prata, em um béquer de 50 mL a 25 °C e 1 atm:

I. Adiciona-se certa quantidade de uma solução aquosa 1 mol.L−1 em cloreto de sódio.


II. Borbulha-se sulfeto de hidrogênio gasoso na solução por certo período de tempo.
III. Adiciona-se certa quantidade de uma solução aquosa 1 mol.L−1 em nitrato de prata.

Em relação aos resultados observados após atingir o equilíbrio, assinale a opção que apresenta o(s)
experimento(s) no(s) qual(is) houve aumento da quantidade de sólido.

A ( ) Apenas I B ( ) Apenas I e II C ( ) Apenas I e III


D ( ) Apenas II e III E ( ) Apenas I, II e III

Questão 19. Assinale a opção com a resina polimérica que mais reduz o coeficiente de atrito entre duas
superfícies sólidas.

A ( ) Acrílica B ( ) Epoxídica C ( ) Estirênica


D ( ) Poliuretânica E ( ) Poli (dimetil siloxano)

Questão 20. Considere uma amostra aquosa em equilíbrio a 60 °C, com pH de 6,5, a respeito da qual são feitas
as seguintes afirmações:

I. A amostra pode ser composta de água pura.


II. A concentração molar de H 3O + é igual à concentração de OH − .
III. O pH da amostra não varia com a temperatura.
IV. A constante de ionização da amostra depende da temperatura.
V. A amostra pode ser uma solução aquosa 0,1mol.L−1 em H 2CO3 , considerando que a constante de
dissociação do H 2CO3 é da ordem de 1×10−7 .

Das afirmações acima está(ão) CORRETA(S) apenas

A ( ) I, II e IV. B ( ) I e III. C ( ) II e IV.


D ( ) III e V. E ( ) V.
AS QUESTÕES DISSERTATIVAS, NUMERADAS DE 21 A 30, DEVEM SER RESPONDIDAS NO CADERNO
DE SOLUÇÕES.
AS QUESTÕES NUMÉRICAS DEVEM SER DESENVOLVIDAS SEQUENCIALMENTE ATÉ O FINAL.

Questão 21. A tabela mostra a variação de entalpia de formação nas Substância ∆H 0f (kJ.mol−1)
condições-padrão a 25 °C de algumas substâncias. AgCl ( s ) −127
Calcule a variação da energia interna de formação, em kJ.mol−1, nas −1207
CaCO3 ( s )
condições-padrão dos compostos tabelados. Mostre os cálculos
realizados. H 2O (l ) −286
H2S (g) −20
NO2 ( g ) +34

Questão 22. Apresente os respectivos produtos (A, B, C, D e E) das reações químicas representadas pelas
seguintes equações:

CH2CH3
Cl2
A + B
Calor
OH
HNO3
C
H2SO4

CH3

KMnO4 HNO3
D E
H2SO4

Questão 23. Uma mistura gasosa é constituída de C3 H 8 , CO e CH 4 . A combustão de 100 L desta mistura em
excesso de oxigênio produz 190 L de CO2 .
Determine o valor numérico do volume, em L, de propano na mistura gasosa original.

Questão 24. Descreva por meio de equações as reações químicas envolvidas no processo de obtenção de
magnésio metálico a partir de carbonato de cálcio e água do mar.

Questão 25. A figura apresenta a variação de velocidade em função do tempo para a reação química hipotética
não catalisada representada pela equação A2 + B2  2 AB .
Reproduza esta figura no caderno de soluções, incluindo no mesmo gráfico, além das curvas da reação
catalisada, as da reação não catalisada, explicitando ambas as condições.
Questão 26. Considere a reação de combustão do composto X, de massa molar igual a 27,7 g.mol−1,
representada pela seguinte equação química balanceada:

−2035 kJ.mol−1
X ( g ) + 3O2 ( g ) → Y ( s ) + 3H 2O( g ); ∆H c0 =

Calcule o valor numérico, em kJ, da quantidade de calor liberado na combustão de:

a) 1, 0 ×103 g de X
b) 1, 0 ×102 mol de X
c) 2, 6 ×1022 moléculas de X
d) uma mistura de 10, 0 g de X e 10, 0 g de O2

Questão 27. Considere dois lagos naturais, um dos quais contendo rocha calcárea ( CaCO3 e MgCO3 ) em
contato com a água.
Discuta o que acontecerá quando houver precipitação de grande quantidade de chuva ácida (pH< 5, 6 ) em
ambos os lagos. Devem constar de sua resposta os equilíbrios químicos envolvidos.

Questão 28. A figura apresenta o diagrama de distribuição de espécies para o ácido fosfórico em função do pH.

Com base nesta figura, pedem-se:


a) Os valores de pK a1 , pK a2 e pK a3 , sendo K a1 , K a2 e K a2 , respectivamente, a primeira, segunda e terceira
constantes de dissociação do ácido fosfórico.
b) As substâncias necessárias para preparar uma solução tampão de pH 7,4, dispondo-se do ácido fosfórico e
respectivos sais de sódio. Justifique.
c) A razão molar das substâncias escolhidas no item b).
d) O procedimento experimental para preparar a solução tampão do item b).

Questão 29. A nitrocelulose é considerada uma substância química explosiva, sendo obtida a partir da nitração
da celulose. Cite outras cinco substâncias explosivas sintetizadas por processos de nitração.

Questão 30. Explique como diferenciar experimentalmente uma amina primária de uma secundária por meio da
reação com o ácido nitroso. Justifique a sua resposta utilizando equações químicas para representar as reações
envolvidas.
CONSTANTES

Constante de Avogadro = 6, 02 × 10 23 mol − 1


Constante de Faraday ( F ) = 9, 65 × 10 4 C ⋅ mol − 1 = 9, 65 × 10 4 A ⋅ s ⋅ mol − 1 = 9, 65 × 10 4 J ⋅ V −1
⋅ mol − 1
Volume molar de gás ideal = 22, 4 L ( CNTP )
Carga elementar = 1, 602 × 10 − 19 C
Constante dos gases ( R ) = 8, 21 × 10 − 2 atm ⋅ L ⋅ K − 1 ⋅ mol − 1 = 8,31 J ⋅ K − 1 ⋅ mol − 1 = 1,98 cal ⋅ K − 1 ⋅ mol − 1 =
62, 4 mmHg ⋅ L ⋅ K − 1 ⋅ mol − 1
Constante gravitacional ( g ) = 9,81 m ⋅ s − 2
(
Constante de Rydberg R ∞ hc ) = 2,18 × 10 − 18 J = 13, 6 eV

DEFINIÇÕES

Pressão de 1 atm = 760 mmHg = 101 325 N ⋅ m − 2 = 760 Torr


1 J = 1 N ⋅ m = 1 kg ⋅ m 2 ⋅ s − 2 ; 1 pm = 1 × 10 − 12 m ; 1 eV = 1,602 × 10 − 19 J
Condições normais de temperatura e pressão ( CNTP ) : 0 0 C e 760 mmHg
Condições ambientes: 25 0 C e 1 atm
Condições-padrão: 25 0 C e 1 atm ; concentração das soluções = 1 mol ⋅ L − 1 (rigorosamente: atividade unitária das espécies);
sólido com estrutura cristalina mais estável nas condições de pressão e temperatura em questão.
( s ) = sólido. ( A ) = líquido. ( g ) = gás. ( aq ) = aquoso. ( CM ) = circuito metálico. ( conc ) = concentrado.
( ua ) = unidades arbitrárias. [ A] = concentração da espécie química A em mol ⋅ L − 1 .

MASSAS MOLARES
Massa Molar Massa Molar
Elemento Químico Número Atômico
(
g ⋅ mol − 1 )
Elemento Químico Número Atômico
( g ⋅ mol )−1

H 1 1,01 Ca 20 40,08
Li 3 6,94 Cr 24 52,00
B 5 10,81 Fe 26 55,85
C 6 12,01 Cu 29 63,55
N 7 14,01 Zn 30 65,38
O 8 16,00 Ge 32 72,63
F 9 19,00 Br 35 79,90
Na 11 22,99 Ag 47 107,90
Mg 12 24,31 I 53 126,90
Al 13 26,98 Xe 54 131,30
P 15 30,97 Ba 56 137,30
S 16 32,07 Pt 78 195,10
Cl 17 35,45 Hg 80 200,60
K 19 39,10 Pb 82 207,20

Questão 1. Uma alíquota de uma solução aquosa constituída de haletos de sódio foi adicionada a uma solução aquosa de
nitrato de prata, com formação de um precipitado. À mistura contendo o precipitado, foi adicionada uma alíquota de
solução aquosa diluída de hidróxido de amônio, com dissolução parcial do precipitado. Ao precipitado remanescente, foi
adicionada uma alíquota de solução aquosa concentrada de hidróxido de amônio, verificando-se uma nova dissolução
parcial do precipitado.
Sabendo que a mistura de haletos é constituída pelo fluoreto, brometo, cloreto e iodeto de sódio, assinale a alternativa
CORRETA para o(s) haleto(s) de prata presente(s) no precipitado não dissolvido.

A ( ) AgBr B ( ) AgCA C ( ) AgF D ( ) AgI E ( ) AgBr e AgCA


Questão 2. Assinale a alternativa CORRETA para a substância química que dissolvida em água pura produz uma solução
colorida.

A ( ) CaCA 2 B ( ) CrC A 3 C ( ) NaOH D ( ) KBr E ( ) Pb ( NO3 )2

Questão 3. Assinale a alternativa CORRETA para o líquido puro com a maior pressão de vapor a 25 0C .

A ( ) n-Butano, C4 H10 B ( ) n-Octano, C8 H18 C ( ) Propanol, C3 H 7 OH


D ( ) Glicerol, C3 H 5 ( OH )3 E ( ) Água, H 2 O

Questão 4. Na temperatura ambiente, hidróxido de potássio sólido reage com o cloreto de amônio sólido, com a liberação
de um gás. Assinale a alternativa CORRETA para o gás liberado nesta reação.

A ( ) CA 2 B ( ) H2 C ( ) HCA D ( ) NH 3 E ( ) O2

Questão 5. Assinale a alternativa CORRETA para o par de substâncias cujas soluções aquosas, ao serem misturadas,
produz um precipitado amarelo.

A ( ) AAC A 3 e KOH B ( ) Ba ( NO3 )2 e Na2 SO4 C ( ) Cu ( NO3 )2 e NaC AO4


D ( ) Pb ( C2 H 3O 2 ) 2 e KI E ( ) AgNO3 e NH 4 OH

Questão 6. Um álcool primário, como o etanol, pode ser obtido pela redução de um ácido carboxílico. Assinale a
alternativa CORRETA para o agente redutor que pode ser utilizado nesta reação.

A ( ) K 2 Cr2 O7 B ( ) K 2 CrO4 C ( ) LiAAH 4 D ( ) H 2 SO4 concentrado E ( ) HNO3 concentrado

Questão 7. Na figura abaixo é apresentada uma disposição bidimensional de bolinhas brancas e cinzas formando um
“cristal”. Assinale a opção que apresenta a reprodução CORRETA para a célula unitária (caixa em destaque) do “cristal”
em questão.

 
 
 
A() B() C()

D() E()
Questão 8. A reação entre os íons brometo e bromato, em meio aquoso e ácido, pode ser representada pela seguinte
equação química balanceada:

5 Br −( aq ) + BrO3−( aq ) + 6 H + ( aq ) → 3 Br2 ( aq ) + 3 H 2 O ( A )

Sabendo que a velocidade de desaparecimento do íon bromato é igual a 5, 63 × 10 − 6 mol ⋅ L −1


⋅s −1
, assinale a alternativa
−1 −1
que apresenta o valor CORRETO para a velocidade de aparecimento do bromo, Br2 , expressa em mol ⋅ L ⋅s .

A ( ) 1, 69 × 10 − 5 B ( ) 5, 63 × 10 − 6 C ( ) 1,90 × 10 − 6 D ( ) 1,13 × 10 − 6 E ( ) 1,80 × 10 − 16

Questão 9. 100 gramas de água líquida foram aquecidos utilizando o calor liberado na combustão completa de 0, 25
gramas de etanol. Sabendo que a variação da temperatura da água foi de 12,5 0 C , assinale a alternativa que apresenta o
valor CORRETO para a entalpia molar de combustão do etanol. Considere que a capacidade calorífica da água é igual a
4,18 kJ ⋅ kg − 1 ⋅ 0 C − 1 e que a energia liberada na combustão do etanol foi utilizada exclusivamente no aquecimento da
água.

A ( ) − 961 kJ B ( ) − 5, 2 kJ C ( ) + 4, 2 kJ D ( ) + 5, 2 kJ E ( ) + 961 kJ

Questão 10. Considere Y a quantidade (em mol) de iodo dissolvido em 100 mL de água, X um solvente praticamente
imiscível em água e K ( = 120 ) a constante de partição do iodo entre o solvente X e a água a 25 0C . Assinale a
alternativa CORRETA para o volume do solvente X necessário para extrair 90% do iodo contido inicialmente em
100 mL de água.

A ( ) 7,5 mL B ( ) 9, 0 mL C ( ) 12 mL D ( ) 100 mL E ( ) 120 mL

Questão 11. Considere as substâncias I , II e III representadas pelas seguintes fórmulas estruturais:

O O O
H
HO C CH2 CH C NH CH C
N OCH3
CH2 CH NH C CH2 CH2 NH2 NH2 CH2
COOH O
N

I . β-alanil L-histidina II . L-alfa-aspartil-L-fenilalanil metil-éster


O O
S
N O Na
H
III . ciclohexilsulfamato de sódio

Sob certas condições de umidade, temperatura, pH e/ou presença de determinadas enzimas, estas substâncias são
hidrolisadas. Assinale a opção CORRETA para o(s) produto(s) formado(s) na reação de hidrólise das respectivas
substâncias.

A ( ) Somente aminoácido é formado em I . B ( ) Somente aminoácido é formado em II .


C ( ) Amina aromática é formada em I e II . D ( ) Amina é formada em I e III .
E ( ) Aminoácido é formado em II e III .
Questão 12. A tabela ao lado apresenta os números de
cargas elétricas ( Z ) e o raio iônico ( r ) apresentados por Cátion metálico Z r ( pm )
alguns cátions metálicos.
Para as mesmas condições de temperatura e pressão é Na + +1 95
CORRETO afirmar que o pH de soluções aquosas, com Fe 2 + +2 76
concentração 1 mol ⋅ L − 1 dos nitratos de cada um dos Mg 2+ +2 65
cátions apresentados na tabela, aumenta na sequência: +3 64
Fe 3 +
AA 3 + +3 50

A ( ) Na + < Fe 2+
< Mg 2+
≅ Fe 3 + < AA 3 + B ( ) Na + < Fe 2+
< Mg 2+
< Fe 3 + < AA 3 +
C ( ) AA 3 + ≅ Fe 3 + < Mg 2+
≅ Fe 2 + < Na + D ( ) AA 3 + < Fe 3 + ≅ Mg 2+
< Fe 2+
< Na +

3+ 3+ 2+ 2+ +
E ( ) AA < Fe < Mg < Fe < Na

Questão 13. Assinale a opção que apresenta a afirmação CORRETA.

A ( ) Um paciente com calor de 42 0C apresenta-se febril.


B ( ) A adição de energia térmica à água líquida em ebulição sob pressão ambiente causa um aumento na sua capacidade
calorífica.
C ( ) Na temperatura de − 4 0C e pressão ambiente, 5 g de água no estado líquido contêm uma quantidade de energia
maior do que a de 5 g de água no estado sólido.
D ( ) A quantidade de energia necessária para aquecer 5 g de água de 20 0 C até 25 0 C é igual àquela necessária para
aquecer 25 g de água no mesmo intervalo de temperatura e pressão ambiente.
E ( ) Sob pressão ambiente, a quantidade de energia necessária para aquecer massas iguais de alumínio (calor específico
0,89 J ⋅ g − 1 ⋅ K − 1 ) e de ferro (calor específico 0, 45 J ⋅ g − 1 ⋅ K − 1 ), respectivamente, de um mesmo incremento de
temperatura, ΔT , é aproximadamente igual.

Questão 14. Considere o produto de solubilidade ( K ps ) , a 25 0 C , das substâncias I , II e III :

I. Ca ( OH )2 ; K ps = 5, 0 × 10 −6
II . Mg ( OH )2 ; K ps = 5, 6 × 10 − 12

III . Zn ( OH ) 2 ; K ps = 3, 0 × 10 − 17

Assinale a opção que contém a ordem CORRETA da condutividade elétrica, à temperatura de 25 0 C , de soluções aquosas
não saturadas, de mesma concentração, dessas substâncias.

A ( ) I < II < III B ( ) I = II = III C ( ) II < I < III D ( ) III < I < II E ( ) III < II < I

Questão 15. É ERRADO afirmar que, à temperatura de 25 0 C , o potencial de um eletrodo de cobre construído pela
imersão de uma placa de cobre em solução aquosa 1 mol ⋅ L − 1 de cloreto de cobre

A() diminui se amônia é acrescentada à solução eletrolítica.


B() diminui se a concentração do cloreto de cobre na solução eletrolítica for diminuída.
C() duplica se a área da placa de cobre imersa na solução eletrolítica for duplicada.
D() permanece inalterado se nitrato de potássio for adicionado à solução eletrolítica tal que sua concentração nesta
solução seja 1 mmol ⋅ L − 1 .
E ( ) aumenta se a concentração de íons de cobre for aumentada na solução eletrolítica.
Questão 16. Uma solução líquida constituída por dois componentes A e B e apresentando comportamento ideal,
conforme Lei de Rauolt, está em equilíbrio com seu vapor. Utilizando a notação:

x A e xB para as respectivas frações em mol das substâncias A e B na solução líquida,


p A e pB para as respectivas pressões de vapor de A e B no vapor em equilíbrio com a solução líquida, e
p 0A e pB0 para as respectivas pressões de vapor de A puro e B puro numa mesma temperatura,

assinale a opção que apresenta a relação CORRETA para a pressão de vapor de A ( pA ) em equilíbrio com a solução
líquida.

A ( ) p A = p 0A ⋅ (1 − x A ) B ( ) p A = pB0 ⋅ (1 − xB ) C ( ) p A = pB0 ⋅ x A D ( ) p A = p 0A ⋅ x A E ( ) p A = pB0 ⋅ xB

Questão 17. Assinale a opção CORRETA para a propriedade físico-química cujo valor diminui com o aumento de forças
intermoleculares.

A ( ) Tensão superficial B ( ) Viscosidade C ( ) Temperatura de ebulição


D ( ) Temperatura de solidificação E ( ) Pressão de vapor

Questão 18. Um átomo A com n elétrons, após ( n − 1) sucessivas ionizações, foi novamente ionizado de acordo com a
equação A ( n − 1) + → A n + + 1 e − . Sabendo o valor experimental da energia de ionização deste processo, pode-se
conhecer o átomo A utilizando o modelo proposto por

A ( ) E. Rutherford. B ( ) J. Dalton. C ( ) J. Thomson. D ( ) N. Bohr. E ( ) R. Mulliken.

Questão 19. Os átomos A e B do segundo período da tabela periódica têm configurações eletrônicas da camada de
valência representadas por ns 2 np 3 e ns 2 np 5 , respectivamente. Com base nessas informações, são feitas as seguintes
afirmações para as espécies gasosas no estado fundamental:

I. O átomo A deve ter maior energia de ionização que o átomo B .


II. A distância da ligação entre os átomos na molécula A2 deve ser menor do que aquela na molécula B2 .
III. A energia de ionização do elétron no orbital 1 s do átomo A deve ser maior do que aquela do elétron no orbital 1 s do
átomo de hidrogênio.
IV. A energia de ligação dos átomos na molécula B2 deve ser menor do que aquela dos átomos na molécula de
hidrogênio ( H 2 ) .

Das afirmações acima está(ão) CORRETA(S) apenas

A ( ) I , II e IV . B ( ) I e III . C ( ) II e III . D ( ) III e IV . E ( ) IV .

Questão 20. Considere as seguintes substâncias:

I. II . III . IV .

O
O O O
H3C C CH2 CH CH3 CH C OH
CH3
O

Dessas substâncias, é (são) classificada(s) como cetona(s) apenas

A ( ) I e II . B ( ) II . C ( ) II e III . D ( ) II , III e IV . E ( ) III .


AS QUESTÕES DISSERTATIVAS, NUMERADAS DE 21 A 30, DEVEM SER RESPONDIDAS NO CADERNO
DE SOLUÇÕES.
AS QUESTÕES NUMÉRICAS DEVEM SER DESENVOLVIDAS ATÉ O FINAL, COM APRESENTAÇÃO DO
VALOR ABSOLUTO DO RESULTADO.

Questão 21. A reação química de um ácido fraco (com um hidrogênio dissociável) com uma base forte produziu um sal.
Uma solução aquosa 0, 050 mol ⋅ L − 1 desse sal puro é mantida à temperatura constante de 25 0 C . Admitindo-se que a
constante de hidrólise do sal é K h, 25 oC = 5, 0 × 10 − 10
, determine o valor numérico da concentração, em mol ⋅ L − 1
, do íon
hidróxido nessa solução aquosa.

Questão 22. Nas condições ambientes, uma placa de ferro metálico puro é mergulhada numa solução aquosa, com pH 9
e isenta de oxigênio, preparada pelo borbulhamento de sulfeto de hidrogênio gasoso em solução alcalina. Nesta solução, o
ferro é oxidado (corroído) pelo íon hidrogenossulfeto com formação de uma camada sólida aderente e protetora sobre a
superfície desse material metálico. A adição de cianeto de potássio à solução aquosa em contato com o substrato metálico
protegido desestabiliza sua proteção promovendo a dissolução da camada protetora formada.
Com base nessas informações, escreva as equações químicas balanceadas das reações que representam:

a) a corrosão eletroquímica do ferro pelo íon hidrogenossulfeto, produzindo hidrogênio atômico.


b) a dissolução da camada passiva sobre o ferro pelo íon cianeto.

Questão 23. Em um gráfico de pressão versus volume, represente o diagrama do ciclo idealizado por Carnot (máquina
térmica) para uma transformação cíclica, ininterrupta, e sem perdas de calor e de trabalho, e vice-versa. Identifique e
denomine as quatro etapas dessa transformação cíclica.

Questão 24. Por exposição à atmosfera ambiente, o hidróxido de cálcio hidratado (cal hidratada) produz um filme que é
utilizado na proteção de superfícies de alvenaria em um processo denominado “caiação”. Escreva a(s) equação(ões)
química(s) balanceada(s) da(s) reação(ões) que representa(m), respectivamente,:

a) a formação do filme acima citado, e


b) o processo de produção industrial da cal hidratada.

Questão 25. A hidrazina ( N 2 H 4 ) e o tetróxido de dinitrogênio ( N 2O4 ) são utilizados na propulsão líquida de foguete. A
equação química não-balanceada que representa a reação global entre esses dois reagentes químicos é

N 2 H 4 ( A ) + N 2 O4 ( A ) → N 2 ( g ) + H 2O ( g )

Analisando esta reação do ponto de vista eletroquímico:

a) esquematize um dispositivo eletroquímico (célula de combustível) no qual é possível realizar a reação química
representada pela equação do enunciado.
b) escreva as equações químicas balanceadas das semirreações anódica e catódica que ocorrem no dispositivo
eletroquímico.

Questão 26. Nas condições ambientes, qual dos cloretos é mais solúvel em etanol puro: cloreto de sódio ou cloreto de
lítio? Justifique.

Questão 27. Nas condições ambientes, 0,500 g de um resíduo sólido foi dissolvido completamente em aproximadamente
13 mL de uma mistura dos ácidos nítrico e fluorídrico ( HNO3 : HF = 10 : 3) . A solução aquosa ácida obtida foi
quantitativamente transferida para um balão volumétrico com capacidade de 250 mL e o volume do balão completado com
água desmineralizada. A análise quantitativa dos íons de ferro na solução do balão revelou que a quantidade de ferro nesta
solução era igual a 40, 0 mg ⋅ L − 1 . Respeitando o número de algarismos significativos, determine a quantidade de ferro (em
% em massa) presente no resíduo sólido. Mostre o raciocínio e os cálculos realizados para chegar à sua resposta.
Questão 28. Os diagramas seguintes, traçados numa mesma escala, referem-se, respectivamente, aos equilíbrios, em fase
gasosa e numa mesma temperatura, representados pelas seguintes equações químicas:

I. AB + CD R AD + CB; K1
II . AX + CY R AY + CX ; K2

ENERGIA ENERGIA

E3

E2 AB + CD E´3
E´2 AX + CY

E1 AC + BD AY+ CX
E´1

COORDENADA DA REAÇÃO COORDENADA DA REAÇÃO

Comparando as informações apresentadas nos dois diagramas, pedem-se:

a) Qual das constantes de equilíbrio, K1 ou K 2 terá valor maior? Justifique sua resposta.
Dado eventualmente necessário: A relação entre a variação da Energia Livre de Gibbs padrão ΔG 0 e a constante de ( )
equilíbrio ( K ) de uma reação é dada por ΔG 0 = − RT ⋅ An K .
b) Para as seguintes misturas numa mesma temperatura:

Mistura 1 Mistura 2

[ AB] inicial = 0,10 mol ⋅ L − 1 [ AD] inicial = ZERO [ AX ] inicial = 0,10 mol ⋅ L − 1 [CY ] inicial = 0, 20 mol ⋅ L − 1
[CD] inicial = 0, 20 mol ⋅ L − 1 [CB] inicial = ZERO [ AY ] inicial = ZERO [CX ] inicial = ZERO

Qual das reações químicas, expressa pela equação I ou II , atinge o equilíbrio mais rapidamente? Justifique sua resposta.

Questão 29. Sabendo que a energia de ionização do processo descrito na Questão 18 é igual a 122, 4 eV , determine qual é
o átomo A utilizando equações e cálculos pertinentes.

Questão 30. Considere o diagrama de fase hipotético


representado esquematicamente na figura ao lado:
O que representam os pontos A , B , C , D e E ? E
PRESSÃO (atm)

C
D

A B

TEMPERATURA (K)
CONSTANTES

23 −1
Constante de Avogadro = 6, 02 x 10 mol
−1 −1 −1 −1
= 9, 65 x 10 A s mol = 9, 65 x 10 J V mol
4 4 4
Constante de Faraday (F) = 9, 65 x 10 C mol
Volume molar de gás ideal = 22, 4 L (CNTP)
−19
Carga elementar = 1, 602 x 10 C
−2 −1 −1 −1 −1 −1 −1 −1 −1
Constante dos gases (R ) = 8, 21 x 10 atm L K mol = 8, 31 J K mol = 1, 98 cal K mol = 62, 4 mmHg L K mol
−2
Constante gravitacional (g) = 9,81 m s
DEFINIÇÕES

−2
Pressão de 1 atm = 760 mmHg = 101 325 N m = 760 Torr = 1, 01325 bar
−2
1 J = 1 N m = 1 kg m s
2

Condições normais de temperatura e pressão (CNTP): 0 º C e 760 mmHg


Condições ambientes: 25 º C e 1 atm
−1
Condições-padrão: 1 bar ; concentração das soluções = 1 mol L (rigorosamente: atividade unitária das espécies); sólido com
estrutura cristalina mais estável nas condições de pressão e temperatura em questão.
(s) = sólido . (ℓ ) = líquido . (g) = gás . (aq) = aquoso . (CM) = circuito metálico . (conc) = concentrado .
(ua) = unidades arbitrárias . [ X ] = concentração da espécie química X em mol L .
−1

MASSAS MOLARES

Elemento Número Massa Molar Elemento Número Massa Molar


Químico Atômico −1 Químico Atômico −1
(g.mol ) (g.mol )
H 1 1,01 K 19 39,10
Li 3 6,94 Ca 20 40,08
B 5 10,81 Cr 24 52,00
C 6 12,01 Mn 25 54,94
N 7 14,01 Fe 26 55,85
O 8 16,00 Zn 30 65,38
F 9 19,00 Br 35 79,90
Na 11 22,99 Ag 47 107,90
P 15 30,97 Pt 78 195,08
S 16 32,07 Hg 80 200,59
Cl 17 35,45 Pu 94 238

Questão 1. Assinale a opção que apresenta os instrumentos de medição de volume mais indicados para a
realização de uma titulação.

A ( ) Bureta e erlenmeyer B ( ) Proveta e erlenmeyer


C ( ) Pipeta volumétrica e erlenmeyer D ( ) Proveta e béquer
E ( ) Pipeta volumétrica e béquer

Questão 2. Cinco amostras idênticas de um mesmo metal são aquecidas a diferentes temperaturas até à
incandescência. Assinale a opção que apresenta a cor da amostra submetida a uma maior temperatura.

A ( ) Vermelho B ( ) Laranja C ( ) Amarelo D ( ) Verde E ( ) Branco

Questão 3. O elemento Plutônio-238 é utilizado para a geração de eletricidade em sondas espaciais.


Fundamenta-se essa utilização porque esse isótopo tem

A( ) longo tempo de meia-vida e é emissor de partículas beta.


B( ) longo tempo de meia-vida e é emissor de partículas gama.
C( ) longo tempo de meia-vida e é emissor de partículas alfa.
D( ) longo tempo de meia-vida e é emissor de partículas delta.
E( ) tempo de meia-vida curto e é emissor de partículas alfa.
Questão 4. Sendo o pK do NH 4 OH igual a 4,74, o pH de uma solução aquosa 0,10 mol L−1 em NH 4 Cℓ é

A ( ) 1,00. B ( ) 3,74. C ( ) 4,74. D ( ) 5,13. E ( ) 8,87.

Questão 5. Considere uma reação química hipotética representada pela equação X → Produtos. São feitas as
seguintes proposições relativas a essa reação:

I. Se o gráfico de [ X ] em função do tempo for uma curva linear, a lei de velocidade da reação dependerá
somente da constante de velocidade.
1
II. Se o gráfico de em função do tempo for uma curva linear, a ordem de reação será 2.
[X]
III. Se o gráfico da velocidade da reação em função de [ X ] for uma curva linear, a ordem de reação será 1.
IV. Se o gráfico da velocidade de reação em função de [ X ] for uma curva linear, a ordem de reação será 2.
2

Das proposições acima, está(ão) CORRETA(S)

A ( ) apenas I. B ( ) apenas I e II. C ( ) apenas I, III e IV. D ( ) apenas III. E ( ) todas.

Questão 6. Considere as seguintes comparações entre as respectivas temperaturas de fusão dos polímeros
representados pelas suas unidades repetitivas:

I. A do H [ OCH CH OOC(CH ) CO ]nOCH CH OH


2 2 2 4 2 2 é maior que a do H [ OO C- COOCH2 CH 2 ]n OH
O

II. A do [ CH CH ]
2 2 é maior que a do [ CH2CH2O ]n
n

III. A do [ CH2- -CH2 ] é maior que a do [ CH CH ]


2 2
n n

IV. A do [ NH ( CH2)7CO ]n é maior que a do [ NH(CH ) CO ]n 2 3

Assinale a opção que apresenta a(s) comparação(ões) ERRADA(S).

A( ) Apenas I
B( ) Apenas I e IV
C( ) Apenas II e III
D( ) Apenas III e IV
E( ) Apenas IV
Questão 7. Considere a reação química hipotética realizada em sistema fechado a pressão e temperatura
constantes representada pela equação X + Y W + Z . Supondo que no início da reação haja apenas os
reagentes X e Y, e considerando um intervalo de tempo que se estende de t = 0 até um instante t após o
equilíbrio ter sido atingido, assinale a opção que apresenta a variação da energia livre de Gibbs.

A( ) B( )

G G

tempo tempo

C( ) D( )

G G

tempo tempo

E( )

tempo

Questão 8. Borbulha-se gás cloro em solução aquosa diluída de hidróxido de sódio a 25 °C. Assinale a opção
que contém apenas produtos clorados resultantes.

A ( ) Cℓ − , CℓO3− B( ) OCℓ − , Cℓ − C ( ) CℓO3− , CℓO 4− , Cℓ −


D ( ) CℓO3− , OCℓ − E( ) CℓO −4 , CℓO3−
Questão 9. O grau de dissociação, α, do ácido acético em solução aquosa 0,10 mol L−1 é 100 vezes menor que o
do ácido clorídrico também em solução aquosa 0,10 mol L−1 . Com base nestas informações, pode-se afirmar que
o pH da solução aquosa do ácido acético 0,10 mol L−1 é

A ( ) zero. B ( ) um. C ( ) dois. D ( ) três. E ( ) quatro.

Questão 10. Para determinar a entalpia de vaporização do composto hipotético MX 4 (ℓ) , o mesmo foi colocado
num recipiente equipado com uma serpentina de aquecimento resistivo, a 80 °C e sob pressão de 1,0 bar. Para a
manutenção da temperatura, foi utilizada uma fonte de 30 V com passagem de corrente de 900 mA durante
30 s, tendo sido vaporizados 2,0 g de MX 4 (ℓ) . Sabendo que a massa molar desse composto é 200 g mol−1 ,
assinale a opção que apresenta a entalpia molar de vaporização em kJ mol−1 , a 80 °C.

A ( ) 4,1 B ( ) 8,1 C ( ) 81 D ( ) 405 E ( ) 810

Questão 11. Os óxidos de metais de transição podem ter caráter ácido, básico ou anfótero. Assinale a opção
que apresenta o caráter dos seguintes óxidos: CrO, Cr2O3 e CrO3.

A ( ) Ácido, anfótero, básico B ( ) Ácido, básico, anfótero C ( ) Anfótero, ácido, básico


D ( ) Básico, ácido, anfótero E ( ) Básico, anfótero, ácido

Questão 12. Considere as seguintes reações químicas e respectivas constantes de equilíbrio:

N 2 (g) + O 2 (g) ⇌ 2NO(g) K1


2NO(g) + O 2 (g) ⇌ 2NO 2 (g) K2
1
NO 2 (g) ⇌ N 2 (g) + O 2 (g) K3
2

Então, K3 é igual a
1 2
1 1 1  1  2  1 
A( ) . B( ) . C( ) . D( )   . E( )   .
(K1K 2 ) (2K1K 2 ) (4K1K 2 )  K 1K 2   K1K 2 

Questão 13. É de 0,76 V a força eletromotriz padrão, E°, de uma célula eletroquímica, conforme a reação

Zn(s) + 2H + (aq) → Zn 2 + (aq) + H 2 (g).

Na concentração da espécie de Zn 2+ igual a 1, 0 mol L−1 e pressão de H2 de 1,0 bar, a 25 °C, foi verificado que a
força eletromotriz da célula eletroquímica é de 0,64 V. Nestas condições, assinale a concentração de íons H+
em mol L−1 .

A ( ) 1, 0 x 10 −12 B( ) 4, 2 x 10 −4 C ( ) 1, 0 x 10−4 D ( ) 1, 0 x 10−2 E ( ) 2, 0 x 10−2

Questão 14. Uma mistura de metanol e água a 25 °C apresenta o volume parcial molar de água igual a
17,8 cm3 mol−1 e o volume parcial molar do metanol igual a 38, 4 cm 3 mol−1 . Com base nestas informações e
sendo a massa específica do metanol de 0, 791 g cm −3 e a da água igual a 1, 000 g cm −3 , assinale a opção
CORRETA do volume total (emcm3 ) quando se adicionam 15 cm3 de metanol em 250 cm3 de água nessa
temperatura.

A ( ) 250 B ( ) 255 C ( ) 262 D ( ) 270 E ( ) 280


Questão 15. Para uma molécula diatômica, a energia potencial em função
da distância internuclear é representada pela figura ao lado. As linhas
horizontais representam os níveis de energia vibracional quanticamente
permitidos para uma molécula diatômica. Uma amostra contendo um mol
de moléculas diatômicas idênticas, na forma de um sólido cristalino, pode
ser modelada como um conjunto de osciladores para os quais a energia
potencial também pode ser representada qualitativamente pela figura. Em
relação a este sólido cristalino,
alino, são feitas as seguintes proposições:

I. À temperatura de 0 K, a maioria dos osciladores estará no estado


vibracional fundamental, cujo número quântico vibracional, n, é igual a zero.
II. À temperatura de 0 K, todos os osciladores estarão no estado vibracional fundamental,
fundamental cujo número
quântico vibracional, n,, é igual a zero.
zero
III. O movimento vibracional cessa a 0 K.
IV. O movimento vibracional não cessa a 0 K.
V. O princípio de incerteza de Heisenberg será violado se o movimento vibracional cessar.
Das proposições acima estão CORRETAS

A ( ) apenas I e III. B ( ) apenas II e III. C ( ) apenas I, IV e V.


D ( ) apenas II, IV e V. E ( ) apenas II, III e V.

Questão 16. Dois béqueres,


équeres, denominados “X”
“ e “Y”, encontram-se se dentro de um recipiente hermeticamente
fechado, à pressão de 1 bar e temperatura de 298 K. O béquer “X”” contém 100 mL de uma solução aquosa de
cloreto de sódio cuja concentração é 0, 3mol L−1 . O béquer “Y”” contém 100 mL de uma solução aquosa de
cloreto de sódio cuja concentração é 0,1mol L−1 . Se o recipiente for mantido fechado e em repouso até alcançar
o equilíbrio termodinâmico, assinale o volume final (em mL) da solução no béquer
équer “Y”:

A ( ) 25 B ( ) 50 C ( ) 100 D ( ) 150 E ( ) 200

Questão 17. São feitas as seguintes comparações sobre as capacidades


capacidade caloríficas
loríficas de diferentes substâncias
puras, todas à temperatura ambiente:
I. A capacidade calorífica da água é menor que a do peróxido de hidrogênio.
II. A capacidade calorífica do bromo é menor que a do tetracloreto de carbono.
III. A capacidade calorífica do metanol é menor que a do mercúrio.
Assinale a opção que apresenta a(s) comparação(ões)
comparaç CORRETA(S).

A ( ) Apenas I B ( ) Apenas I e II C ( ) Apenas II


D ( ) Apenas II e III E ( ) Apenas III

Questão 18. Considere a reação química representada pela equação NH 3 + BF3 → H 3 NBF3 . Pode-se afirmar
que o BF3 age

A ( ) como ácido de Bronsted. B ( ) como ácido de Lewis. C ( ) como base de Bronsted.


D ( ) como base de Lewis. E ( ) tanto como ácido como base.
Questão 19. A figura mostra a variação da massa específica de uma
substância pura com a temperatura à pressão de 1 bar. Então, é
CORRETO afirmar que Tx pode representar a temperatura de

A( ) ebulição da água.
B( ) ebulição do benzeno.
C( ) fusão da água.
D( ) fusão do benzeno.
E( ) fusão do dióxido de carbono.

Questão 20. Contribuíram de forma direta para o desenvolvimento do


d conceito de pressão atmosférica

A ( ) Friedrich August Kekulé


kulé e John Dalton. B ( ) Michael Faraday e Fritz Haber.
Haber
C ( ) Galileu Galilei e Evangelista Torricelli.
Torricelli D ( ) Jöns Jacob Berzelius e Eduard Büchner.
B
E ( ) Robert Bunsen e Henry Louis Le Chatelier.
Ch

AS QUESTÕES DISSERTATIVAS, NUMERADAS DE 21 A 30, DEVEM SER RESPONDIDAS NO CADERNO


DE SOLUÇÕES.
AS QUESTÕES NUMÉRICAS DEVEM M SER DESENVOLVIDAS SEQUENCIALMENTE ATÉ O FINAL.

Questão 21. 3,64 gramas de fosfeto de cálcio foram adicionados a uma certa quantidade de água. Após a reação
completa, todo o produto gasoso formado foi recolhido em um recipiente de 8,2 mL. Calcule
Calcul o valor numérico
da pressão, em atm, exercida pelo produto gasoso a 27 °C.

Questão 22. Considere uma solução saturada do sal MX que é pouco solúvel em água destilada a 25 °C.
°C Seja y
−7 −1 −1
a condutância da água destilada e ( y + 2, 0.10 ) ohm cm a condutância da solução. Sabendo que as
condutividades iônicas molares dos íons M + e X − são, respectivamente, 60 ohm −1 cm 2 mol−1 e
d MX em água em mol dm −3 .
40 ohm −1 cm 2 mol −1 , determine a solubilidade do

Questão 23. Considere uma reação genérica reversível A + B ⇌ 2C e os dados cinéticos para a reação direta
(D) e inversa (I):

Sentido da reação Constante de velocidade Energia de ativação


kD E a,D
A + B → 2C
3 1
2C →A+B kI = kD E a,I = E a ,D
2 2

a) Desenhe o gráfico de energia potencial versus coordenada da reação direta.


b) Determine o valor numérico da constante
cons de equilíbrio da reação.
c) Qual sentido da reação é endotérmico?

Questão 24. Uma amostra de ferro foi totalmente dissolvida a Fe(II) em 25,0 mL de solução aquosa ácida. A
(II) foi titulada com 20 mL de uma solução aquosa 0, 01mol L−1 em permanganato de
seguir, a solução de Fe(II)
potássio. Baseando-se
se nessas informações, responda os seguintes itens:
a) Qual é a equação iônica balanceada que descreve a reação de titulação?
titulação
b) É necessária a adição de indicador para visualização do ponto final da titulação? Por quê?
c) Qual será a variação de cor e as espécies responsáveis por essa variação no ponto de viragem?
d) Qual é o valor numérico da massa (em g) de ferro na amostra dissolvida, considerando que não há
interferentes na solução?
Questão 25. Descreve-se o seguinte experimento:
i. São dissolvidas quantidades iguais de ácido benzóico e ciclohexanol em diclorometano.
ii. É adicionada uma solução aquosa 10% massa/massa em hidróxido de sódio à solução descrita no item
(i) sob agitação. A seguir, a mistura é deixada em repouso até que o equilíbrio químico seja atingido.
Baseando-se nessas informações, pedem-se:

a) Apresente a(s) fase(s) líquida(s) formada(s).


b) Apresente o(s) componente(s) da(s) fase(s) formada(s).
c) Justifique a sua resposta para o item b, utilizando a(s) equação(ões) química(s) que representa(m) a(s)
reação(ões).

Questão 26. Considere um elemento galvânico formado por dois semielementos contendo soluções aquosas
ácidas e cujos potenciais na escala do eletrodo de hidrogênio (E°) nas condições–padrão são
E ο ( Pt / PtO 2 ) = 1, 00 V e E ο ( Br2 / BrO3− ) = 1, 48 V .
Baseando-se nessas informações, pedem-se:

a) Calcule o valor numérico da força eletromotriz do elemento galvânico.


b) Apresente as equações químicas que representam as semirreações do anodo e catodo.
c) Apresente a equação química que representa a reação global.

Questão 27. Com base no modelo atômico de Bohr:

a) Deduza a expressão para o módulo do momento angular orbital de um elétron na n-ésima órbita de Bohr,
em termos da constante da Planck, h.
b) O modelo de Bohr prevê corretamente o valor do módulo do momento angular orbital do elétron no átomo
de hidrogênio em seu estado fundamental? Justifique.

Questão 28. Escreva a fórmula estrutural do produto majoritário formado na reação entre 0,1 mol de tolueno
(metilbenzeno) e 0,1 mol de Cℓ 2 nas seguintes condições:

a) Ausência de luz e presença de pequena quantidade de Fe(s).


b) Presença de luz e ausência de Fe(s).

Questão 29. Considere os compostos orgânicos metilfenilcetona e propanona.

a) Apresente a equação química que representa o equilíbrio tautomérico para cada um dos compostos.
b) Qual das duas cetonas acima tem maior conteúdo enólico? Justifique.

Questão 30. Desenhe a fórmula estrutural (IUPAC) das seguintes espécies químicas aromáticas.
a) Naftaleno
b) Fenantreno
c) Antraceno
d) Peróxido de benzoíla
CONSTANTES

−1
= 6, 02 × 10 mol
23
Constante de Avogadro
−1 −1 −1 −1
= 9, 65 × 10 C ⋅ mol= 9, 65 × 10 A ⋅ s ⋅ mol = 9, 65 × 10 J ⋅ V ⋅ mol
4 4 4
Constante de Faraday ( F )
Volume molar de gás ideal = 22,4 L ( CNTP )
− 19
Carga elementar = 1,602 ×10 C
Constante dos gases ( R ) = 8,21×10 − 2 atm ⋅ L ⋅ K − 1 ⋅ mol − 1 = 8,31 J ⋅ K − 1 ⋅ mol − 1 = 1,98 cal ⋅ K − 1 ⋅ mol − 1 =
62,4 mmHg ⋅ L ⋅ K − 1 ⋅ mol − 1
Constante gravitacional ( g ) = 9,81 m ⋅ s − 2

DEFINIÇÕES

Pressão de 1 atm = 760 mmHg = 101 325 N ⋅ m − 2 = 760 Torr


1 J = 1 N ⋅ m = 1 kg ⋅ m 2 ⋅ s − 2 ; 1 pm = 1×10 − 12 m ; 1 eV = 1,602 ×10 − 19 J
Condições normais de temperatura e pressão ( CNTP ) : 0 °C e 760 mmHg
Condições ambientes: 25 °C e 1 atm
Condições-padrão: 25 °C e 1 atm ; concentração das soluções = 1 mol ⋅ L − 1 (rigorosamente: atividade unitária
das espécies); sólido com estrutura cristalina mais estável nas condições de pressão e temperatura em questão.
( s ) = sólido. (ℓ) = líquido. ( g ) = gás. ( aq ) = aquoso. ( CM ) = circuito metálico. ( conc ) = concentrado.
( ua ) = unidades arbitrárias. [ A] = concentração da espécie química A em mol ⋅ L − 1 .

MASSAS MOLARES

Elemento Número Massa Molar Elemento Número Massa Molar


Químico Atômico ( g ⋅ mol − 1 ) Químico Atômico ( g ⋅ mol )
−1

H 1 1,01 Ti 22 47,87
B 5 10,81 Cr 24 52,00
C 6 12,01 Ni 28 58,69
N 7 14,01 Cu 29 63,55
O 8 16,00 Zn 30 65,38
F 9 19,00 As 33 74,92
Na 11 22,99 Se 34 78,96
Mg 12 24,31 Ag 47 107,90
Al 13 26,98 Sn 50 118,70
P 15 30,97 Te 52 127,60
S 16 32,07 I 53 126,90
Cℓ 17 35,45 Xe 54 131,30
Ar 18 39,95 Au 79 197,00
K 19 39,10 U 92 238,00
Questão 1. Metanol (CH3OH) e água deuterada (D2O) são misturados numa razão volumétrica de 7:3,
respectivamente, nas condições ambientes. A respeito dessa mistura, são feitas as seguintes afirmações:

I. Imediatamente após a mistura das duas substâncias é observada uma fase única.
II. Após o equilíbrio, observa-se uma fase única que contém as seguintes substâncias: CH3OH, D2O, CH3OD e
HOD.
III. Se for adicionado um cubo de D2O(s) à mistura, este flutuará na superfície da mistura líquida.

Assinale a opção que contém a(s) afirmação(ões) CORRETA(S).

A( ) Apenas I B( ) Apenas I e II C( ) Apenas II


D( ) Apenas III E( ) Todas

Questão 2. Considere os seguintes compostos:

I. CH3CH2OH II. CH3CH2COOCH3 III. HCℓ


IV. H3PO4 V. POCℓ3

Assinale a opção que contém os produtos que podem ser formados pela reação de ácido acético com pentacloreto
de fósforo.

A( ) Apenas I, III e IV B( ) Apenas I e IV C( ) Apenas II e III


D( ) Apenas II e V E( ) Apenas III e V

Questão 3. Nas condições ambientes são feitas as seguintes afirmações sobre o ácido tartárico:

I. É um sólido cristalino.
II. É solúvel em tetracloreto de carbono.
III. É um ácido monoprótico quando em solução aquosa.
IV. Combina-se com íons metálicos quando em solução aquosa.

Das afirmações acima, está(ão) CORRETA(S) apenas

A( ) I e II. B( ) I e IV. C( ) II e III.


D( ) III e IV. E( ) IV.

Questão 4. Considere que 1 mol de uma substância sólida está em equilíbrio com seu respectivo líquido na
temperatura de fusão de −183 °C e a 1 atm. Sabendo que a variação de entalpia de fusão dessa substância é
6,0 kJ ∙ mol −1, assinale a opção que apresenta a variação de entropia, em J ∙ K −1 ∙ mol −1.

A( ) − 20 B( ) − 33 C( ) + 50 D( ) + 67 E( ) + 100

Questão 5. Assinale a opção que contém o(s) produto(s) formado(s) durante o aquecimento de uma mistura de
Cu2O e Cu2S, em atmosfera inerte.

A( ) CuSO4 B( ) Cu2SO3 C( ) Cu e SO2


D( ) Cu e SO3 E( ) CuO e CuS

Questão 6. Assinale a opção que contém o momento angular do elétron na 5ª órbita do átomo de hidrogênio,
segundo o modelo atômico de Bohr.

A( ) h/2π B( ) h/π C( ) 2,5 h/2π


D( ) 2,5 h/π E( ) 5 h/π
Questão 7. Assinale a opção que contém a base conjugada de OH─.

A( ) O2─ B( ) O─ C( ) O2─ D( ) H2O E( ) H+

Questão 8. Assinale a opção que contém o número de oxidação do crômio no composto [Cr(NH3)4Cℓ2]+.

A( ) Zero B( ) +1 C( ) +2 D( ) +3 E( ) +4

Questão 9. Assinale a opção que apresenta o elemento químico com o número CORRETO de nêutrons.

19 24
A( ) 9F tem zero nêutrons. B( ) 12 Mg tem 24 nêutrons.
197 75
C( ) 79 Au tem 79 nêutrons. D( ) 33 As tem 108 nêutrons.
238
E( ) 92 U tem 146 nêutrons.

Questão 10. A pressão de vapor de uma solução ideal contendo um soluto não-volátil dissolvido é diretamente
proporcional à

A( ) fração molar do soluto.


B( ) fração molar do solvente.
C( ) pressão osmótica do soluto.
D( ) molaridade, em mol ∙ L −1, do solvente.
E( ) molalidade, em mol ∙ kg −1, do solvente.

Questão 11. Considere um mol de um gás que se comporta idealmente, contido em um cilindro indeformável
provido de pistão de massa desprezível, que se move sem atrito. Com relação a este sistema, são feitas as
seguintes afirmações:

I. Se o gás for resfriado contra pressão externa constante, o sistema contrai-se.


II. Se pressão for exercida sobre o pistão, a velocidade média das moléculas do gás aumenta.
III. Se o sistema for aquecido a volume constante, a velocidade média das moléculas aumenta,
independentemente da natureza do gás.
IV. A velocidade média das moléculas será maior se o gás for o xenônio e menor se for o argônio.

Das afirmações acima, está(ão) ERRADA(S) apenas

A( ) I e II. B( ) I, III e IV. C( ) II e III.


D( ) II e IV. E( ) IV.

Questão 12. Considere três cubos maciços de 2 cm de aresta, constituídos, respectivamente, de Cr, Ni e Ti
puros. Os três cubos são aquecidos até 80 ºC e cada cubo é introduzido em um béquer contendo 50 g de água a
10 ºC. Com base nas informações constantes da tabela abaixo, assinale a opção que apresenta a relação
CORRETA entre as temperaturas dos cubos, quando o conteúdo de cada béquer atingir o equilíbrio térmico.

Substância Massa específica (g ∙ cm −3) Calor específico (J ∙ g −1 ∙ K −1)


H2O 1,00 4,18
Ti 4,54 0,52
Cr 7,18 0,45
Ni 8,90 0,44

A( ) TCr > TNi > TTi . B( ) T=


Ni TTi > TCr . C( ) TNi > TCr > TTi .
D( ) TTi > TCr > TNi . E( ) TTi > TCr = TNi .
Questão 13. Considere a reação química genérica A → B + C . A concentração do reagente [A] foi
acompanhada ao longo do tempo, conforme apresentada na tabela que também registra os logaritmos neperianos
(ℓn) desses valores e os respectivos recíprocos (1/[A]).

t (s) [A] (mol ∙ L −1) ℓn [A] 1/[A] (L ∙ mol −1)


0 0,90 − 0,11 1,11
100 0,63 − 0,46 1,59
200 0,43 − 0,84 2,33
300 0,30 − 1,20 3,33
400 0,21 − 1,56 4,76
500 0,14 − 1,97 7,14
600 0,10 − 2,30 10,00

Assinale a opção que contém a constante de velocidade CORRETA desta reação.

A( ) 4 x 10 ─3 s ─1 B( ) 4 x 10 ─3 mol ∙ L ─1 ∙ s ─1
C( ) 4 x 10 ─3 L ∙ mol ─1 ∙ s ─1 D( ) 4 x 103 s ─1
E( ) 4 x 103 mol ∙ L─1 ∙ s ─1

Questão 14. São feitas as seguintes comparações de valores de pKa de compostos orgânicos:

I. pKa (CH3COOH) > pKa (CℓCH2COOH)


II. pKa (F3CCOOH) > pKa (Cℓ3CCOOH)
III. pKa (CH3CH2CHCℓCOOH) > pKa (CH3CHCℓCH2COOH)

Das comparações acima, está(ão) CORRETA(S) apenas

A ( ) I. B ( ) I, II e III. C ( ) I e III. D ( ) II. E( ) II e III.

Questão 15. São feitas as seguintes afirmações sobre o que Joule demonstrou em seus experimentos do século
XIX:

I. A relação entre calor e trabalho é fixa.


II. Existe um equivalente mecânico do calor.
III. O calor pode ser medido.

Das afirmações acima, está(ão) CORRETA(S) apenas

A( ) I. B ( ) I, II e III. C( ) I e III. D ( ) II. E ( ) II e III.

Questão 16. Joseph Black (1728-1799), médico, químico e físico escocês, conceituou o calor específico. Esta
conceituação teve importantes aplicações práticas, dentre elas a máquina a vapor, desenvolvida pelo engenheiro
escocês James Watt (1736-1819). Que componente do motor a vapor desenvolvido por Watt revolucionou seu
uso e aplicação?

A( ) Boiler ou fervedor B( ) Bomba de recalque C( ) Caldeira


D( ) Condensador E( ) Turbina a vapor

Questão 17. Assinale a opção que contém a concentração (em mol ∙ L −1) de um íon genérico M+, quando se
adiciona um composto iônico MX sólido até a saturação a uma solução aquosa 5 x 10─3 mol ∙ L −1 em PX.
Dado Kps(MX) = 5 x 10 ─12.

A( ) 2,3 x 10 ─6 B( ) 1,0 x 10 ─7 C( ) 2,3 x 10 ─8


D( ) 1,0 x 10 ─9 E( ) 1,0 x 10 ─10
Questão 18. Considere os seguintes compostos:

I. álcoois II. aldeídos III. carbono particulado (negro de fumo) IV. cetonas

Dos componentes acima, é (são) produto(s) da combustão incompleta do n-octano com ar atmosférico apenas

A( ) I e II. B( ) I e IV. C( ) II e III.


D( ) III. E( ) IV.

Questão 19. Considere a reação do tetraborato de sódio:

Na 2 B4 O7 . 10H 2 O( s ) + H 2SO 4 (aq ) → 4B(OH)3 (aq ) + Na 2SO 4 (aq ) + 5H 2 O (ℓ)

Em relação ao produto da reação B(OH) 3 são feitas as seguintes afirmações:

I. B(OH) 3 é um ácido de Brönsted – Lorry.


II. B(OH) 3 é uma base de Arrhenius.
III. O produto da primeira ionização do B(OH)3(aq) é o BO(OH) −
2 (aq).

Das afirmações acima, está(ão) CORRETA(S) apenas

A( ) I. B( ) I e III. C( ) II. D( ) II e III. E ( ) III.

Questão 20. Considere uma célula a combustível alcalina (hidrogênio-oxigênio) sobre a qual são feitas as
seguintes afirmações:

I. Sob condição de consumo de carga elétrica, a voltagem efetiva de serviço desse dispositivo eletroquímico
é menor que a força eletromotriz da célula.
II. O combustível (hidrogênio gasoso) é injetado no compartimento do anodo e um fluxo de oxigênio gasoso
alimenta o catodo dessa célula eletroquímica.
III. Sendo o potencial padrão dessa célula galvânica igual a 1,229 VEPH (volt na escala padrão do hidrogênio),
a variação de energia livre de Gibbs padrão (∆G°) da reação global do sistema redox atuante é igual a
− 237,2 kJ ∙ mol−1.

Das afirmações acima, está(ão) CORRETA(S) apenas

A( ) I. B( ) I, II e III. C ( ) I e III. D ( ) II. E ( ) II e III.

AS QUESTÕES DISSERTATIVAS, NUMERADAS DE 21 A 30, DEVEM SER RESPONDIDAS NO


CADERNO DE SOLUÇÕES.
AS QUESTÕES NUMÉRICAS DEVEM SER DESENVOLVIDAS ATÉ O FINAL, COM
APRESENTAÇÃO DO VALOR ABSOLUTO DO RESULTADO.

Questão 21. O dióxido de potássio tem várias aplicações, entre as quais, a

(a) produção de peróxido de hidrogênio (g) na presença de água.


(b) conversão de dióxido de carbono (g) para oxigênio (g).
(c) absorção de dióxido de carbono (g) na presença de H2O com formação de oxigênio (g).

Apresente as equações químicas balanceadas que representam as reações descritas nos itens acima.
Questão 22. São descritos dois experimentos:

I. Ovo cozido em água fervente teve sua casca quebrada, de modo que parte de sua clara permaneceu em
contato com esta água, na qual a seguir foi também imerso um objeto polido de prata. Após um certo período
de tempo, observou-se o escurecimento desse objeto, que foi retirado da água e lavado.
II. Em um béquer, foi aquecida água até a fervura e adicionada uma colher das de sopa de cloreto de sódio. A
seguir, esta solução foi transferida para um béquer revestido com papel alumínio. O objeto de prata utilizado
no experimento I foi então imerso nesta solução e retirado após alguns minutos.

Em relação a esses experimentos:

(a) apresente a equação global que representa a reação química ocorrida na superfície do objeto de prata no
experimento I e calcule a diferença de potencial elétrico da reação química.
(b) preveja a aparência do objeto de prata após a realização do segundo experimento.
(c) apresente a equação global da reação química envolvida no experimento II e sua diferença de potencial
elétrico.

Dados: Ag 2S( s ) + 2 e −  2Ag( s ) + S2− (aq ) E° = − 0, 691V


O 2 ( g ) + 4 H + (aq ) + 4 e −  2H 2 O () E° =1, 229 V
3+ −
Al (aq ) + 3e  Al( s ) E° = − 1, 662 V
Ag 2S( s ) + 2 H + (aq ) + 2 e −  2Ag( s ) + H 2S( g ) E° = − 0, 037 V

Questão 23. Apresente as equações que representam as reações químicas de nitração do tolueno, na presença de
ácido sulfúrico, levando a seus isômeros. Indique o percentual de ocorrência de cada isômero e seus respectivos
estados físicos, nas condições-padrão.

Questão 24. Escreva a reação de combustão completa de um hidrocarboneto genérico (Cα Hβ ) com ar
atmosférico. Considere a presença do nitrogênio gasoso no balanço estequiométrico da reação e expresse os
coeficientes estequiométricos dessa reação em função de α e β.

Questão 25. Em um processo de eletrodeposição, níquel metálico é eletrodepositado no catodo de uma célula
eletrolítica e permanece coeso e aderido a esse eletrodo. Sabendo que a massa específica do níquel metálico
(ρNi,25 °C) é igual a 8,9 x 103 kg ∙ m ─3 e que a espessura total da camada eletrodepositada, medida no final do
processo, foi de 2,0 x 10 ─6 m, calcule a densidade de corrente aplicada (admitida constante), expressa em
A ∙ m─2, considerando nesse processo uma eficiência de corrente de eletrodeposição de 100% e um tempo de
operação total de 900 s.

Questão 26. Água líquida neutra (pH = 7,0), inicialmente isenta de espécies químicas dissolvidas, é mantida em
um recipiente de vidro aberto e em contato com a atmosfera ambiente sob temperatura constante. Admitindo-se
que a pressão parcial do oxigênio atmosférico seja igual a 0,2 atm e sabendo-se que esse gás é solúvel em H2O(ℓ)
e que o sistema está em equilíbrio à temperatura de 25 °C, pedem-se:

(a) escrever a equação química balanceada da semirreação que representa o processo de redução de oxigênio
gasoso em meio de água líquida neutra e aerada.
(b) determinar o potencial de eletrodo (VEPH), à temperatura de 25 °C, da semirreação obtida no item (a),
considerando as condições estabelecidas no enunciado desta questão.
(c) determinar o valor numérico, expresso em kJ ∙ mol −1, da variação de energia livre de Gibbs padrão (∆Go) da
semirreação eletroquímica do item (a).

São dados: EoO − = 0, 401 VEPH VEPH = volt na escala padrão do hidrogênio
2 /OH

log =  n 2,303 0, 2 = 10(0,30 −1)


Questão 27. Considere uma mistura gasosa constituída de C3H8, CO e CH4. A combustão, em excesso de
oxigênio, de 50 mL dessa mistura gasosa forneceu 70 mL de CO2 (g). Determine o valor numérico do percentual
de C3H8 na mistura gasosa.

Questão 28. O ácido nítrico reage com metais, podendo liberar os seguintes produtos: NO (que pode ser
posteriormente oxidado na presença do ar), N2O, NO2 ou NH3 (que reage posteriormente com HNO3, formando
NH4NO3). A formação desses produtos depende da concentração do ácido, da natureza do metal e da
temperatura da reação.
Escreva qual(is) dos produtos citados acima é(são) formado(s) nas seguintes condições:

(a) Zn ( s ) + HNO3 muito diluído ( 2%)


(b) Zn ( s ) + HNO3 diluído (10%)
(c) Zn ( s ) + HNO3 concentrado
(d) Sn ( s ) + HNO3 diluído
(e) Sn ( s ) + HNO3 concentrado

Questão 29. Considere os seguintes dados:

Entalpia de vaporização da água a 25 °C: ∆ vap H = 44 kJ ⋅ mol−1


Massa específica da água a 25 °C: ρH2O = 1, 0 g ⋅ cm −3
Temperaturas de ebulição a 1 bar:
Teb,H2O =100 °C; Teb,H2S =− 60 °C; Teb,H2Se =− 41°C e Teb,H2Te =− 2 °C

Com base nestas informações:

(a) determine o valor numérico da energia liberada, em J, durante a precipitação pluviométrica de 20 mm de


chuva sobre uma área de (10 x 10) km2.
(b) justifique, em termos moleculares, por que H 2O apresenta Teb muito maior que outros calcogenetos de
hidrogênio.
(c) como se relaciona, em termos moleculares, a elevada Teb,H2O com a quantidade de energia liberada durante
uma precipitação pluviométrica?

Questão 30. Velocidades inciais (vi) de decomposição de peróxido de hidrogênio foram determinadas em três
experimentos (A, B e C), conduzidos na presença de I ─(aq) sob as mesmas condições, mas com diferentes
concentrações iniciais de peróxido ([H2O2]i), de acordo com os dados abaixo:

Experimento [H2O2]i (mol ∙ L ─1) vi (10─3 mol ∙ L ─1 ∙ s ─1)


A 0,750 2,745
B 0,500 1,830
C 0,250 0,915

Com base nestes dados, para a reação de decomposição do peróxido de hidrogênio:

(a) escreva a equação estequiométrica que representa a reação.


(b) indique a ordem desta reação.
(c) escreva a lei de velocidade da reação.
(d) determine o valor numérico da constante de velocidade, k.
(e) indique a função do I ─(aq) na reação.
INSTITUTO TECNOLÓGICO DE AERONÁUTICA

VESTIBULAR 2017

PROVA DE QUÍMICA

INSTRUÇÕES

1. Esta prova tem duração de quatro horas.


2. Não é permitido deixar o local de exame antes de decorridas duas horas do início da prova.
3. Você poderá usar apenas lápis (ou lapiseira), caneta preta de material transparente, borracha e régua. É proibido
portar qualquer outro material escolar.
4. Esta prova é composta de 20 questões de múltipla escolha (numeradas de 01 a 20) e de 10 questões dissertativas
(numeradas de 21 a 30).
5. As 20 questões de múltipla escolha correspondem a 50% do valor da prova e as questões dissertativas, aos 50%
restantes.
6. Você recebeu este caderno de questões e um caderno de soluções com duas folhas de rascunho. Verifique se o
caderno de questões está completo.
7. Numere sequencialmente de 21 a 30, a partir do verso da capa, cada página do caderno de soluções. O número
atribuído a cada página corresponde ao da questão a ser resolvida. Não escreva no verso da parte superior da capa
(região sombreada) do caderno de soluções. As folhas centrais coloridas deverão ser utilizadas apenas como
rascunho e, portanto, não devem ser numeradas e nem destacadas pelo candidato.
8. Cada questão de múltipla escolha admite uma única resposta.
9. As resoluções das questões dissertativas, numeradas de 21 a 30, podem ser feitas a lápis e devem ser apresentadas de
forma clara, concisa e completa. Respeite a ordem e o espaço disponível no caderno de soluções. Sempre que
possível, use desenhos e gráficos.
10. Antes do final da prova, você receberá uma folha de leitura óptica, destinada à transcrição das questões
numeradas de 1 a 20. Usando caneta preta de material transparente, assinale a opção correspondente à resposta
de cada uma das questões de múltipla escolha. Você deve preencher todo o campo disponível para a resposta, sem
extrapolar-lhe os limites, conforme instruções na folha de leitura óptica.
11. Cuidado para não errar no preenchimento da folha de leitura óptica. Se isso ocorrer, avise o fiscal, que lhe fornecerá
uma folha extra, com o cabeçalho devidamente preenchido.
12. Não haverá tempo suplementar para o preenchimento da folha de leitura óptica.
13. Na última página do caderno de soluções, existe uma reprodução da folha de leitura óptica, que deverá ser preenchida
com um simples traço a lápis durante a realização da prova.
14. A não devolução do caderno de soluções, do caderno de questões e/ou da folha de leitura óptica implicará a
desclassificação do candidato.
15. No dia 20/12/2016, a partir das 10:00 horas, o gabarito da parte objetiva desta prova estará disponibilizado no site do
ITA (www.vestibular.ita.br).
16. Aguarde o aviso para iniciar a prova. Ao terminá-la, avise o fiscal e aguarde-o no seu lugar.
CONSTANTES
Constante de Avogadro (NA) = 6,02 x 1023 mol 1

Constante de Faraday (F) = 9,65 x 104 C·mol 1


= 9,65 x 104 A·s·mol 1
= 9,65 x 104 J·V 1·mol 1

Volume molar de gás ideal = 22,4 L (CNTP)


19
Carga elementar = 1,602 x 10 C
Constante dos gases (R) = 8,21 x 10 atm·L·K 1·mol
2 1
= 8,31 J·K 1·mol 1
= 1,98 cal·K 1·mol 1
=
1 1
= 62,4 mmHg·L·K ·mol
2
Constante gravitacional (g) = 9,81 m·s
34
Constante de Planck (h) = 6,626 x 10 m2·kg·s 1

Velocidade da luz no vácuo = 3,0 x 108 m·s 1

DEFINIÇÕES

Pressão de 1 atm = 760 mmHg = 1,01325 x 105 N·m 2


= 760 Torr = 1,01325 bar
2 2
1 J = 1 N·m = 1 kg·m ·s . ln 2 = 0,693
Condições normais de temperatura e pressão (CNTP): 0° C e 760 mmHg
Condições ambientes: 25° C e 1 atm
Condições padrão: 1 bar; concentração das soluções = 1 mol·L 1 (rigorosamente: atividade unitária das espécies); sólido com estrutura
cristalina mais estável nas condições de pressão e temperatura em questão.
(s) = sólido. (ℓ) = líquido. (g) = gás. (aq) = aquoso. (CM) = circuito metálico. (conc) = concentrado.
(ua) = unidades arbitrárias. [X] = concentração da espécie química X em mol·L 1.

MASSAS MOLARES

Elemento Número Massa Molar Elemento Número Massa Molar


Químico Atômico (g·mol 1) Químico Atômico (g·mol 1)
H 1 1,01 Cl 17 35,45
He 2 4,00 K 19 39,10
Be 4 9,01 Cr 24 52,00
B 5 10,81 Mn 25 54,94
C 6 12,01 Fe 26 55,85
N 7 14,01 Ni 28 58,69
O 8 16,00 Cu 29 63,55
F 9 19,00 Zn 30 65,38
Na 11 22,99 Br 35 79,90
Mg 12 24,31 Pd 46 106,42
Al 13 26,98 Ag 47 107,87
Si 14 28,09 Xe 54 131,30
P 15 30,97 Pt 78 195,08
S 16 32,06 Hg 80 200,59

Questão 1. Pode-se utilizar metais de sacrifício para proteger estruturas de aço (tais como pontes, antenas e
cascos de navios) da corrosão eletroquímica. Considere os seguintes metais:

I. Alumínio II. Magnésio III. Paládio IV. Sódio V. Zinco

Assinale a opção que apresenta o(s) metal(is) de sacrifício que pode(m) ser utilizado(s).

A ( ) Apenas I, II e V. B ( ) Apenas I e III. C ( ) Apenas II e IV.


D ( ) Apenas III e IV. E ( ) Apenas V.

Questão 2. A reação do mercúrio metálico com excesso de ácido sulfúrico concentrado a quente produz um gás
mais denso do que o ar. Dois terços deste gás são absorvidos e reagem completamente com uma solução aquosa
de hidróxido de sódio, formando 12,6 g de um sal. A solução de ácido sulfúrico utilizada tem massa específica
igual a 1,75 g·cm 3 e concentração de 80 % em massa. Assinale a alternativa que apresenta o volume
consumido da solução de ácido sulfúrico, em cm3.

A ( ) 11 B ( ) 21 C ( ) 31 D ( ) 41 E ( ) 51

2
Questão 3. Um frasco fechado contém dois gases cujo comportamento é considerado ideal: hidrogênio
molecular e monóxido de nitrogênio. Sabendo que a pressão parcial do monóxido de nitrogênio é igual a 3/5 da
pressão parcial do hidrogênio molecular, e que a massa total da mistura é de 20 g, assinale a alternativa que
fornece a porcentagem em massa do hidrogênio molecular na mistura gasosa.

A ( ) 4% B ( ) 6% C ( ) 8% D ( ) 10% E ( ) 12%

Questão 4. A reação química genérica X Y tem lei de velocidade de primeira ordem em relação ao reagente
X. À medida que a reação ocorre a uma temperatura constante, é ERRADO afirmar que

A( ) a constante de velocidade da reação não se altera.


B( ) o tempo de meia-vida do reagente X permanece constante.
C( ) a energia de ativação da reação não se altera.
D( ) a velocidade da reação permanece constante.
E( ) a ordem de reação não se altera.

Questão 5. Barreiras térmicas de base cerâmica são empregadas em projetos aeroespaciais. Considere os
materiais a seguir:

I. BN II. Fe2O3 III. NaN3 IV. Na2SiO3 V. SiC

Assinale a opção que apresenta o(s) material(is) geralmente empregado(s) como componente(s) principal(is) de
barreiras térmicas em projetos aeroespaciais.

A ( ) Apenas I e V. B ( ) Apenas II. C ( ) Apenas III.


D ( ) Apenas III e IV. E ( ) Apenas V.

Questão 6. A adição de certa massa de etanol em água diminui a temperatura de congelamento do solvente em
18,6 oC. Sabendo que a constante crioscópica da água é de 1,86 oC·kg·mol 1, assinale a porcentagem em massa
do etanol nesta mistura.

A ( ) 10,0 %. B ( ) 18,6 %. C ( ) 25,0 %. D ( ) 31,5 %. E ( ) 46,0 %.

Questão 7. Na figura ao lado são respectivamente


apresentadas as curvas de titulação de 50 mL de
soluções aquosas 0,1 mol·L 1 dos ácidos I, II e III,
tituladas com uma solução aquosa 0,1 mol·L 1 em
NaOH. Baseado nas informações contidas na figura,
assinale opção ERRADA.

A ( ) A constante de ionização do ácido III é


aproximadamente 10 9.
B ( ) A região W da curva de titulação do ácido II é
uma região-tampão.
C ( ) No ponto X o pH da solução I é igual ao pKa
do ácido I.
D ( ) O ponto Y é o ponto de equivalência do ácido
II.
E ( ) No ponto Z, para todos os ácidos o pH só
depende da quantidade em excesso de OH
adicionada.

3
Questão 8. Considere duas soluções, X e Y, de um mesmo soluto genérico. A solução X tem 49% em massa do
soluto, enquanto a solução Y possui 8% em massa do mesmo soluto. Quer-se obter uma terceira solução, que
tenha 20% em massa deste soluto, a partir da mistura de um volume VX da solução X com um volume VY da
solução Y. Considerando que todas as soluções envolvidas exibem comportamento ideal, assinale a opção que
apresenta a razão VX/VY CORRETA.

A ( ) 12/29. B ( ) 29/12. C ( ) 19/12. D ( ) 12/19. E ( ) 8/49.

Questão 9. O diagrama de van Arkel-Ketelar apresenta uma


visão integrada das ligações químicas de compostos
binários, representando os três tipos clássicos de ligação nos
vértices de um triângulo. Os vértices esquerdo e direito da
base correspondem, respectivamente, aos elementos menos
e mais eletronegativos, enquanto o vértice superior do
triângulo representa o composto puramente iônico. Com
base no diagrama, assinale a opção que apresenta o
composto binário de maior caráter covalente.

A ( ) CCl4 B ( ) C3N4 C ( ) CO2


D ( ) NO E ( ) OF2

Questão 10. São feitas as seguintes proposições a respeito de reações químicas orgânicas:

I. Etanoato de etila com amônia forma etanamida e etanol.


II. Ácido etanóico com tricloreto de fósforo, a quente, forma cloreto de etanoíla.
III. n-Butilbenzeno com permanganato de potássio, a quente, forma ácido benzóico e dióxido de carbono.

Das proposições acima, está(ão) CORRETA(S)

A( ) apenas I.
B( ) apenas I e II.
C( ) apenas II.
D( ) apenas II e III.
E( ) I, II e III.

Questão 11. Em relação às funções termodinâmicas de estado de um sistema, assinale a proposição ERRADA.

A ( ) A variação de energia interna é nula na expansão de n mols de um gás ideal a temperatura constante.
B ( ) A variação de energia interna é maior do que zero em um processo endotérmico a volume constante.
C ( ) A variação de entalpia é nula em um processo de várias etapas em que os estados inicial e final são os
mesmos.
D ( ) A variação de entropia é maior do que zero em um processo endotérmico a pressão constante.
E ( ) A variação de entropia é nula quando n mols de um gás ideal sofrem expansão livre contra pressão
externa nula.

Questão 12. A 25 oC, o potencial da pilha descrita abaixo é de 0,56 V. Sendo Eo(Cu2+/Cu) = + 0,34 V, assinale
a opção que indica aproximadamente o valor do pH da solução.

Pt(s)|H2(g, 1 bar), H+(aq, x mol·L 1)||Cu2+(aq, 1,0 mol·L 1)|Cu(s)

A ( ) 6,5 B ( ) 5,7 C ( ) 3,7 D ( ) 2,0 E ( ) 1,5

4
Questão 13. A pressão de vapor da água pura é de 23,8 torr a 25 oC. São dissolvidos 10,0 g de cloreto de sódio
em 100,0 g de água pura a 25 oC. Assinale a opção que indica o valor do abaixamento da pressão de vapor da
solução, em torr.

A ( ) 22,4 B ( ) 11,2 C ( ) 5,6 D ( ) 2,8 E ( ) 1,4

Questão 14. Considere que a decomposição do N2O5, representada pela equação química global
2 N 2 O5 4 NO 2 O2 ,
apresente lei de velocidade de primeira ordem. No instante inicial da reação, a concentração de N2O5 é de
0,10 mol·L 1 e a velocidade de consumo desta espécie é de 0,022 mol·L 1·min 1. Assinale a opção que
apresenta o valor da constante de velocidade da reação global, em min 1.

A ( ) 0,0022 B ( ) 0,011 C ( ) 0,022 D ( ) 0,11 E ( ) 0,22

Questão 15. Um motor pulso-jato é uma máquina térmica que pode ser representada por um ciclo
termodinâmico ideal de três etapas:

I. Aquecimento isocórico (combustão).


II. Expansão adiabática (liberação de gases).
III. Compressão isobárica (rejeição de calor a pressão atmosférica).

Considerando que essa máquina térmica opere com gases ideais, indique qual dos diagramas pressão versus
volume a seguir representa o seu ciclo termodinâmico.

A( ) B( ) C( )

D( ) E( )

5
Questão 16. Deseja-se depositar uma camada de 0,85 g de níquel metálico no catodo de uma célula eletrolítica,
mediante a passagem de uma corrente elétrica de 5 A através de uma solução aquosa de nitrato de níquel.
Assinale a opção que apresenta o tempo necessário para esta deposição, em minutos.

A ( ) 4,3 B ( ) 4,7 C ( ) 5,9 D ( ) 9,3 E ( ) 17,0

Questão 17. Considere as seguintes proposições para espécies químicas no estado gasoso:

I. A energia de ionização do íon Be3+ é maior do que a do íon He+.


II. O momento dipolar elétrico total da molécula de XeF4 é maior do que o da molécula de XeF2.
III. A energia necessária para quebrar a molécula de F2 é maior do que a energia necessária para quebrar a
molécula de O2.
IV. A energia do orbital 2s do átomo de berílio é igual à energia do orbital 2s do átomo de boro.

Das proposições acima, está(ão) CORRETA(S)

A ( ) apenas I. B ( ) apenas I e IV. C ( ) apenas II.


D ( ) apenas II e III. E ( ) apenas IV.

Questão 18. Considere as proposições a seguir:

I. A reação do ácido butanóico com a metilamina forma N-metil-butanamida.


II. A reação do ácido propanóico com 1-propanol forma propanoato de propila.
III. 3-etil-2,2-dimetil-pentano é um isômero estrutural do 2,2,3,4-tetrametil-pentano.
IV. O 2-propanol é um composto quiral.

Das proposições acima estão CORRETAS

A ( ) apenas I e II. B ( ) apenas I, II e III. C ( ) apenas II e III.


D ( ) apenas II, III e IV. E ( ) apenas III e IV.

Questão 19. Assinale a opção que indica a técnica de química analítica empregada em etilômetros (bafômetros)
que utilizam dicromato de potássio.

A ( ) Calorimetria. B ( ) Densimetria. C ( ) Fotometria.


D ( ) Gravimetria. E ( ) Volumetria.

Questão 20. São feitas as seguintes proposições a respeito dos hidrocarbonetos cuja fórmula molecular é C5H10:

I. Existem apenas seis isômeros do C5H10.


II. Pelo menos um dos isômeros do C5H10 é quiral.
III. Em condições ambiente e na ausência de luz todos os isômeros do C5H10 são capazes de descolorir água de
bromo.

Das proposições acima é (são) CORRETA(S)

A( ) apenas I.
B( ) apenas II.
C( ) apenas III.
D( ) apenas I e III.
E( ) apenas II e III.

6
AS QUESTÕES DISSERTATIVAS, NUMERADAS DE 21 A 30, DEVEM SER RESPONDIDAS NO CADERNO
DE SOLUÇÕES.
AS QUESTÕES NUMÉRICAS DEVEM SER DESENVOLVIDAS SEQUENCIALMENTE ATÉ O FINAL.

Questão 21. Gás cloro é borbulhado em uma solução aquosa concentrada de NaOH a quente, obtendo-se dois
ânions X e Y.

a) Quais são estas espécies X e Y ?


b) Com a adição de solução aquosa de nitrato de prata poder-se-ia identificar estes ânions? Justifique sua
resposta utilizando equações químicas e descrevendo as características do(s) produto(s) formado(s).
Questão 22. Ambos os íons sulfeto e sulfito reagem, em meio ácido, com o íon bromato, provocando o
aparecimento de uma coloração no meio reacional.

a) Escreva as equações químicas balanceadas que representam as reações que provocam o aparecimento de
coloração no meio reacional.
b) Escreva a equação química balanceada que representa a reação envolvendo o sulfito quando há excesso do
agente redutor. Nestas condições, explique o que ocorre com a coloração do meio reacional.

Questão 23. A reação do benzeno com cloreto de metila, catalisada por cloreto de alumínio, forma um produto
orgânico X.
a) Escreva, utilizando fórmulas estruturais, a equação química que representa a síntese de TNT
(trinitrotolueno) a partir do produto X, incluindo as condições experimentais de síntese.
b) Escreva o nome sistemático, segundo a IUPAC, do isômero mais estável do TNT.
c) Sabendo que a sensibilidade à fricção e ao impacto do TNT está relacionada à presença de diferentes
distâncias intermoleculares no sólido, em que condições a sensibilidade do TNT é minimizada?
Questão 24. Após inalar ar na superfície, uma pessoa mergulha até uma profundidade de 200 m, em apneia,
sem exalar. Desconsiderando as trocas gasosas que ocorrem nos alvéolos pulmonares, calcule a pressão parcial
do nitrogênio e do oxigênio do ar contido no pulmão do mergulhador.

Questão 25. Com base no fato de que o esmalte dentário é sujeito à desmineralização, explique

a) como se forma o ácido lático na saliva humana.


b) como o ácido lático provoca a desmineralização.
c) como a uréia contida na saliva ajuda a proteger contra a desmineralização do esmalte dentário causada pelo
ácido lático.
Questão 26. Descreva a síntese da uréia, desenvolvida por Wöhler em 1828, a partir do cianeto de prata,
oxigênio molecular e cloreto de amônio.

Questão 27. Considere que a radiação de comprimento de onda igual a 427 nm seja usada no processo de
fotossíntese para a produção de glicose. Suponha que esta radiação seja a única fonte de energia para este
processo. Considere também que o valor da variação de entalpia padrão da reação de produção de glicose, a
25 oC, seja igual a +2808 kJ·mol 1.

a) Escreva a equação que representa a reação química de produção de um mol de glicose pelo processo de
fotossíntese.
b) Calcule a variação de entalpia envolvida na produção de uma molécula de glicose, via fotossíntese, a 25 oC.
c) Calcule a energia de um fóton de radiação com comprimento de onda de 427 nm.
d) Quantos destes fótons (427 nm), no mínimo, são necessários para produzir uma molécula de glicose?

7
Questão 28. Considere as reações químicas reversíveis I e II:
2
k1
I. BrO3 (aq) 3SO3 (aq) Br (aq) 3SO42 (aq) .
k2
II. O2 (g) O (g) O3 (g) .

A respeito das reações I e II responda às solicitações dos itens a e b, respectivamente:

a) Sabendo que a reação I ocorre em meio ácido e que a sua reação direta é sujeita à lei de velocidade dada por
v k1 BrO3 SO32 H , expresse a lei de velocidade para a reação reversa.

b) Calcule a constante de equilíbrio da reação II dadas as seguintes reações e suas respectivas constantes de
equilíbrio:
h 49
NO 2 (g) NO (g) O (g) Keq. = 4,0x10
34
O3 (g) NO(g) NO2 (g) O2 (g) Keq. = 2,0x10

Questão 29. Sobre um motor pulso jato como o apresentado na Questão 15, considere verdadeiras as seguintes
afirmações:

I. A temperatura de fusão do material que compõe a câmara de combustão é 1500 K, e acima de 1200 K o
material do motor começa a sofrer desgaste considerável pelos gases de combustão;
II. O material do motor resiste a pressões de até 30 atm;
III. O motor opera, em cada ciclo termodinâmico, com 0,2 mol de uma mistura de gases com comportamento
ideal, iniciando o ciclo em pressão atmosférica e a temperatura de 300 K.

a) A partir destas informações e considerando que se deseja obter, de forma segura, o máximo de trabalho por
ciclo, quais devem ser a pressão e a temperatura no ponto de intersecção entre os processos I e II do ciclo
termodinâmico (vide Questão 15)?
b) Na mistura de gases que opera em cada ciclo há uma fração de combustível, o qual tem a reação de
combustão dada por:

1
CH 4 (g) 2O 2 (g) CO 2 (g) 2H 2O(g) QV 45 kJ g

em que QV é o calor liberado a volume constante, por grama de metano. Considerando a capacidade
calorífica molar a volume constante da mistura de gases igual a 25 J·K 1·mol 1, qual é a massa de metano
utilizada pelo ciclo projetado no item anterior?

Questão 30. Considere as substâncias o-diclorobenzeno e p-diclorobenzeno.

a) Escreva as fórmulas estruturais de ambas as substâncias.


b) Para ambas as substâncias, forneça um nome sistemático diferente daquele informado no enunciado.
c) Qual das duas substâncias tem maior ponto de ebulição? Justifique sua resposta.

8
CONSTANTES
Constante de Avogadro (NA) = 6,02 x 1023 mol−1
Constante de Faraday (F) = 9,65 x 104 C·mol−1 = 9,65 x 104 A·s·mol−1 = 9,65 x 104 J·V−1·mol−1
Volume molar de gás ideal = 22,4 L (CNTP)
Carga elementar = 1,602 x 10−19 C
Constante dos gases (R) = 8,21 x 10−2 atm·L·K−1·mol−1 = 8,31 J·K−1·mol−1 = 1,98 cal·K−1·mol−1 =
= 62,4 mmHg·L·K−1·mol−1
Constante gravitacional (g) = 9,81 m·s−2
Constante de Planck (h) = 6,626 x 10−34 m2·kg·s−1
Velocidade da luz no vácuo = 3,0 x 108 m·s−1
DEFINIÇÕES

Pressão de 1 atm = 760 mmHg = 101325 N·m−2 = 760 Torr = 1,01325 bar
1 J = 1 N·m = 1 kg·m2·s−2. ln 2 = 0,693
Condições normais de temperatura e pressão (CNTP): 0° C e 760 mmHg
Condições ambientes: 25° C e 1 atm
Condições padrão: 1 bar; concentração das soluções = 1 mol·L−1 (rigorosamente: atividade unitária das espécies); sólido com estrutura
cristalina mais estável nas condições de pressão e temperatura em questão.
(s) = sólido. (ℓ) = líquido. (g) = gás. (aq) = aquoso. (CM) = circuito metálico. (conc) = concentrado.
(ua) = unidades arbitrárias. [X] = concentração da espécie química X em mol·L−1.

MASSAS MOLARES

Elemento Número Massa Molar Elemento Número Massa Molar


Químico Atômico (g·mol−1) Químico Atômico (g·mol−1)
H 1 1,01 Cr 24 52,00
He 2 4,00 Mn 25 54,94
Li 3 6,94 Fe 26 55,85
C 6 12,01 Co 27 58,93
N 7 14,01 Cu 29 63,55
O 8 16,00 Zn 30 65,38
Na 11 22,99 Br 35 79,90
Mg 12 24,31 Ag 47 107,87
P 15 30,97 I 53 126,90
S 16 32,06 Pt 78 195,08
Cl 17 35,45 Hg 80 200,59
K 19 39,10 Pb 82 207,21
Ca 20 40,08

Questão 1. Considerando condições ambientes, assinale a opção ERRADA.

A( ) Em solução aquosa, Br− é classificado como base de Brønsted-Lowry e de Lewis.


B( ) Em solução aquosa, NH3 é classificada como base de Arrhenius, de Brønsted-Lowry e de Lewis.
C( ) Quando adicionado à água, KH(s) forma uma solução aquosa básica.
D( ) Quando LiCl(s) é adicionado à água, a solução permanece neutra.
E( ) Uma solução aquosa de CH3OH a 0,10 mol·L−1 pode ser considerada essencialmente neutra.

Questão 2. Assinale a opção que apresenta o sal solúvel em água a 25 oC.

A ( ) CaSO4 B ( ) PbCl2 C ( ) Ag2CO3


D ( ) Hg2Br2 E ( ) FeBr3

Questão 3. A constante ebulioscópica da água é 0,51 K·kg·mol−1. Dissolve-se em água 15,7 g de um composto
solúvel, não volátil e não eletrólito, cuja massa molar é de 157 g·mol−1. Assinale a alternativa que corresponde à
variação na temperatura de ebulição desta solução aquosa, em kelvin.

A ( ) 0,05 B ( ) 0,20 C ( ) 0,30 D ( ) 0,40 E ( ) 0,50


Questão 4. A respeito de reações químicas descritas pela equação de Arrhenius, são feitas as seguintes
proposições:

I. Para reações bimoleculares, o fator pré-exponencial na equação de Arrhenius é proporcional à


frequência de colisões, efetivas ou não, entre as moléculas dos reagentes.
II. O fator exponencial na equação de Arrhenius é proporcional ao número de moléculas cuja energia
cinética relativa é maior ou igual à energia de ativação da reação.
III. Multiplicando-se o negativo da constante dos gases (−R) pelo coeficiente angular da reta ln k versus 1/T
obtém-se o valor da energia de ativação da reação.
IV. O fator pré-exponencial da equação de Arrhenius é determinado pela intersecção da reta ln k versus 1/T
com o eixo das abscissas.
Das proposições acima, está(ão) ERRADA(S)

A ( ) apenas I. B ( ) apenas I e II. C ( ) apenas I e IV.


D ( ) apenas II e III. E ( ) apenas IV.

Questão 5. Considere os seguintes compostos químicos que se encontram no estado líquido à temperatura de
298 K e pressão ambiente de 1 bar:

I. 2-metil-pentano
II. 3-metil-pentano
III. 2,2-dimetil-butano
IV. 2,3-dimetil-butano
V. Hexano

Nestas condições, assinale a opção que apresenta a ordem decrescente da magnitude da pressão de vapor dos
respectivos compostos.

A ( ) I > II > III > IV > V B ( ) II > I > V > III > IV C ( ) III > IV > I > II > V
D ( ) IV > III > I > II > V E ( ) V > II > I > IV > III

Questão 6. Assinale a opção que apresenta a afirmação ERRADA.

A ( ) O número de massa, A, de um isótopo é um número inteiro positivo adimensional que corresponde à


soma do número de prótons e de nêutrons no núcleo daquele isótopo.
B ( ) Massa atômica refere-se à massa de um único átomo, e é invariante para átomos de um mesmo isótopo.
Quando medida em unidades padrão de massa atômica, ela nunca é um número inteiro exceto para o
átomo de 12C.
C ( ) A soma do número de prótons e nêutrons em qualquer amostra de matéria cuja massa é exatamente 1 g
vale exatamente 1 mol.
D ( ) A massa molar de um dado elemento químico pode variar em diferentes pontos do sistema solar.
E ( ) Multiplicando-se a unidade padrão de massa atômica pela constante de Avogadro, obtém-se exatamente
1 g·mol−1.

Questão 7. Considere a reação descrita pela seguinte equação química:

H2(g, 1bar) + 2AgBr(s) → 2H+(aq) + 2Br−(aq) + 2Ag(s).

Sendo X o potencial padrão (E°) da reação, o pH da solução a 25 oC quando o potencial da reação (E) for Y
será dado por

A ( ) (X-Y)/0,059. B ( ) (Y-X)/0,059. C ( ) (X-Y)/0,118.


D ( ) (Y-X)/0,118. E ( ) 2(X-Y)/0,059.
Questão 8. Uma amostra de 4,4 g de um gás ocupa um volume de 3,1 L a 10 oC e 566 mmHg. Assinale a
alternativa que apresenta a razão entre as massas específicas deste gás e a do hidrogênio gasoso nas mesmas
condições de pressão e temperatura.

A ( ) 2,2 B ( ) 4,4 C ( ) 10 D ( ) 22 E ( ) 44

Questão 9. No estado padrão, é de 0,240 V o potencial da pilha cuja reação pode ser descrita pela seguinte
equação química:
2 NO + ½ O2 + H2O → 2 HNO2.

Assinale a alternativa que apresenta o valor da energia livre padrão da reação, em kJ·mol−1.

A ( ) −11,6 B ( ) −23,2 C ( ) −34,8 D ( ) −46,3 E ( ) −69,5

Questão 10. Quantidades


uantidades iguais de H2(g) e I2(g) foram colocadas em um frasco, com todo o sistema à
temperatura T, resultando na pressão total de 1 bar.
bar Verificou-se
se que houve a produção de HI(g), cuja pressão
parcial foi de 22,8 kPa. Assinale a alternativa que apresenta o valor que mais se aproxima do valor CORRETO
da constante de equilíbrio desta reação.

A ( ) 0,295 B ( ) 0,350 C ( ) 0,490 D ( ) 0,590 E ( ) 0,700

Questão 11. Considere uma célula eletrolítica na forma de


um tubo em H, preenchido com solução aquosa de NaNO3
e tendo eletrodos inertes mergulhados em cada ramo
vertical do tubo e conectados a uma fonte externa. Num
determinado instante, injeta-se
se uma solução aquosa de
CuCrO4 verde na parte central do ramo horizontal do tubo.
Após algum tempo de eletrólise, observa-se
observa uma mancha
azul e uma amarela, separadas (em escala) de acordo com o
esquema da figura.

Com base nas informações do enunciado e da figura, assinale


a a opção ERRADA.

A( ) O eletrodo Ee corresponde ao anodo.


B( ) Há liberação de gás no Ed.
C( ) Há liberação de H2 no Ee.
D( ) O íon cromato tem velocidade de migração maior que o íon cobre.
E( ) O pH da solução em torno do Ed diminui.

Questão 12. Considere que 20 g de tiossulfato de potássio com pureza de 95% reagem com ácido clorídrico em
excesso, formando 3,2 g de um sólido de coloração amarela. Assinale a alternativa que melhor representa o
rendimento desta reação.

A ( ) 100% B ( ) 95% C ( ) 80% D ( ) 70% E ( ) 65%

Questão 13. Considere ass entalpias padrão de formação dos seguintes compostos:

CH4(g) O2(g) CO2(g) H2O(g)


∆Hfo / kJ·mol−1 −74,81 zero −393,51
393,51 −285,83

Sabendo que a capacidade calorífica da água, à pressão constante, vale 75,9 J·mol−1 e que sua entalpia de
vaporização é igual a 40,66 kJ·mol−1, assinale
a a alternativa que melhor corresponda ao número de mols de
metano necessários para vaporizar 1 L de água pura, cuja temperatura inicial é 25 oC, ao nível do mar.

A ( ) 1,0 B ( ) 2,0 C ( ) 2,9 D ( ) 3,8 E ( ) 4,7


abalho do zinco metálico é 5,82 x 10−19 J, assinale
Questão 14. Sabendo que a função trabalho ssinale a opção que apresenta a
energia cinética máxima, em joules, de um dos elétrons emitidos,
emitidos quando luz de comprimento de onda igual a
140 nm atinge a superfície do zinco.

A ( ) 14,2 x 10−18 B ( ) 8,4 x 10−18 C 14,2 x 10−19


D ( ) 8,4 x 10−19 E ( ) 14,2 x 10−20

Questão 15. Considerando


onsiderando um gás monoatômico ideal, assinale a opção que contém o gráfico que melhor
representa como a energia cinética média (Ec) das partículas que compõem este gás varia em função da
temperatura absoluta (T) deste gás.

A( ) B( ) C( )

D( ) E( )

Questão 16. Considere a expansão de um gás ideal inicialmente contido em um recipiente de 1 L sob pressão
de 10 atm. O processo de expansão pode ser realizado de duas maneiras diferentes,
diferentes ambas à temperatura
constante:

I. Expansão em uma etapa, contra a pressão externa constante de 1 atm, levando o volume final do
recipiente a 10 L.
II. Expansãoo em duas etapas: na primeira,
primeira o gás expande contra a pressão externa constante de 5 atm até
atingir um volume de 2 L; na segunda etapa, o gás expande contra uma pressão constante de 1 atm
atingindo o volume final de 10 L.

Com base nestas informações, assinale


ssinale a proposição CORRETA.

A( ) O trabalho realizado pelo gás é igual nos dois processos de expansão.


B( ) O trabalho realizado no primeiro processo é metade do trabalho realizado no segundo processo.
C( ) A variação da energia interna do gás é igual em ambos os processos.
D( ) A variação da energia interna do gás no
n primeiro processo é metade da do segundo processo.
processo
E( ) O calor trocado pelo gás é igual em ambos os processos.
Questão 17. Considere a seguinte reação química e a respectiva lei de velocidade experimental:

2NO(g) + O2(g) →2NO2(g) , v = k[NO]2[O2]

Para esta reação, são propostos os mecanismos reacionais I, II e III com suas etapas elementares de reação:
I. 2NO ( g ) ⇌ N 2O2 ( g ) equilíbrio rápido
N 2O2 ( g ) + O2 ( g ) → 2NO2 ( g ) lenta

II. NO ( g ) + O2 ( g ) ⇌ NO3 ( g ) equilíbrio rápido


NO ( g ) + NO3 ( g ) → 2NO2 ( g ) lenta

III. NO ( g ) + ½ O2 ( g ) → NO2 ( g ) rápida


NO2 ( g ) + ½ O2 ( g ) ⇌ NO3 ( g ) equilíbrio rápido
NO ( g ) + NO3 ( g ) ⇌ N 2O4 ( g ) equilíbrio rápido
N 2O4 ( g ) → 2NO2 ( g ) lenta

Dos mecanismos propostos, são consistentes com a lei de velocidade observada experimentalmente

A ( ) apenas I. B ( ) apenas I e II. C ( ) apenas II.


D ( ) apenas II e III. E ( ) apenas III.

Questão 18. Uma reação hipotética de decomposição de uma substância gasosa catalisada em superfície
metálica tem lei de velocidade de ordem zero, com uma constante de velocidade (k) igual a 10−3 atm·s−1.
Sabendo que a pressão inicial do reagente é igual a 0,6 atm, assinale a opção que apresenta o tempo necessário,
em segundos, para que um terço do reagente se decomponha.

A ( ) 0,00001 B ( ) 200 C ( ) 400 D ( ) 600 E ( ) 693

Questão 19. Duas placas de platina são conectadas a um potenciostato e imersas em um béquer contendo uma
solução aquosa de sulfato de cobre. Entre estas duas placas ocorre a passagem de corrente elétrica. Após certo
tempo foi verificado que a cor azul, inicialmente presente na solução, desapareceu e que houve a liberação de
um gás em uma das placas de platina. A solução, agora totalmente incolor, contém

A ( ) hidróxido de cobre. B ( ) sulfato de platina. C ( ) hidróxido de platina.


D ( ) ácido sulfúrico. E ( ) apenas água.

Questão 20. A energia do estado fundamental do átomo de hidrogênio é −13,6 eV. Considerando todas as
espécies químicas no estado gasoso e em seu estado eletrônico fundamental, é CORRETO afirmar que o valor
absoluto

A ( ) da energia do orbital 1s do átomo de hélio é menor que 13,6 eV.


B ( ) da energia da molécula de H2, no seu estado de mínima energia, é menor do que o valor absoluto da
soma das energias de dois átomos de hidrogênio infinitamente separados.
C ( ) da afinidade eletrônica do átomo de hidrogênio é igual a 13,6 eV.
D ( ) da soma das energias de dois átomos de deutério, infinitamente separados, é maior do que o valor
absoluto da soma das energias de dois átomos de hidrogênio infinitamente separados.
E ( ) da energia do íon He+ é igual ao valor absoluto da soma das energias de dois átomos de hidrogênio
infinitamente separados.
AS QUESTÕES DISSERTATIVAS, NUMERADAS DE 21 A 30, DEVEM SER RESPONDIDAS NO CADERNO
DE SOLUÇÕES.
AS QUESTÕES NUMÉRICAS DEVEM SER DESENVOLVIDAS SEQUENCIALMENTE ATÉ O FINAL.

Questão 21. Considere a rota de síntese do 1-bromo-2-metil-2-propanol mostrada abaixo:

CH3
H2SO4 Br2
X 50 oC
Y H2O
H3 C C CH2 Br
OH

a) Escreva a fórmula estrutural do composto orgânico X.


b) Escreva a fórmula estrutural do composto orgânico Y.
c) Escreva a fórmula estrutural do composto orgânico que seria formado se, ao invés de água, o solvente
utilizado na última reação química fosse o metanol.

Questão 22. Reações de Grignard são geralmente realizadas utilizando éter dietílico anidro como solvente.
a) Escreva a fórmula estrutural do reagente de Grignard cuja reação com gás carbônico e posterior hidrólise
produz ácido di-metil-propanóico.
b) Por que o solvente utilizado em reações de Grignard deve ser anidro? Escreva uma equação química para
justificar sua resposta.

Questão 23. Sabendo que o produto de solubilidade do calomelano (cloreto de mercúrio I) é Kps = 2,6 x 10−18 e
que seu logaritmo natural é ln(Kps) = −40,5, determine:

a) a concentração, em mol·L−1, de Hg22+ e de Cl− numa solução aquosa saturada de calomelano.


b) o potencial padrão de um eletrodo de calomelano.

Questão 24. Dadas as informações:

I. O poder calorífico de um combustível representa a quantidade de calor gerada na combustão por


unidade de massa.
II. O poder calorífico do H2(g) é aproximadamente 3 vezes o da gasolina.
III. O calor latente de ebulição do H2(ℓ) é desprezível frente ao poder calorífico do H2(g).
IV. A massa específica do H2(ℓ) é de 0,071 g·cm−3 e a da gasolina é de 0,740 g·cm−3.

Com base nestas informações, determine o valor numérico:

a) da massa de 45 L de gasolina.
b) do volume de H2(ℓ) que, ao sofrer combustão, fornece a mesma quantidade de calor liberada na combustão
de 45 L de gasolina.
c) do volume que o H2 ocuparia se estivesse na forma de gás, à pressão de 1 bar e a 25 oC.

Questão 25. Dado o seguinte mecanismo reacional, constituído de duas etapas elementares (I e II).

k k2
I. A↽ k 1 ⇀M II. M + A  →C
−1

Escreva a expressão para a taxa de variação temporal da concentração do:


a) reagente A.
b) intermediário M.
c) produto C.
Questão 26. No diagrama de fases da água pura, o ponto triplo ocorre à temperatura absoluta de 273,16 K e à
pressão de 0,006037 atm. A temperatura de ebulição da água à pressão de 1 atm é 373,15 K. A temperatura
crítica da água pura é de 647,096 K e sua pressão crítica é de 217,7 atm.
a) Esboce o diagrama de fases da água pura e indique neste diagrama o ponto triplo, o ponto de ebulição a
1 atm e o ponto crítico. No mesmo diagrama, usando linhas tracejadas, desenhe as curvas de equilíbrio
sólido-líquido e líquido-gás quando se dissolve na água pura um soluto não volátil e não solúvel na fase
sólida.
b) Esboce o diagrama de fases de uma substância que sublime à pressão ambiente, cuja temperatura crítica seja
216,6 K e cuja fase sólida seja mais densa do que a fase líquida.

Questão 27. A saliva humana pode ser considerada uma solução tampão. Cite quais espécies químicas
inorgânicas compõem este tampão e explique como elas atuam.

Questão 28. A toda reação química corresponde uma variação de energia interna, ∆U, e uma variação de
entalpia, ∆H. Explique em que condições ∆U tem valor igual ao de ∆H.

Questão 29. Uma amostra de 50 g de iodeto de potássio, com pureza de 83%, reage com ácido sulfúrico e
dióxido de manganês. O iodo liberado nesta reação reage com fósforo vermelho e o composto resultante sofre
hidrólise. Sabendo que o rendimento da primeira reação é de 80%:
a) calcule a massa de iodo produzida na primeira reação química.
b) escreva a equação química balanceada para a primeira reação química.
c) escreva a equação química balanceada para a segunda reação química.
d) escreva a equação química balanceada para a terceira reação química.

Questão 30. O ácido hipocloroso sofre, em solução aquosa, três diferentes processos de transformação que
ocorrem de forma independente. Escreva as equações balanceadas que representam as reações químicas que
ocorrem nas seguintes condições:

a) sob a ação da luz solar direta ou em presença de sais de cobalto como catalisador.
b) reação ocorrendo na presença de CaCl2 como substância desidratante.
c) sob ação de calor.
&2167$17(6

&RQVWDQWHGH$YRJDGUR 1$   [PPRO
&RQVWDQWHGH)DUDGD\ )   [ &PRO [$VPRO [-9P

PRO

9ROXPHPRODUGHJiVLGHDO  / &173 3 
&DUJDHOHPHQWDU  [&
&RQVWDQWHGRVJDVHV 5   [DWWP/.PRO -.PRO FDO.PRO 
  PP+J/ /.PRO
&RQVWDQWHJUDYLWDFLRQDO J   PV
&RQVWDQWHGH3ODQFN K   [PNJV
9HORFLGDGHGDOX]QRYiFXR  [PV

1~PHURGH(XOHU H   


'(),1,d®(6

3UHVVmRDWP PP+J [1P 7RUU EDU


(QHUJLD- 1P NJPV  
&RQGLo}HVQRUPDLVGHWHPSHUDWXUDHSUHVVmR & &173 ƒ&HPP+J
&RQGLo}HVDPELHQWHVƒ&HDWP
&RQGLo}HVSDGUmREDUFRQFHQWUDomRGDVVROXXo}HV PRO/ ULJRURVDPHQWHDWLYLGDGHXQLWiULDGGDVHVSpFLHV VyOLGRFRPHVWUXWXUD
FULVWDOLQDPDLVHVWiYHOQDVFRQGLo}HVGHSUHVVmRRHWHPSHUDWXUDHPTXHVWmR
V  VyOLGR Ɛ  OtTXLGR J  JiV DT  DTXRVR &0  FLUFXLWRPHWiOLFR FRQF  FRQFHQWWUDGR
XD  XQLGDGHVDUELWUiULDV>;@ FRQFHQWUDomRRGDHVSpFLHTXtPLFD;HPPRO/


0$66$602/$5(6

(OHPHQWR 1~PHUR (OHPHQWR 1~PHUR


0RODU JPRO 
0DVVD0 0DVVD0RODU JPRO 
4XtPLFR $W{PLFR 4XtPLFR $W{PLFR
+   )H  
+H   &X  
&   =Q  
1   %U  
2   3W  
1D  
 3E  
6  
 5D  QmRSRVVXLLVyWRSRVHVWiYHLV 
&O  
 8  
&D  
   



4XHVWmR$PLQRiFLGRVVmRFRPSRVWRRVRUJkQLFRVTXHFRQWrPXPJUXSRDPLQDHXXPJUXSRFDUER[tOLFR1RVĮ
DPLQRiFLGRVRVGRLVJUXSRVHQFRQWUDPPVHQDVH[WUHPLGDGHVGDPROpFXODHHQWUHHOHVKiXPiWRPRGHFDUERQR
GHQRPLQDGRFDUERQRĮTXHWDPEpPHVVWiOLJDGRDXPJUXSR5FRQIRUPHDILJXUD
&RQVLGHUHRVVHJXLQWHVDPLQRiFLGRV
, $ODQLQDHPTXH5 &+
,, $VSDUDJLQDHPTXH5 &+&221+
,,,)HQLODODQLQDHPTXH5 &+&+
,9 *OLFLQDHPTXH5 +
9 6HULQDHPTXH5 &+2+
$VVLQDOHDRSomRTXHFRQWpPR V DPLQQRiFLGR V TXHSRVVXL HP JUXSR V 5SRODU HVV 
$   $ODQLQDH)HQLODODQLQD   %   $VSDUDJLQDH*OLFLQD
&   $VSDUDJLQDH6HULQD    '   )HQLODODQLQD
(   *OLFLQD)HQLODODQLQDH6HULQD



4XHVWmR  &RQVLGHUH DV VHJXLQWHV SURSRVLo}HV D UHVSHLWR GRV YDORUHV HP PyGXOR GD HQHUJLD GH RUELWDLV
DW{PLFRVVHS
, ( V ( S SDUDRiWRPRGHKLGURJrQLR
,, ( V ( S SDUDRtRQGHKpOLRFDUUHJDGRFRPXPDFDUJDSRVLWLYD
,,, ( V ! ( S SDUDRiWRPRGHKpOLR

'DVSURSRVLo}HVDFLPDHVWi mR &255(7$ 6 
$   DSHQDV,    %   DSHQDV,,   &   DSHQDV,,,
'   DSHQDV,H,,,    (   WRGDV

4XHVWmR(QWUHDVVXEVWkQFLDV&+&+&O&+%U&+&O&+%UH&%U
$   &%UpDGHPDLRUSRQWRGHHEXOLomR
%   &+%UpPDLVYROiWLOTXHR&+&O
&   &+%UWHPPDLRUSUHVVmRGHYDSRUTXHR&+&O
'   &+pDGHPDLRUIRUoDGHLQWHUDomRLQWHUPROHFXODU
(   TXDWURGHVWDVPROpFXODVVmRDSRODUHV

4XHVWmR&RQVLGHUHDVSURSRVLo}HVDVHJXLU
, 2DOFHQR&+DSUHVHQWDFLQFRLV{PHURV
,, ([LVWHPWUrVGLIHUHQWHVFRPSRVWRVFRPDIyUPXOD&+&O
,,, ([LVWHPTXDWURGLIHUHQWHVpWHUHVFRPDIyUPXODPROHFXODU&+2
,9 2WULPHWLOEHQ]HQRWHPWUrVLV{PHURVHVWUXWXUDLV
'DVSURSRVLo}HVDFLPDHVWmR&255(7$6
$   DSHQDV,,,H,9   %   DSHQDV,H,,,   &   DSHQDV,,,,,H,9
'   DSHQDV,,H,9   (   WRGDV

4XHVWmR8PUHFLSLHQWHGH/GHFDSDFLGDGHFRQWpPXPDPLVWXUDGRVJDVHVLGHDLVKLGURJrQLRHGLy[LGR
GH FDUERQR D ƒ& 6DEHQGR TXH DSUHVVmRSDUFLDO GR GLy[LGR GH FDUERQR p WUrV YH]HV PHQRU TXH DSUHVVmR
SDUFLDOGRKLGURJrQLRHTXHDSUHVVmRWRWDOGDPLVWXUDJDVRVDpGHDWPDVVLQDOHDDOWHUQDWLYDTXHDSUHVHQWD
UHVSHFWLYDPHQWHDVPDVVDVGHKLGURJrQLRHGHGLy[LGRGHFDUERQRFRQWLGDVQRUHFLSLHQWH
$   JHJ    %   JHJ   &   JHJ
'   JHJ    (   JHJ

4XHVWmR  'HVHMDVH DTXHFHU J GH iJXD SXUD GD WHPSHUDWXUD DPELHQWH DWp ƒ& HP SUHVVmR DPELHQWH
8WLOL]DQGR XP IRUQR GH PLFURRQGDV FRQYHQFLRQDO TXH HPLWH UDGLDomR HOHWURPDJQpWLFD FRP IUHTXrQFLD GH
*+] H FRQVLGHUDQGR D FDSDFLGDGH FDORUtILFD GD iJXD FRQVWDQWH H LJXDO D -Jƒ& DVVLQDOH D
DOWHUQDWLYDTXHDSUHVHQWDRQ~PHURDSUR[LPDGRGHIyWRQVQHFHVViULRSDUDUHDOL]DUHVWHDTXHFLPHQWR
$   î    %   î   &   î
'   î    (   î

4XHVWmR&RQVLGHUHXPUHFLSLHQWHGH/DRTXDOVmRDGLFLRQDGRVJDVHVLGHDLVQDVVHJXLQWHVFRQGLo}HV
, +pOLRFPDFP+JHƒ&
,, 0RQy[LGRGHFDUERQR/DPP+JHƒ&
,,, 0RQy[LGRGHQLWURJrQLRPDDWPHƒ&
6DEHQGRTXHDSUHVVmRWRWDOGDPLVWXUDJDVRVDpGHDWPDVVLQDOHDRSomRTXHDSUHVHQWDDSUHVVmRSDUFLDOGR
KpOLRQDPLVWXUDJDVRVD
$   DWP    %   DWP   &   DWP
'   DWP    (   DWP


4XHVWmR'HQWUHRVSURFHVVRVTXtPLFRVDEDL[RDVVLQDOHDTXHOHTXHRFRUUHHPXPD~QLFDHWDSDHOHPHQWDU
$   (OHWUyOLVHGRPHWDQRO
%   'HFRPSRVLomRGRSHUy[LGRGHKLGURJrQLR
&   )RWRGHFRPSRVLomRGRR]{QLR
'   3URGXomRGHiJXDDSDUWLUGH+ J H2 J 
(   3URGXomRGHFORUHWRGHVyGLRDSDUWLUGH1D V H&O J 

4XHVWmR&RQVLGHUHDVVHJXLQWHVSURSRVLo}HV
, 0DVVDFUtWLFDUHSUHVHQWDDPDVVDPtQLPDGHXPQXFOtGHRItVVLO HPXPGHWHUPLQDGRYROXPHQHFHVViULD
SDUDPDQWHUXPDUHDomRHPFDGHLD
,, 5HDo}HV QXFOHDUHV HP FDGHLD UHIHUHPVH D SURFHVVRV QRV TXDLV HOpWURQV OLEHUDGRV QD ILVVmR SURGX]HP
QRYDILVVmRHPQRPtQLPRXPRXWURQ~FOHR
,,, 2VQ~FOHRVGH 5DSRGHPVRIUHUGHFDLPHQWRVUDGLRDWLYRVFRQVHFXWLYRVDWpDWLQJLUHPDPDVVDGH
FKXPER DGTXLULQGRHVWDELOLGDGH
'DVSURSRVLo}HVDFLPDHVWi mR &255(7$ 6 
$   DSHQDV,    %   DSHQDV,,   &   DSHQDV,,,
'   DSHQDV,H,,    (   DSHQDV,H,,,

4XHVWmR6mRIHLWDVDVVHJXLQWHVSURSRVLo}HVDUHVSHLWRGDSURGXomRGHELRFRPEXVWtYHLV
, $KLGUyOLVHiFLGDGHWULDFLOJOLFHUyLVpDHWDSDILQDOQDSURGXomRGRELRGLHVHO
,, (WDQROpFRPXPHQWHSURGX]LGRSRUSURFHVVRGHIHUPHQWDomRRTXDOJHUD&2FRPRVXESURGXWR
,,,1D VtQWHVH GR ELRTXHURVHQH SRGHP VHU XWLOL]DGRV iFLGRV JUD[RV FRP FDGHLDV OLQHDUHV RX FtFOLFDV
VDWXUDGDVRXLQVDWXUDGDV
'DVSURSRVLo}HVDFLPDHVWi mR &255(7$ 6 
$   DSHQDV,    %   DSHQDV,,   &   $SHQDV,,,
'   DSHQDV,H,,    (   DSHQDV,,H,,,

4XHVWmR&RQVLGHUHDVVHJXLQWHVSURSRVLo}HV
, $SURSULHGDGHEiVLFDDVVRFLDGDDRIUDFLRQDPHQWRGRSHWUyOHRpRSRQWRGHHEXOLomR
,, (PJHUDOQRFUDTXHDPHQWRWpUPLFRGRSHWUyOHRRFRUUHIRUPDomRGHUDGLFDLVOLYUHVSRUPHLRGDTXHEUD
GHOLJDomRKRPROtWLFDHQTXDQWRTXHQRFUDTXHDPHQWRFDWDOtWLFRRFRUUHDUXSWXUDKHWHUROtWLFD
,,, 0HWDQRQmRpSURGX]LGRQDGHVWLODomRIUDFLRQDGDGRSHWUyOHR
,9 ,QG~VWULD SHWURTXtPLFD p R WHUPR XWLOL]DGR SDUD GHVLJQDU R UDPR GD LQG~VWULD TXtPLFD TXH XWLOL]D
GHULYDGRVGHSHWUyOHRFRPRPDWpULDSULPDSDUDDIDEULFDomRGHQRYRVPDWHULDLVFRPRPHGLFDPHQWRV
IHUWLOL]DQWHVHH[SORVLYRV
9 2VUHQGLPHQWRVGHGHULYDGRVGLUHWRVGRSHWUyOHRQRSURFHVVRGHGHVWLODomRIUDFLRQDGDQmRGHSHQGHPGR
WLSRGHSHWUyOHRXWLOL]DGR
'DVSURSRVLo}HVDFLPDVmR&255(7$6
$   DSHQDV,,,H,9   %   DSHQDV,,,,,9H9  &   DSHQDV,,,,H9
'   DSHQDV,,,9H9   (   WRGDV

4XHVWmR  2 FRPSRVWR GLPHWLOSHQWHQR UHDJH FRP iJXD HP PHLR iFLGR H QD DXVrQFLD GH SHUy[LGRV
IRUPDQGRXPFRPSRVWR;TXHDVHJXLUpR[LGDGRSDUDIRUPDUXPFRPSRVWR<2VFRPSRVWRV;H<IRUPDGRV
SUHIHUHQFLDOPHQWHVmRUHVSHFWLYDPHQWH
$   XPiOFRROHXPpVWHU    %   XPiOFRROHXPDFHWRQD
&   XPDOGHtGRHXPiFLGRFDUER[tOLFR   '   XPDFHWRQDHXPDOGHtGR
(   XPDFHWRQDHXPpVWHU



4XHVWmR8PUHFLSLHQWHGHSDUHGHVDGLDEiWLFDVHGHYROXPHFRQVWDQWHFRQWpPGXDVDPRVWUDVGHiJXDSXUD
VHSDUDGDVSRUXPDSDUHGHWDPEpPDGLDEiWLFDHGHYROXPHGHVSUH]tYHO8PDGDVDPRVWUDVFRQVLVWHHPJGH
iJXDDƒ&HDRXWUDHPJDƒ&&RQVLGHUHTXHDSDUHGHTXHVHSDUDDVDPRVWUDVpUHWLUDGDHTXHDV
DPRVWUDVGHiJXDVHPLVWXUDPDWpDWLQJLURHTXLOtEULR6REUHHVVHSURFHVVRVmRIHLWDVDVVHJXLQWHVDILUPDo}HV
, $WHPSHUDWXUDGDPLVWXUDQRHTXLOtEULRpGH.
,, $YDULDomRGHHQWDOSLDQRSURFHVVRpQXOD
,,, $YDULDomRGHHQHUJLDLQWHUQDQRSURFHVVRpQXOD
,9 $YDULDomRGHHQWURSLDQRSURFHVVRpQXOD
$VVLQDOHDRSomRTXHDSUHVHQWDD V DILUPDomR }HV &255(7$ 6 VREUHDPLVWXUDGDVDPRVWUDVGHiJXD
$   $SHQDV,    %   $SHQDV,H,,   &  $SHQDV,,H,,,
'   $SHQDV,,,H,9   (   $SHQDV,9

4XHVWmR6mRIHLWDVDVVHJXLQWHVSURSRVLo}HVDUHVSHLWRGHSURSULHGDGHVFROLJDWLYDV
, $SUHVVmRRVPyWLFDGHSHQGHGRWLSRGHVROYHQWHSDUDXPGDGRVROXWR
,, $ FULRPHWULD XVD R DEDL[DPHQWR GR SRQWR GH FRQJHODPHQWR GR VROYHQWH SDUD PHGLU D PDVVD PRODU GR
VROXWR
,,,1D HEXOLRPHWULD D YDULDomR GD WHPSHUDWXUD GH HEXOLomR GHSHQGH GD FRQFHQWUDomR PRODO GH VROXWR QmR
YROiWLOXWLOL]DGR
,9 1D WRQRPHWULD RFRUUH DEDL[DPHQWR GD SUHVVmR GH YDSRU GH XPD VROXomR TXH FRQWpP XP VROXWR QmR
YROiWLOHPUHODomRDRVROYHQWHSXUR
'DVSURSRVLo}HVDFLPDp VmR &255(7$ 6 
$   DSHQDV,    %   DSHQDV,H,,,   &  DSHQDV,,,,,H,9
'   DSHQDV,,H,9   (   WRGDV

4XHVWmR(PWHPSHUDWXUDDPELHQWHDGLFLRQRXVHXPDSRUomRGHiFLGRFORUtGULFRPRO/DXPDVROXomR
DTXRVDFRQWHQGRRVtRQVPHWiOLFRV&R&X+JH3E$VVLQDOHDRSomRTXHDSUHVHQWDRVtRQVPHWiOLFRV
TXHQmRIRUDPSUHFLSLWDGRV
$   &RH&X    %   &RH+J   &   &XH+J
'   &XH3E    (   +JH3E

4XHVWmR&RQVLGHUHGDGDVDVFRQVWDQWHVGHGLVVRFLDomRiFLGD .D RXEiVLFD .E GDVVHJXLQWHVVXEVWkQFLDV


Dƒ&IHQRO &+2+ .D îHDQLOLQD &+1+ .E î
6REUHRS+GHVROXo}HVDTXRVDVGHVVDVVXEVWkQFLDVVmRIHLWDVDVVHJXLQWHVDILUPDo}HV
, $VROXomRDTXRVDGHIHQRODîPRO/WHPS+
,, $VROXomRDTXRVDGHDQLOLQDDîPRO/WHPS+!
,,, $PEDVDVVROXo}HVDTXRVDVDîPRO/WrPS+DSUR[LPDGDPHQWHLJXDLV
'DVDILUPDo}HVDFLPDHVWi mR &255(7$ 6 
$   DSHQDV,    %   DSHQDV,H,,   &  DSHQDV,,
'   DSHQDV,,H,,,   (   DSHQDV,,,

4XHVWmR6REUHLQGLFDGRUHVGHS+p(55$'2DILUPDUTXH
$   VmRiFLGRVRXEDVHVIUDFDV
%   HPVROXomRDTXRVDVmRXVDGRVFRPRWDPSmR
&   JHUDOPHQWHSRVVXHPDQpLVDURPiWLFRVHPVXDHVWUXWXUDPROHFXODU
'   GHYHPDSUHVHQWDUPtQLPDLQWHUIHUrQFLDQRVLVWHPDTXtPLFRGHLQWHUHVVH
(   UHVSRQGHP j SUHVHQoD GH tRQV KLGURJrQLR HP VROXomR DTXRVD SRU GHVORFDPHQWR GH HTXLOtEULR HQWUH DV
IRUPDVDVVRFLDGDHLRQL]DGD



4XHVWmR&RQVLGHUHDVVHJXLQWHVVHPLUUHDo}HVGHR[LUUHGXomRHVHXVUHVSHFWLYRVSRWHQFLDLVSDGUmRQDHVFDOD
GRHOHWURGRSDGUmRGHKLGURJrQLR (3+ 
, &2  +   H U & +2+  + 2 (q,  9
  q

,, 2  +  H U + 2 ( ,,  9
$VVLQDOHDRSomRTXHDSUHVHQWDDDILUPDomR(55$'$VREUHXPDFpOXODHOHWURTXtPLFDHPTXHDVHPLUUHDomR,
RFRUUHQRDQRGRHDVHPLUUHDomR,,QRFDWRGR
$   $UHDomRJOREDOpH[RWpUPLFD
%   7UDWDVHGHXPDFpOXODDFRPEXVWtYHO
&   2SRWHQFLDOSDGUmRGDFpOXODpGH9
'   2WUDEDOKRPi[LPRTXHSRGHVHUREWLGRpHPPyGXORGHN-SRUPROGHHWDQRO
(   $FpOXODFRQYHUWHHQHUJLDOLYUHGDUHDomRGHFRPEXVWmRGRHWDQROHPWUDEDOKRHOpWULFR

4XHVWmR2SHUFORUDWRGHDP{QLR 3$ pXPGRVFRPSRQHQWHVPDLVXWLOL]DGRVHPSURSHOHQWHVGHIRJXHWHV
3DUDDSHUIHLoRDUVHXGHVHPSHQKRKLGURJrQLRSRGHVHUXWLOL]DGRFRPRDGLWLYR&RQVLGHUHGDGDVDVHQWDOSLDVGH
FRPEXVWmRGHVWDVHVSpFLHV '+ F3$  N- PRO  '+ F+   N- PRO  
&RP EDVH QHVVDV LQIRUPDo}HV DVVLQDOH D RSomR TXH DSUHVHQWD D HTXDomR OLQHDU GD YDULDomR GD HQWDOSLD GH
FRPEXVWmRGDPLVWXUDGH3$FRP+HPIXQomRGDTXDQWLGDGHGH+
$   \  [       %   \  [   
&   \  [       '   \ [   
(   \ [   

4XHVWmR8PGDGRPDWHULDOVyOLGRHPHTXLOtEULRWpUPLFRHPLWHUDGLDomRVHPHOKDQWHDGHXPFRUSRQHJUR
$VVLQDOHDRSomRTXHDSUHVHQWDDFXUYDTXHH[SUHVVDDUHODomRH[SHULPHQWDO&255(7$HQWUHRFRPSULPHQWR
GHRQGDGRPi[LPRGHHPLVVmR OPi[ HDWHPSHUDWXUDGHVVHPDWHULDO

$       %       &       


OPi[

OPi[

OPi[





 7HPSHUDWXUD 7HPSHUDWXUD 7HPSHUDWXUD


'       (  


OPi[
OPi[





 7HPSHUDWXUD 7HPSHUDWXUD

 



$648(67®(6',66(57$7,9$6180(5$'$6'($'(9(06(55(6321','$612&$'(512
'(62/8d®(6
$648(67®(6180e5,&$6'(9(06(5'(6(192/9,'$66(48(1&,$/0(17($7e2),1$/

4XHVWmR8PDPLVWXUDGH&X62DQLGURH)H&OFRPPDVVDGHJpGLVVROYLGDHPiJXDHWUDWDGDFRP
XPD VROXomR GH 1D2+ HP H[FHVVR 2 SUHFLSLWDGR IRUPDGR FRQVLGHUH UHQGLPHQWR GH   p VHSDUDGR SRU
ILOWUDomRHDVHJXLUpWUDWDGRFRPiFLGRQtWULFRDJ/6mRQHFHVViULRVFPGHVVHiFLGRSDUDGLVVROYHU
WRGRRSUHFLSLWDGR
D  (VFUHYDD V HTXDomR }HV TXtPLFD V EDODQFHDGD V TXHUHSUHVHQWD P DVUHDo}HVHQYROYLGDVQRWUDWDPHQWR
FRP1D2+
E  (VFUHYD D V  HTXDomR }HV  TXtPLFD V  EDODQFHDGD V  TXH UHSUHVHQWD P  D GLVVROXomR GR SUHFLSLWDGR FRP
iFLGRQtWULFR
F  'HWHUPLQHDVPDVVDVHPJGH&X62DQLGURHGH)H&OSUHVHQWHVQDPLVWXUD

4XHVWmR&RQVLGHUHDVVHJXLQWHVUHDo}HVTXtPLFDV
&D&2 
'
o $  %
1D&O  1+   +  2  % o &  '
& 
'
o 1D &2  + 2  % 
$  + 2 o (
(  ' o &D&O   + 2  1+ 
(VFUHYDDVIyUPXODVTXtPLFDVGDVHVSpFLHV$%&'H(HQYROYLGDVQDVUHDo}HVDFLPD

4XHVWmR(PXPH[SHULPHQWRWLWXODUDPVHP/GHXPDVROXomRDTXRVDGHFDUERQDWRGHVyGLRFRPiFLGR
FORUtGULFRDPERVFRPFRQFHQWUDomRLJXDODPRO/5HJLVWURXVHDYDULDomRGRS+GDVROXomRDWpDDGLomR
GHXPYROXPHGHP/GHiFLGR
D  (VERFHDFXUYDGHWLWXODomR S+YHUVXVYROXPH 
E  ([SOLTXHRFRPSRUWDPHQWRGDFXUYDGHWLWXODomRXVDQGRHTXDo}HVTXtPLFDV
F  (VFUHYDDHTXDomRJOREDOEDODQFHDGD

4XHVWmR2VHJXLQWHVLVWHPDHOHWURTXtPLFRpFRQVWUXtGR
, 6HPLFpOXOD$FRQVWLWXtGDGHSODFDGHFKXPERSDUFLDOPHQWHLPHUVDHPXPDVROXomRDTXRVDGH3E
,, 6HPLFpOXOD%FRQVWLWXtGDGHSODFDGHSODWLQDSDUFLDOPHQWHLPHUVDHPXPDVROXomRDTXRVD;
,,,$VVROXo}HVDTXRVDVGDVVHPLFpOXODV$H%VmRFRQHFWDGDVSRUPHLRGHXPDSRQWHVDOLQD
,9 $VSODFDVPHWiOLFDVGDVVHPLFpOXODV$H%VmRFRQHFWDGDVSRUPHLRGHILRVFRQGXWRUHV
&RQVLGHUDQGRFRQGLo}HVSDGUmRHVDEHQGRTXHRSRWHQFLDOSDGUmRGDVHPLFpOXOD$FRQWUDRHOHWURGRSDGUmRGH
q
KLGURJrQLRQDWHPSHUDWXUDGHƒ&p (3E 3E  9 SHGHPVH
D  'HVHQKHHVTXHPDWLFDPHQWHDFpOXODHOHWURTXtPLFDFRQVWUXtGD
E  &RQVLGHUDQGRTXHDVROXomR;pXPDVROXomRDTXRVDGH+&OHVFUHYDDVHPLUUHDomRDQyGLFDDVHPLUUHDomR
FDWyGLFDHDUHDomRJOREDOTXHRFRUUHQHVVDFpOXOD
F  &RQVLGHUDQGR DJRUD TXH D VROXomR ; p XPD VROXomR DTXRVD GH )H H )H H TXH D SODFD GH FKXPER p
FRQHFWDGD DR WHUPLQDO QHJDWLYR GH XPD EDWHULD H D SODFD GH SODWLQD DR WHUPLQDO SRVLWLYR HVFUHYD D
VHPLUUHDomRDQyGLFDDVHPLUUHDomRFDWyGLFDHDUHDomRJOREDOTXHRFRUUHQHVVDFpOXOD

4XHVWmR  (VFUHYD DV HTXDo}HV TXtPLFDV TXH UHSUHVHQWDP DV UHDo}HV GHSROLPHUL]DomR RX FRSROLPHUL]DomR
GRVPRQ{PHURVDEDL[RDSUHVHQWDQGRDVIyUPXODVHVWUXWXUDLVGHUHDJHQWHVHSURGXWRV
D  (WHQR
E  SURSHQRQLWULOD
F  PHWLOSURSHQRDWRGHPHWLOD
G  (WHQLOEHQ]HQR YLQLOEHQ]HQR 
H  EXWDGLHQRFRPHWHQLOEHQ]HQR YLQLOEHQ]HQR 



4XHVWmR8PDGDGDUHDomR , FXMRFDORUOLEHUDGRpGHVFRQKHFLGRpFRQGX]LGDHPXPUHDWRUTXHXWLOL]DXP
JiV PDQWLGR D YROXPH FRQVWDQWH 9  FRPR EDQKR WpUPLFR 2XWUDV GXDV UHDo}HV ,, H ,,,  FRQGX]LGDV HP
FRQGLo}HVVLPLODUHVDSUHVHQWDPFDORUOLEHUDGRDYROXPHFRQVWDQWH 49 FRQIRUPHDSUHVHQWDGRQDWDEHODDEDL[R
&RQVLGHUH DV VHJXLQWHV LQIRUPDo}HV VREUH R JiV GR EDQKR 5HDomR (TXDomR 49 N-PRO 
WpUPLFRTXHWHPFRPSRUWDPHQWRQmRLGHDOHREHGHFHjHTXDomR , $   % o '  "
§ Q D·

,, $%o & 
¨ 3   ¸ 9  QE Q57  ,,, ' % o & 
© 9 ¹ 

HP TXH D /DWPPRO   E /PRO   Q PRO   9 /   FDSDFLGDGH FDORUtILFD PRODU D
YROXPHFRQVWDQWH &9P  -.PROWHPSHUDWXUDLQLFLDO 7L  .
D  6DEHQGRTXHPROGH$VmRXWLOL]DGRVQDUHDomR,FDOFXOHR49OLEHUDGRQHVVDUHDomR
E  'HWHUPLQHDWHPSHUDWXUDILQDOGREDQKRWpUPLFR
F  'HWHUPLQHDSUHVVmRLQLFLDOHDSUHVVmRILQDOGREDQKRWpUPLFR

4XHVWmR  3DUD FDGD XPD GDV GLVSHUV}HV FRORLGDLV GH QDWXUH]D GHILQLGD QD WDEHOD DEDL[R FLWH XP H[HPSOR
SUiWLFRH[SOLFLWDQGRTXDLVVmRRGLVSHUJHQWHHRGLVSHUVR&RSLHHFRPSOHWHDWDEHODQRFDGHUQRGHUHVSRVWDV
'LVSHUVmRFRORLGDO 1DWXUH]D ([HPSOR 'LVSHUJHQWH 'LVSHUVR
(VSXPDVyOLGD 3ROtPHUR   
(VSXPDOtTXLGD 3URGXWRDOLPHQWtFLR   
$HURVVROOtTXLGR )HQ{PHQRQDWXUDO   
$HURVVROVyOLGR )HQ{PHQRDUWLILFLDO   

4XHVWmR&RQVLGHUHDUHDomRJHQpULFDHTXLPRODU;<U=VHQGRTXH
, DVFRQFHQWUDo}HVLQLFLDLVGH;HGH<VmRLJXDLV
,, DUHDomRGLUHWDDSUHVHQWDOHLGHYHORFLGDGHGHRUGHP
,,,DHQHUJLDGHDWLYDomRGDUHDomRLQYHUVDpN-PROD.
&RQVLGHUH GDGRV R IDWRU SUpH[SRQHQFLDO GD UHDomR LQYHUVD $ î /PRO V H D FRQVWDQWH GH
HTXLOtEULRGDUHDomRGLUHWD. 
&RP EDVH QHVVDV LQIRUPDo}HV GHWHUPLQH R YDORU QXPpULFR GD YHORFLGDGH GD UHDomR GLUHWD TXDQGR D
FRQFHQWUDomRGH=IRUPRO/RTXHFRUUHVSRQGHDGHUHQGLPHQWRGDUHDomR

4XHVWmR&RQVLGHUHRVH[SHULPHQWRVDEDL[RH[HFXWDGRVFRQVHFXWLYDPHQWH
, 8PD SHoD SROLGD GH FREUH PHWiOLFR p FRPSOHWDPHQWH PHUJXOKDGD HP XP EpTXHU TXH FRQWpP XPD
VROXomRDTXRVDFRQFHQWUDGDGHVXOIDWRGH]LQFRHWDPEpPDSDUDVSROLGDVGH]LQFRPHWiOLFRQRIXQGRGR
EpTXHU$SHoDSHUPDQHFHFRPSOHWDPHQWHPHUJXOKDGDQDVROXomRHHPFRQWDWRFRPDVDSDUDVGH]LQFR
HQTXDQWRDVROXomRpPDQWLGDHPHEXOLomRGXUDQWHPLQXWRV$SyVWUDQVFRUULGRHVVHWHPSRDSHoDGH
FREUHDGTXLUHXPDFRORUDomRSUDWHDGD
,, $VHJXLUDSHoDGHFREUHFRPFRORUDomRSUDWHDGDpUHPRYLGDGREpTXHUHQ[DJXDGDFRPiJXDGHVWLODGD
HFRORFDGDHPXPIRUQRDƒ&SRUGH]PLQXWRVDGTXLULQGRXPDFRORUDomRGRXUDGD
&RPEDVHQHVVHVH[SHULPHQWRV
D  H[SOLTXHRIHQ{PHQRTXtPLFRTXHSURYRFDDPXGDQoDGHFRORUDomRGDSHoDGHFREUHQRLWHP,
E  H[SOLTXHRIHQ{PHQRTXtPLFRTXHSURYRFDDPXGDQoDGHFRORUDomRGDSHoDGHFREUHQRLWHP,,

4XHVWmR2WHWUDHWLOFKXPERHUDDGLFLRQDGRjJDVROLQDQDPDLRULDGRVSDtVHVDWpFHUFDGH
D  (VFUHYDDHTXDomRTXtPLFDEDODQFHDGDTXHUHSUHVHQWDDUHDomRGHFRPEXVWmRGRFRPSRVWRWHWUDHWLOFKXPER
FRQVLGHUDQGRTXHRFKXPERHOHPHQWDUpR~QLFRSURGXWRIRUPDGRTXHFRQWpPFKXPER
E  2 8GHFDLD 3EFRPWHPSRGHPHLDYLGDGH[DQRV8PDDPRVWUDGHVHGLPHQWRFROKLGDHP
FRQWLQKDPJGH8HPJGH3E$VVXPLQGRTXHWRGRR3EpIRUPDGRVRPHQWHSHOR
GHFDLPHQWRGR8HTXHR3EQmRVRIUHGHFDLPHQWRHVWLPHDLGDGHGRVHGLPHQWR
F  -XVWLILTXHRUHVXOWDGRREWLGRQRLWHPE VDEHQGRTXHDLGDGHGR8QLYHUVRpGHELOK}HVGHDQRV
'DGRVOQ OQ 


AS QUESTÕES NUMÉRICAS DEVEM SER DESENVOLVIDAS SEQUENCIALMENTE ATÉ O FINAL.

Questão 1. Considere reações de combustão do etanol.


a) Escreva a equação química balanceada para a reação com oxigênio puro.
b) Escreva a equação química balanceada para a reação com ar atmosférico.
c) Escreva a equação química balanceada para a reação com 50% da quantidade estequiométrica de ar
atmosférico.
d) Classifique as reações dos itens a), b) e c) em ordem crescente de variação de entalpia reacional.

Questão 2. Uma determinada quantidade de um composto A foi misturada a uma quantidade molar três
vezes maior de um composto B, ou seja, A + 3B. Essa mistura foi submetida a dois experimentos de
combustão (I e II) separadamente, observando-se:
I. A combustão dessa mistura A + 3B liberou 550 kJ de energia.
II. A combustão dessa mistura A + 3B, adicionada de um composto C em quantidade correspondente
a 25% em mol do total da nova mistura, liberou 814 kJ de energia.
Considerando que os compostos A, B e C não reagem entre si, determine os valores numéricos
a) da quantidade, em mol, de A, B e C.
b) do calor de combustão, em kJ mol−1, do composto C, ∆Hc(C).
Dados: ∆Hc(A) = −700 kJ mol−1; ∆Hc(B) = −500 kJ mol−1.

Questão 3. Considere uma porção de uma solução aquosa de um eletrólito genérico AB, em formato de um
cilindro de 2 cm de diâmetro e 314 cm de comprimento, cuja concentração seja de 1,0 x 10−2 mol L−1.
Sabendo que a resistência elétrica dessa porção é de 1,0 x 104 ohm, calcule a sua condutividade molar em
S cm2 mol−1.

Questão 4. Uma solução aquosa de água oxigenada a 3% (v/v) foi adicionada a soluções aquosas ácidas em
dois experimentos diferentes. Foram observados:
I. No primeiro experimento: a adição a uma solução aquosa ácida de permanganato de potássio
resultou na perda da coloração da solução, tornando-a incolor.
II. No segundo experimento: a adição a uma solução aquosa ácida de iodeto de potássio inicialmente
incolor resultou em uma solução de coloração castanha.
Com base nas observações experimentais, escreva as reações químicas balanceadas para cada experimento e
indique os agentes oxidantes e redutores em cada caso, quando houver.

Questão 5. Classifique cada uma das substâncias abaixo como óxidos ácido, básico ou anfótero.
a) SeO2
b) N2O3
c) K2O
d) BeO
e) BaO

2
Questão 6. Considere a seguinte reação genérica, nas condições padrão e a 25 °C:

2A3 + + 2B−  2A2+ + B2

Determine a constante de equilíbrio dessa reação a 25 °C, sabendo que os valores dos potenciais de eletrodo
padrão de semicélula das espécies envolvidas são iguais a + 0,15 V e − 0,15 V.

Questão 7. Uma solução comercial de HCl é vendida com 37% (em massa) de HCl em água. A densidade
dessa solução de HCl é de 1,15 g cm−3.
a) Considerando que o HCl se dissocia completamente, determine o pH dessa solução comercial.
b) O valor do pH determinado no item a) possui significado físico? Justifique.

Questão 8. Considere as variações de entalpia de processo abaixo tabeladas.


Processo ∆H (kJ mol−1)
Ionização do Na0 495,8
Energia de ligação Cl-Cl 242,6
Entalpia de vaporização do Na 0
97,4
Afinidade eletrônica do Cl −349
Entalpia de rede do NaCl −787
a) Esboce o diagrama de Born-Haber para a formação do NaCl(s) a partir de Na0(s) e Cl2(g) e calcule
a variação de entalpia de formação do NaCl(s).
b) Sabe-se que o valor absoluto (em módulo) da entalpia de rede do CaO(s) é maior do que a do
NaCl(s). Explique por quê.

Questão 9. A figura ao lado mostra a estrutura básica de um triacilglicerídeo, em que R, O

R’ e R’’ representam cadeias carbônicas, saturadas ou insaturadas, com pelo menos oito H2C O C R

átomos de carbono. Sabe-se que o triacilglicerídeo pode reagir tanto por transesterificação O

(reação A) quanto por hidrólise básica (reação B). Em ambos os casos, um produto HC O C R'

comum dessas reações pode ser usado na produção de nitroglicerina (reação C). Com base O

nessas informações, escreva as equações que descrevem as reações A, B e C. H 2C O C R''

Questão 10. Considere a reação genérica 2A(g)  B(g). No instante inicial, apenas o reagente A está
presente. Sabendo que a reação direta é exotérmica, construa os gráficos de concentração de cada
substância em função do tempo de reação para as seguintes condições:
a) desde o instante inicial até o equilíbrio, na presença e na ausência de catalisador.
b) a partir do equilíbrio inicial, com um rápido aumento da temperatura, até um novo equilíbrio.
c) a partir do equilíbrio inicial, com um rápido aumento da pressão, até um novo equilíbrio.
d) a partir do equilíbrio inicial, com a remoção rápida de 50% do produto B, até um novo equilíbrio.

3
QUÍMICA

AS QUESTÕES NUMÉRICAS DEVEM SER DESENVOLVIDAS SEQUENCIALMENTE ATÉ O


FINAL.

Constantes
Constante de Avogadro (NA) = 6,02 x 1023 mol−1
Constante de Faraday (F) = 9,65 x 104 C mol−1 = 9,65 x 104 A s mol−1 = 9,65 x 104 J V−1 mol−1
Volume molar de gás ideal = 22,4 L (CNTP)
Carga elementar = 1,60 x 10− C
19

Constante dos gases (R) = 8,21 x 10−2 atm L K−1 mol−1 = 8,31 J K−1 mol−1 = 1,98 cal K−1 mol−1
Constante gravitacional (g) = 9,81 m s−2
Constante de Planck (h) = 6,63 x 10−34 m2 kg s−1
Velocidade da luz no vácuo = 3,0 x 108 m s−1
Número de Euler (e) = 2,72

Definições
Pressão:1 atm = 760 mmHg = 1,01325 x 105 N m−2 = 1,01325 bar
Energia:1 J = 1 N m = 1 kg m2 s−2 = 6,24 x 1018 eV
Condições normais de temperatura e pressão (CNTP): 0° C e 760 mmHg
Condições ambientes: 25° C e 1 atm
Condições padrão:1 bar; concentração das soluções = 1 mol L−1 (rigorosamente: atividade unitária das
espécies); sólido com estrutura cristalina mais estável nas condições de pressão e temperatura em
questão.
(s) = sólido. (ℓ) = líquido. (g) = gás. (aq) = aquoso. (conc) = concentrado. (ua) = unidades arbitrárias.
u.m.a. = unidade de massa atômica. [X] = concentração da espécie química X em mol L−1
ln X = 2,3 log X

Massas Molares
Elemento Número Massa Molar Elemento Número Massa Molar
Químico Atômico (g mol ) Químico Atômico (g mol )
-1 -1

H 1 1,01 S 16 32,06
C 6 12,01 K 19 39,10
N 7 14,01 Cr 24 52,00
O 8 16,00 Fe 26 55,85
Na 11 22,99 Zn 30 65,38
Cl 17 35,45 I 53 126,90

Questão 1. Para uma reação reversível de uma etapa 2A+B⇌C+D, a constante de velocidade para a
reação direta, k1, é de 406 L mol−1 min−1, e a constante de velocidade para a reação inversa, k−1, é de
244 L mol−1 min−1. A energia de ativação para a reação direta é de 26,2 kJ mol−1 (Ea,direta), e para a
reação inversa é de 42,4 kJ mol−1 (Ea,inversa).

a) Desenhe um diagrama de energia para essa reação, apresentando os valores de (i) ∆E, (ii) Ea,d, e
(iii) Ea,i.
b) Discuta o efeito de elevação da temperatura na constante de velocidade direta (k1) e inversa
(k-1).
c) Calcule a constante de equilíbrio (K) e descreva o efeito de elevação de temperatura.
3
Questão 2. Os biodigestores possibilitam o reaproveitamento de detritos convertendo material orgânico
em metano, que é utilizado como combustível em sistemas de geração de energia. Um laticínio utiliza a
queima do metano para aquecer 1 m3/h de água, de 25 ºC a 100 ºC em uma caldeira que opera a
1 atm. Sabendo-se que 25 % do calor produzido no processo é perdido e que, nessas condições, a
combustão completa do metano produz água líquida, determine

a) a entalpia molar da combustão do metano;


b) a taxa de calor necessária para aquecer a água;
c)
a vazão de metano, em kg/h, que deve alimentar a caldeira.

Dados: ∆Hfº(CH4(g)) = −17,9 kcal mol−1; ∆Hfº(CO2(g)) = −94,1 kcal mol−1; ∆Hfº(H2O(g)) = −57,9 kcal mol−1;
∆Hebº(H2O(l)) = −10,5 kcal mol−1; cpº(H2O(l)) = 1 cal g−1 ºC−1; ρ(H2O(l)) = 1 g cm−3

Questão 3. A obtenção de biodiesel a partir de óleos vegetais (triacilgliceróis) é uma alternativa para a
produção de combustíveis menos poluentes, sendo possível catalisar a reação com um ácido ou uma base.
Escreva a equação química balanceada que representa a reação

a) de obtenção de triacilglicerol a partir de glicerol e ácido graxo com cadeia alquílica representada
por R1.
b) de obtenção de biodiesel a partir do triacilglicerol obtido em (a) e etanol.
c) paralela e indesejada que poderia ocorrer se, na reação descrita em (b), fosse utilizado hidróxido
de sódio como catalisador, tendo também a presença de água na reação.

Questão 4. Uma barra de zinco foi soldada a um tubo de ferro fundido para protegê-lo contra a
corrosão, estando ambos enterrados no solo. Sabendo que uma corrente constante de 0,02 A escoa entre
os dois, responda:

a) Qual é a semirreação que ocorre na superfície da barra de zinco?


b) Como a reação descrita em (a) atua para proteger o ferro contra corrosão?
c) Como se chama este sistema de proteção contra a corrosão?
d) Qual deve ser a massa do metal consumida em 10 anos?

Questão 5. A partir do isótopo ocorrem três processos sucessivos de decaimento radioativo que
levam à formação do isótopo final D. A partir de há emissão de uma partícula beta, produzindo o
nuclídeo B. Este, por sua vez, libera uma partícula beta formando o nuclídeo C. O nuclídeo D é
produzido a partir de C por meio de emissão de uma partícula alfa. Escreva as equações nucleares dessas
três etapas, fornecendo os números de massa e atômico dos nuclídeos B, C e D em função de x e y.
Esboce um gráfico da quantidade de cada nuclídeo em função do tempo até a produção de D e o
consumo de todos os demais nuclídeos. Considere que a constante de velocidade é a mesma em todas as
etapas.

4
Questão 6. A reação de isomerização do cis-2-buteno para formar o isômero trans-2-buteno, que é mais
estável por 4 kJ mol−1, ocorre em fase gasosa em uma única etapa com energia de ativação de
264 kJ mol−1. Essa reação ocorre de forma muito mais rápida quando assistida por iodo molecular em fase
gasosa como catalisador. A lei de velocidade da reação catalisada é dada por

velocidade k cis 2 buteno I

O mecanismo proposto para a reação catalisada é baseado em cinco etapas:

I. As moléculas de iodo se dissociam para formar átomos de iodo com energia de dissociação igual a
75 kJ mol−1;
II. Um dos átomos de iodo é adicionado a um dos átomos de carbono que tem ligação dupla, quebrando
essa ligação para formar uma ligação simples C-C. O sistema molecular formado encontra-se a
118 kJ mol−1 acima dos reagentes;
III. Uma das extremidades da molécula sofre torção livre em relação à outra extremidade. A energia do
sistema molecular após a torção continua a 118 kJ mol−1 acima dos reagentes;
IV. O átomo de iodo ligado ao carbono dissocia-se do sistema molecular intermediário e a ligação dupla é
formada novamente no isômero trans. Esse processo libera 47 kJ mol−1 de energia;
V. Os átomos de iodo se recombinam para formar o iodo molecular, liberando 75 kJ mol−1 de energia.

Baseado nessas informações:

a) esboce em uma mesma figura os perfis de energia para a reação de isomerização do cis-2-buteno
com e sem a presença de catalisador. Deixe claro, usando diferentes notações, os dois perfis e os
valores das energias envolvidas;
b) escreva as reações químicas que ocorrem em cada etapa da reação catalisada para formar a reação
global.

Questão 7. Considere a conformação estrutural das moléculas 1,3-dietilcicloexano, 1,4-dietilcicloexano e


2,3 diclorobutano. Pedem-se:
a) Desenhe todas as estruturas conformacionais;
b) Determine o número de centros quirais em cada molécula;
c) Identifique todos os pares enantioméricos e os compostos meso, se presentes.

Questão 8. Dicromato de potássio, enxofre e água reagem produzindo hidróxido de potássio, óxido de
cromo III e dióxido de enxofre. Para oxidar 96 g de enxofre, são utilizados 50% de dicromato de potássio
em excesso. Sabendo que o rendimento da reação é de 80%, determine:
a) a equação balanceada da reação química;
b) a massa de dicromato de potássio utilizada;
c) a massa de dióxido de enxofre produzida.

5
Questão 9. A produção de borrachas e espumas é comumente realizada pela síntese de poliuretanos.
Para tal produção, a polimerização ocorre a partir de um poliol e um isocianato.
a) Apresente a(s) reação(ões) químicas da polimerização e formação de poliuretano a partir de um
diol e um diisocianato.
b) A água, quando presente no meio, gera reação(ões) paralela(s) e é determinante na produção de
espumas. Apresente essa(s) reação(ões).

Questão 10. Considere a titulação de um ácido por meio da adição de uma base. Calcule o pH inicial e
o pH no ponto de equivalência e construa a curva de titulação, ou seja, o gráfico do pH em função da
porcentagem de ácido neutralizado. Apresente os cálculos realizados para os três casos. Dados
eventualmente necessários: log2 = 0,3; √2=1,4; log1,4 = 0,14.

a) Ácido forte (HCl, 0,1 mol L− ) com uma base forte (NaOH, 0,1 mol L− );
1 1

b) Ácido forte (HCl, 0,2 mol L-1) com uma base fraca hipotética (XOH, 0,2 mol L−1;

Kb(XOH) = 1,0 x 10− );


5

c) Ácido fraco hipotético (HZ, 0,2 mol L-1; Ka(HZ) = 1,0 x 10−5) com uma base forte

(NaOH, 0,2 mol L−1).

Вам также может понравиться